NR 603 WEEK 1

Ace your homework & exams now with Quizwiz!

Parents of a 6-month-old infant are very worried about a severe reaction that occurred in the infant after the first dose of DTaP. The infant had a high fever (up to 105ºF), was irritable, and was crying incessantly for a few hours following the injection. Parents do not want to give any more doses of DTaP to their child. Question What adverse reaction is a contraindication for giving the next dose of DTaP in this infant? Answer Choices 1 Persistent inconsolable crying for 3 or more hours 2 Encephalopathy (coma or longer or multiple seizures) within 7 days of dose 3 Fever of more than 105º F within 48 hours of dose 4 Seizure within 3 days after dose 5 Collapse or shock like state within 48 hours of dose ANS;2

2 Encephalopathy (coma or longer or multiple seizures) within 7 days of dose If the infant develops signs of encephalopathy (coma or diminished level of consciousness, prolonged or multiple seizures not attributable to another identifiable cause, within 7 days of giving a dose of DTaP or DTP), it is a contraindication for subsequent doses of DTaP. Acute neurological illness after receiving DPT can have chronic neurological sequelae. The National Childhood Encephalopathy Study (NCES) data also are consistent with a possibility that some children with underlying brain or metabolic abnormality can have acute neurologic illness triggered by DPT, which might go to chronic nervous system dysfunction, which further affects the physical, social, behavioral, and educational outcome of children. Therefore, encephalopathy is a contraindication for subsequent doses of DTaP. If the infant has progressive or unstable neurologic disorder, including infantile spasms, uncontrolled seizures, or progressive encephalopathy, defer vaccination with DTaP or TdaP until the condition is stabilized. If the infant develops a fever of 105º or above following a dose of DTaP, it is not a contraindication for subsequent dose of DTaP. Risk and benefits of the vaccine should be carefully considered. Consider giving acetaminophen before and every 4 hours thereafter for 24 hours after giving the vaccine. Collapse or shock-like state, i.e., hypotonic hyporesponsive episode, within 48 hours after receiving a previous dose of DTP or DTaP is not a contraindication, but it is important to consider the benefits and the risks. If the benefits are believed to outweigh the risk (e.g., during an outbreak or foreign travel), give the vaccine under careful observation in a facility where anaphylaxis can be recognized and managed. Seizure within 3 days of a dose of DTaP is not a contraindication, but needs precaution and a consideration of the risks and the benefits. If the benefits outweigh the risk (e.g., during an outbreak or foreign travel), the vaccine should be given under observation. Persistent inconsolable crying lasting for 3 or more hours occurring within 48 hours of dose is not a contraindication, but it is important consider the benefits and the risks of this vaccine under the circumstances. If the benefits outweigh the risk (e.g., during an outbreak or foreign travel), give the vaccine.

A 36-year-old woman presents with fatigue and diplopia; symptoms started in the morning and worsened during her time at work. On examination, the patient has ptosis bilaterally and decreased ocular muscle power. A CT scan of the chest shows a thymoma. What is the pathophysiology of this patient's neuromuscular disorder? Answer Choices 1 A genetic defect resulting in the production of atypical, or faulty, plasma cholinesterase 2 An autoimmune response that initially blocks, then destroys, nicotinic receptors at the neuromuscular junction 3 An increase in the ratio of muscle fibers to motor nerve fibers, resulting in production of a motor unit (MU) that provides less control over muscle actions 4 Production of IgA antibodies against acetylcholine receptors of the motor end plates of skeletal muscle 5 Genetic defect resulting in the deficiency of dystrophin

An autoimmune response that initially blocks, then destroys, nicotinic receptors at the neuromuscular junction Myasthenia gravis is a neuromuscular disorder caused by immune mediated loss of acetylcholine receptors. It is associated with thymic hyperplasia in 65% of the cases and with thymoma in 15% of the cases. Myasthenia gravis (MG) is an autoimmune disorder affecting the neurochemical transmission of information between motor nerve fibers and skeletal muscle fibers. In this disorder, nerve impulses are transmitted in a normal fashion to the axon terminals, and skeletal muscle is capable of contracting normally. The dysfunction occurs at the interface between the 2 components of the neuromuscular junction (NMJ). In cases of MG, an autoimmune response causes antibodies of the IgG (not IgA) class to be produced against the nicotinic receptors of the NMJ. The antibodies fix onto the nicotinic receptors, block acetylcholine (ACh) from complexing with the receptors, and ultimately bring about destruction of the receptors. The patient may present with ophthalmic or non-ophthalmic manifestations. The ocular disturbances mainly include ptosis and diplopia; the non-ophthalmic symptoms include difficulty in talking, swallowing, or chewing and upper limb and trunk weakness. Anti-acetylcholine receptor antibodies assay is positive in 90% of the patients with generalized MG, whereas it is positive in 50-70% in ocular MG. Other tests include the Tensilon test, imaging studies such as CT or MRI scanning, repetitive nerve stimulation, and single fiber electromyography. Medical treatment includes neostigmine therapy, plasmapheresis, or intravenous immunoglobulin therapy. Thymectomy may be considered in cases of thymoma. The reduction in receptor population decreases neurotransmission, and the result is that muscle depolarization is not produced. The quantity and quality of cholinesterase enzymes is unaffected by the disorder, and so is any alteration in the composition of motor units at the neuromuscular junctions. A genetic disorder associated with deficiency of the structural protein dystrophin in muscles occurs in Duchenne muscular dystrophy, and it is not seen in myasthenia gravis.

A 54-year-old woman has suffered a stroke that has resulted in dramatic changes to her personality, left leg and foot weakness, loss of sensation in the left leg, and apathy. Which of the following arteries was most likely affected by the stroke? A left anterior cerebral B left middle cerebral C right anterior cerebral D right middle cerebral E right posterior cerebral ANS C

C. the RAC The anterior cerebral arteries supply the frontal lobes as well as the medial aspects of the parietal and occipital lobes rostral to the parietooccipital sulcus. The prefrontal cortex of the frontal lobe is concerned with a person's personality, depth of feeling, and initiative. Hence, occlusion of an anterior cerebral artery can cause neuronal injury to this area, leading to feelings of apathy and personality changes. The paracentral lobule represents the medial aspects of the precentral gyrus (frontal lobe) and postcentral gyrus (parietal lobe), which are responsible for motor control and somatosensory perception, respectively, of the leg and foot. Hence, occlusion of an anterior cerebral artery can produce contralateral hemiparesis and hemisensory loss involving the leg and foot. With the 54-year-old patient, the symptoms were occurring on the left side, which points to a right anterior cerebral artery occlusion.

A 45-year-old secretary presents with a 1-month history of paresthesias on the medial aspect of her right hand. She has no neck pain or shoulder pain, but she has had some difficulty typing with the right hand. Neurological exam reveals diminished sensation to pain and light touch on the medial aspect of the palmar and dorsal surfaces of the hand to the wrist, the 5th finger, and the medial aspect of the ring finger. Mild weakness of the right abductor minimi digiti is present, but the other intrinsic hand muscles are of normal strength. The remainder of the neurological exam is normal. Question What condition does patient have? Answer Choices 1 Compressive ulnar neuropathy at the elbow 2 Compressive median neuropathy at the wrist 3 Compressive radial neuropathy at the mid-humerus 4 Compressive C8-radiculopathy 5 Compression of axillary nerve ANS:1

Compressive ulnar neuropathy at the elbow This patient has a right ulnar neuropathy, which is most likely due to nerve compression at the elbow. The abnormalities on neurologic exam are limited to weakness of the right abductor digiti minimi, which is innervated by the ulnar nerve, and a sensory disturbance in the distribution of the ulnar nerve, which includes splitting of the 4th finger. The ulnar nerve exits the brachial plexus as the terminal branch of the medial cord. The nerve then descends through the axilla and upper arm, after which it traverses the groove ("ulnar groove") between the medial epicondyle of the lower humerus and the olecranon of the ulnar bone. The nerve runs through the forearm deep to the flexor carpi ulnaris muscle; it exits the wrist through an anatomic space distinct from the carpal tunnel, which is where the median nerve is located. Ulnar nerve compression can occur in the axilla, elbow, or wrist. Occupational injury is most often located in the elbow because of the nerve's proximity to the bony surfaces; therefore, it is susceptible to external compression (e.g., in persons who lean their elbow against a hard surface). In addition, work-related repetitive flexion and extension of the elbow causes narrowing of the ulnar groove, and it results in ulnar nerve entrapment. The diagnosis can be made by radiographs of the elbow, neck, chest, and wrist. Definitive diagnosis is made by electromyography and nerve conduction tests. Conservative treatment can be tried; it should consist of elbow rest and padding, non-steroidal anti-inflammatory drugs (NSAIDs), and vitamin B-6 supplements. If the paresthesias worsen, surgical decompression coupled with transposition can be performed, depending on the site of entrapment. Median neuropathy (carpal tunnel syndrome) characteristically produces a sensory disturbance of the palmar aspect of the hand, thumb, 2nd and 3rd fingers, and the lateral aspect of the 4th finger. Radial neuropathy produces weakness of wrist extension. A C8-radiculopathy produces weakness of all intrinsic hand muscles and does not cause splitting sensory loss of the 4th finger. Radial neuropathy does not cause weakness of the abductor digiti minimi. When there is axillary nerve entrapment, the patient complains of weakness and numbness of the shoulder, with signs of deltoid atrophy.

A 12-month-old girl presents with her parents after a 3-day history of intermittent episodes of strange behavior. A neurologist is consulted because the parents are concerned she is having some form of seizure activity. Her health history includes 2 episodes of otitis media, but she is otherwise healthy. Her initial vital signs and physical exam by the emergency room staff are all normal. Question What description of the infant's strange behavior would lead the neurologist to suspect a diagnosis of simple partial seizures? Answer Choices 1 Facial and right arm twitching lasting about a minute, then loss of consciousness 2 Blank stare and impaired awareness for 10-20 seconds 3 Eye deviation with facial twitching that lasts 1-2 minutes 4 Loss of consciousness, followed by stiffening of the body and extremities 5 Very brief periods of whole body going limp, followed by a brief loss of consciousness ANS:3

Correct Answer: Eye deviation with facial twitching that lasts 1-2 minutes Eye deviation with facial twitching that lasts 1-2 minutes' is the correct answer. Simple partial seizures can vary depending on what portion of the cortex has been effected. Patients may display isolated tonic or clonic activity or may have transient altered sensory perception. Clonic movements are jerks that may involve a specific body part or may spread to adjacent body parts, known as "Jacksonian march." Tonic activity is stiffening or sustained posturing, such as eye deviation in this case. Sensory seizures can involve illusions or hallucinations, but this would be difficult to diagnose in a young child. One important hallmark of a simple partial seizure is that the patient does NOT lose consciousness. 'Facial and right arm twitching lasting about a minute then loss of consciousness' is not the correct answer. While a patient having a simple partial seizure might experience facial and right arm twitching for that period of time, they are not going to have loss of consciousness. The type of seizure involved in this answer choice best describes a complex partial seizure. 'Blank stare and impaired awareness for 10-20 seconds' is not the correct answer. This description is best used to describe a patient who is having a nonconvulsive generalized seizure, otherwise known as an absence seizure. Patients having an absence seizure will rarely have any movement during the seizure. At most, they may have some minor motor activity, such as blinking. In an older child or adult, these episodes will leave the patient feeling as though they have short periods of time that they do not remember. 'Loss of consciousness followed by stiffening of the body and extremities' is not the correct answer. The fact that this description includes loss of consciousness is what excludes it from being a description of a patient having a simple partial seizure. This scenario better describes a person who is having a generalized tonic-clonic seizure. 'Very brief periods of whole body going limp followed by brief loss of consciousness' is not the correct answer. The fact that this description includes loss of consciousness is what excludes it from being a description of a patient having a simple partial seizure. This scenario better describes a person who is having an atonic seizure, which is characterized as an abrupt, brief loss of body tone, possibly followed by brief loss of consciousness.

A 19-year-old man presents with a sudden onset of vomiting, headache, vision changes, numbness and tingling of extremities, difficulty speaking, difficulty writing, loss of coordination, and loss of balance. His past medical history is significant for sickle cell anemia. Physical exam findings are consistent with acute cerebrovascular insufficiency. Early detection and prevention of stroke in sickle cell anemia is now possible by using screening tests. What imaging modality is recommended as a screening test for cerebrovascular disease/stroke in sickle cell disease (SCD) patients? Answer Choices 1 Magnetic resonance imaging (MRI) 2 Computed tomography (CT) skull 3 X-ray skull 4 Neurocognitive testing 5 Transcranial Doppler ultrasonography (TCD)

Correct Transcranial Doppler ultrasonography (TCD) Stroke or cerebrovascular insufficiency occurs in about 11% of patients under 20 years of age with sickle cell anemia. The major symptom includes sudden hemiparesis with or without aphasia. Most of these strokes occur in the distribution of the intracranial internal carotid artery or the middle cerebral artery. The Stroke Prevention Trial in Sickle Cell Anemia (STOP Trial) confirmed that TCD can identify children with SCD at high risk for first-time stroke. As the greatest risk of stroke occurs in early childhood, it is recommended that children aged 2 - 16 years receive TCD screening. Stroke in SCD is a medical emergency and requires mandatory exchange transfusion followed by maintenance hypertransfusion. Studies have shown that about 90% of strokes could be prevented in children with stenotic cranial-artery lesions as demonstrated on transcranial Doppler ultrasonography if they are maintained on a regular program of chronic transfusion. The chronic transfusion should be such that it suppresses erythropoiesis so that no more than 30% of the circulating red cells are their own; this is done in order to maintain the Hb S level below 30%. Computed tomography (CT) and magnetic resonance imaging (MRI) are used for diagnosing strokes, but they are not used as a screening modality. Patients with poor educational performance should undergo neurocognitive testing, as they may have early ischemia of the central nervous system. It is recommended that children with sickle cell disease be screened for cerebrovascular disease with TCD and not MRI, CT, X-ray skull, or neurocognitive testing.

A 12-year-old girl presents with a 3-day history of progressive dysarthria, dysphagia, and weakness. The patient was well until 3 days ago, when she developed the onset and subsequent gradual worsening of dysarthria. She attributed this to a sore throat, which she had about 2 weeks before. Three days prior to presentation, she had the onset of mild dysphagia, which occurred primarily with liquids. 24 hours prior to presentation, she developed weakness in both upper extremities; the weakness worsened and began to involve the lower extremities. This limb weakness was not worsened by activity, but it was not improved by rest. She developed tingling in her toes 24 hours prior to presentation. When she became unable to walk without assistance (on the day of presentation), she decided to seek medical care. Past medical history is significant for measles and mumps. Because of family religious beliefs, she has not had any immunizations. Physical examination reveals a well-developed, well-nourished girl. She is awake, alert, cooperative, and in no acute distress. Temperature is 98.7 F by mouth, blood pressure is 140/80 mm Hg, heart rate is 84/min and regular, and respirations are 22/min and unlabored. Signs of trauma include multiple scratches and abrasions over most of her extremities; they are in varying stages of healing. Speech is moderately dysarthric. She experiences some mild choking when she tries to drink a glass of water. She can smile weakly, but cannot raise her eyebrows against resistance. She shows mild bilateral weakness of eye adduction. Pupillary responses are normal. There is mild to moderate upper extremity and mild lower extremity weakness, greater distally than proximally. Her motor strength is sustained over at least 30 seconds without fatigue. Reflexes are hypoactive to absent, and the response to plantar stimulation is downgoing bilaterally. Sensation is intact, except for mildly impaired position and vibratory sensation in both feet. A complete blood count, chemistry profile, chest X-ray, and EKG are all found to be normal. Computed tomography of the brain, with and without contrast, is negative. Question What is the most likely diagnosis? Answer Choices 1 Tetanus 2 Diphtheria 3 Guillain-Barre Syndrome 4 Botulism 5 Multiple sclerosis 6 Myasthenia Gravis ANS:3

GBS Guillain-Barre is the most common cause of acute and subacute generalized paralysis. In 60% of cases, it is preceded by mild respiratory or gastrointestinal infection 1-3 weeks before the onset of paralysis (1). Although it more typically begins distally and spreads proximally, the pattern seen here is a well-known variant. While classically described as a motor neuropathy, varying degrees of sensory neuropathy can be seen, as evidenced here by her paresthesias and decreased position and vibration sensation. Reduced or absent deep tendon reflexes are usual findings. Oculomotor nerves are paralyzed in only the most severe of cases; even the pupils may be unreactive. Diphtheria of the faucial-pharyngeal type produces an inflammatory exudate in the throat and trachea. Neurological sequelae usually begin after about 5-12 days, however, and usually with soft palate paralysis, which may produce dysphagia but also regurgitation and nasal (dysphonic) speech (1). Soon, trigeminal, facial, vagus, and hypoglossal nerves become involved. Loss of accommodation and blurring of vision (with preserved light reaction) usually appears by weeks 2-3. External ocular muscles are rarely involved. While cranial nerve findings may resolve around this time, the patient may go on to develop a mixed sensorimotor peripheral neuropathy at 5-8 weeks into the clinical course. Although this peripheral neuropathy may resemble Guillain-Barre, the other symptoms mentioned above and this long time delay, are not seen in this patient. Botulism is a rare form of blood borne illness caused by the exotoxin of C. botulinum (1). This exotoxin acts at the presynaptic endings of neuromuscular junctions, interfering with acetylcholine release. Outbreaks of botulism are usually due to ingestion of home preserved foods, especially vegetables and cured ham (1). There is no indication of an outbreak in this case. Rarely, botulism can be due to contaminated wounds (she played in horse fields and had multiple skin lesions). Symptoms usually come on within 12-36 hours (not unlike this patient), but they usually consist of nausea, vomiting, and anorexia. Generally, the first neurologic symptoms are blurring of vision and diplopia. These symptoms are rapidly followed by other cranial nerve findings, weakness of neck, trunk, upper then lower extremity weakness, and respiratory insufficiency. While the clinical course of botulism may be confused with Guillain-Barre, sensory loss would not be expected in botulism. Multiple Sclerosis of an acute and fulminant nature could be considered. The symptoms did come on rather suddenly, but their clear and rapid progression, with all symptoms being bilateral and symmetric, would be most unusual. While a negative computed tomographic scan does not fully rule out MS, in a patient with such dramatic and extensive bulbar findings, some brainstem or bilateral cerebral computed tomogram lesions would be expected. For the same reason, normal pupils would be most unusual. Also, with such widespread findings, Babinski signs would be expected. The deep tendon reflexes, if changed, would be increased. Tetanus might come to mind in a patient with skin injuries associated with playing in fields contaminated by horse feces (a common source of Clostridium tetani), who has had no tetanus immunizations, and who has the onset of bulbar symptoms first, with subsequent descending motor symptoms. However, while all the other listed diseases produce weakness, tetanus produces a progressively severe motor irritability that usually begins in the jaw with trismus. In some cases, it begins with a feeling of stiffness in the jaw and neck, slight fever, aches, and pains. This progresses to spasm as it spreads to other cranial nerve muscles, the neck, trunk, and extremities. This proceeds to tonic rigidity and tonic spasms that can be spontaneous or induced by the slightest stimulation. Certainly, sensory symptoms or signs would not be expected. Deep tendon reflexes would be increased. Myasthenia Gravis can be considered since the patient appears to a have a motor weakness that began in bulbar musculature then spread distally (distended). This pattern of weakness can be seen in myasthenia gravis, but it is also seen in a relatively common variant of Guillain-Barre. However, in myasthenia gravis, the weakness is more proximal than distal, which is the opposite of the more distal severity expected in neuropathies like Guillain-Barre. Myasthenic weakness tends to improve after rest and worsen with exercise, a pattern not seen in this patient. Again, sensory findings would not be expected in myasthenia; also, they would not be expected in a patient of this age.

A 25-year-old woman is seeking medical attention because she is concerned about a genetic disease that is running in her family. Both her father and her paternal grandfather are affected. Her 50-year-old father has suffered for several years with coordination difficulties, chorea, decline in cognitive ability, and aggressive outbursts. His symptoms are getting progressively worse. This patient has a 50% chance of developing what disorder? Answer Choices 1 Creutzfeldt-Jakob disease 2 Early-onset familial Alzheimer disease 3 Hallervorden-Spatz disease 4 Huntington's disease 5 Lesch-Nyhan disease ANS:4

Huntington Disease Huntington's disease is a progressive disorder that affects motor skills and cognition. It is caused by a CAG trinucleotide repeat expansion in the HD gene, located on chromosome 4p16. It is inherited in an autosomal dominant manner. The majority of affected individuals present with neurological manifestations; symptoms include coordination problems and involuntary movements. Oculomotor disturbances, dysarthria, and hyperreflexia occur early in the disease. It is characterized by a progressive deterioration of both voluntary and involuntary muscle control, cognitive decline, and psychiatric disturbances. The average age of onset is between 35 and 44 years. Median survival time is 15 to 18 years. Eventually, the patient becomes severely disabled due to a complete loss of motor control, muteness, and incontinence. Molecular genetic diagnosis is available. Creutzfeldt-Jakob disease is a subacute spongiform encephalopathy caused by a prion, which is a proteinaceous infectious particle. These infectious agents are thought to act by transforming normal protein molecules into deadly ones. This disorder can be inherited, transmitted through infection, or occur sporadically. Inherited cases (15% of CJD cases) show a mutation in the gene that codes for the normal prion protein. The disease is characterized by rapid and progressive dementia and neuromuscular symptoms. Initial symptoms include insomnia, personality or behavioral changes, problems with memory, incoordination, and vision disturbances. This is followed by dementia and myoclonus. Eventually, the patient becomes bedridden and enters a coma. Duration is usually 1 year or less. Average age of onset is between 50 and 75 years of age. Presently, there is no treatment. Early-onset familial Alzheimer disease is an adult onset progressive dementia. It is associated with cerebral cortical atrophy, β-amyloid plaques and neurofibrillary tangles. As the name implies, patients with this disorder have a positive family history. Mutations have been found in 2 genes: the amyloid precursor protein gene (APP), presenilin 1 (PSEN1), and presenilin 2 (PSEN2). Inheritance occurs in an autosomal dominant manner. Mean age of onset is less than 65 years. Initial symptoms are vague and include memory failure, confusion, language disturbance, agitation, and withdrawal. Progression of the disorder is slow, taking place over the course of years (mean range is 8-10 years). Alzheimer disease is the most common cause of dementia in North America and Europe. Eventually, the disease worsens, and death occurs due to malnutrition and pneumonia. Diagnosis is based upon the progressive dementia with the presence of β-amyloid plaques and neurofibrillary tangles. Molecular genetic testing is available for all 3 genes involved. Hallervorden-Spatz disease is a rare neurodegenerative disorder inherited in an autosomal recessive manner. It is characterized by a progressive rigidity that starts in the lower extremities and moves to the upper extremities. Rigidity may be accompanied by involuntary movements. Muscle tone and voluntary movement deteriorate progressively, leading to feeding difficulties and emaciation. During the late stages of the disorder, mental deterioration and progressive dementia are present. Average age of onset is the 1st or 2nd decade; mean survival time is approximately 10 years. Although the responsible gene has not been identified, it has been mapped to chromosome 20p13-p12.3. Diagnosis has generally been made postmortem. Lesch-Nyhan syndrome is a neurological disorder caused by overproduction of uric acid. It is caused by mutations in the hypoxanthine-guanine phosphoribosyl transferase (HPRT) gene located on chromosome Xq26.1. It is inherited in an X-linked recessive manner. Only men are affected (with exception). The syndrome is characterized by neurologic dysfunction, cognitive disturbances, behavioral disturbances, and the overproduction of uric acid. Initially, affected individuals show developmental delay and hypotonia, usually within the 1st year of life. Most never walk. Over the next few years, spasticity and hyperreflexia emerge, as does the hallmark sign of self-injurious behavior. This includes biting of the hands, lips, and cheeks, as well as head or limb banging. Survival into the 2nd or 3rd decade is common. The disorder can be diagnosed by molecular genetic testing.

In myasthenia gravis, weakness is a result of insufficient acetylcholine transmission at the neuromuscular junction; however, weakness can also occur with overdosing of the cholinergic medications used to treat myasthenia. What symptom helps differentiate a myasthenic crisis from a cholinergic crisis? Answer Choices 1 Respiratory failure 2 Bilateral ptosis 3 Muscle fasciculations 4 Diplopia 5 Normal muscle stretch reflexes ANS: 3

Muscle Fasiculations Signs of cholinergic overdosage include muscle fasciculation, rhinorrhea, lacrimation, salivation, increased bronchial secretions, nausea, or diarrhea. The presence of any of these suggests that the patient's weakness may be due to cholinergic crisis. The other signs are due to weakness and can occur in either condition.

During the examination of the infant, either for a well baby checkup or if ill, it is important to listen carefully to the infant's breathing and the quality of the cry. What might a shrill high-pitched cry indicate? Answer Choices 1 Cri-du-chat 2 Profound mental retardation 3 Congenital hypothyroidism 4 Tracheomalacia 5 Increased intracranial pressure

increased intracranial pressure ncreased intracranial pressure (ICP) may result from cerebral edema associated with head trauma, hypoxic-ischemic events, infection, metabolic disorders, hydrocephalus, and space-occupying lesions. Besides a high-pitched cry, infants may also present with widened sutures, a bulging fontanelle, persistent downward eye deviation (sunset eye sign), and increasing head circumference. If left unrecognized and untreated, increased ICP can result in significant morbidity and mortality by causing irreversible neurologic injury. Cri-du-chat syndrome is an autosomal deletion syndrome caused by a partial deletion of chromosome 5p. It is characterized by a distinctive mewing, catlike cry that occurs in infancy. This cry is due to structural abnormalities of the larynx and CNS dysfunction. In addition, affected individuals also display growth failure, microcephaly, facial abnormalities, and mental retardation. Profound mental retardation is suggested by the absence of any cry. Congenital hypothyroidism may result from absent or abnormal development, destruction, or failure of stimulation of the thyroid gland by the pituitary, and/or by defective or abnormal synthesis of thyroid hormones. Infants profoundly affected have a distinctive appearance that has been termed cretinism, which includes a puffy-appearing face, a dull look, a large protruding tongue, dry brittle hair, a low hairline, and jaundice. Feeding is poor, with possible choking; constipation is common. Cries are hoarse and infrequent, and the infants are sluggish and inactive. Muscle tone is decreased. If left untreated, it can lead to severe mental and growth retardation. If recognized and treated early, significant improvement can be seen. Tracheomalacia is the most common cause of expiratory stridor. It is caused by a defect of the cartilage, resulting in loss of rigidity or partial collapse of the tracheal cartilage with respiration. This causes an expiratory high-pitched, crowing noise. Tracheomalacia may also be found with lesions, such as vascular rings and slings, that compress the airway Most children outgrow this condition by the time they are 3 years-old.

A 1-year-old boy presents with increasing lethargy. He is barely responsive, and his parents deny any trauma or injury. What is the most common cause of nontraumatic altered levels of consciousness? Answer Choices 1 Seizure disorder 2 Diabetic ketoacidosis 3 Inborn errors of metabolism 4 Toxic ingestion 5 Infection ANS:5

infection Awareness of self and the surrounding environment or consciousness may be altered into different abnormal states of consciousness. Consciousness can shift from loss of clear thinking or confusion, usually accompanied by disorientation, to delirium, a succession of confused and unconnected ideas manifested in children as extreme mental and motor excitement, to lethargy, a profound type of slumber where movement or speech is limited, to stupor or deep sleep where arousal is achieved only by repeated vigorous stimuli, finally to coma, unresponsiveness to even painful stimuli. Non-traumatic coma is most common in infants and toddlers with another smaller peak of occurrence in adolescence. The most common cause of non-traumatic altered level of consciousness in children is infection of either the brain (encephalitis), meninges (meningitis), or both; infections account for more than 1/3 of cases. Prolonged seizures, anticonvulsive therapy, and postictal states can also lead to altered levels of consciousness. The most common metabolic cause of alteration of consciousness is diabetic ketoacidosis, which can occur at any age, but is most common in adolescence. Caused by severe insulin deficiency, hyperglycemia and ketogenesis lead initially to polyuria, polydipsia, hyperpnea, vomiting, and abdominal pain. As the process progresses, hyperosmolar dehydration and acid/base and electrolyte disturbances occur. Advanced stages alter level of consciousness and can lead to coma. Alterations of consciousness due to inborn errors of metabolism that present with electrolyte and glucose abnormalities typically present in infancy. The availability of gluconeogenic precursors or the functions of the enzymes required for production of hepatic glucose are affected. Metabolic defects causing hypoglycemia include glycogen storage disease, galactosemia, fatty acid oxidation defects, carnitine deficiency, several of the amino acidemias, hereditary fructose intolerance, and defects of other gluconeogenic enzymes. Toxic ingestion and exposure are very common in toddlers and adolescents, with a toddler's ability to explore his environment filled with often brightly colored medications and intentional ingestion by adolescents typically involving over-the-counter medications or psychotropic drugs such as antidepressants.

Your patient is a 77-year-old man complaining of stiffness and slowness that started several months ago and is more pronounced on the left side of his body. Neurological examination reveals tremor in rest, bradykinesia, cogwheel rigidity in his neck and limbs, and postural instability. His face is expressionless (mask-like) and he has a stooped posture; when he walks, there is mild retropulsion and no arm swings. Question What transient factor would you also expect to see? Answer Choices 1 Freezing 2 Titubation 3 Hemiparesis 4 Chorea 5 Paraparesis 6 Slapping ANS: 1

freezing your patient most probably has Parkinson disease or parkinsonism (tremor in rest, rigidity, bradykinesia, postural instability). Parkinsonian gait is characterized by the absence of arm swings, small shuffling steps, hypokinesia, and reduced walking speed. The patient has difficulty starting and stopping after starting. Freezing of gait is most often seen in the advanced stages of the disease. It is typically transient and lasts less than a minute, during which gait is halted and the patient feels that his/her feet are glued to the ground. When the patient overcomes the block, walking can be performed relatively smoothly. The most common form of freezing is "start hesitation", which is when the patient wants to start walking. It is provoked by placing a patient in a narrow space or in stressful situations, such as in the doctor's office. Later in the course of the disease, it can appear spontaneously.

A 16-year-old girl is brought to the emergency room for evaluation of continuous seizures. The patient was first diagnosed with idiopathic epilepsy at age 10. She was started on valproic acid. Phenobarbital and carbamazepine (Tegretol) have been tried without success, and the phenobarbital resulted in transient severe personality changes. 2 previous EEG's have been normal. This spell occurred abruptly at the family dinner table without apparent warning. Past medical history has been normal except for the usual childhood diseases without complications. 2 days prior to presentation, she saw her primary physician for nausea and vomiting; as he believed she had the flu, he started her on prochlorperazine (Compazine), which relieved her nausea. Physical exam reveals a well-developed, well-nourished young woman lying on a gurney; there is continuous but variable motor jerking of all 4 extremities. The jerking varies in intensity from side to side. The head intermittently turns from side to side. Eyes are closed tightly and cannot be passively opened. Old cutdown sites in the right antecubital fossa and left supraclavicular regions are noted. Deep tendon reflexes cannot be assessed. The response to plantar stimulation is withdrawal bilaterally. There are no signs of trauma to the head or elsewhere. Skin is warm and dry with normal color, and vitals are normal. The general medical exam is otherwise normal. Labs are done stat, and they include normal complete blood count and differential, normal chest X-ray, normal urinalysis on cathed specimen, normal chemistry profile, and normal arterial blood gases. Question Pseudoseizure (psychological rather than epileptic event) is suspected. What test would be of greatest value in this acute clinical setting in differentiating pseudoseizures from true epileptiform seizures? Answer Choices 1 Stat electroencephalogram (EEG) 2 Stat serum prolactin level 3 Stat creatine kinase (CK) 4 Stat referral to a comprehensive epilepsy center 5 Stat psychiatric referral ANS:1

stat EEG If an electroencephalogram can be obtained on a stat basis, it can be of immeasurable help in differentiating pseudoseizures from true seizures. During a spell suggesting generalized tonic-clonic seizure, the electroencephalogram should show generalized spike and wave activity; after the spell, it should show generalized slowing and disorganization lasting 30-90 minutes. Even if the active phase of the spell were over before the electroencephalogram, if done in this 30-90 minutes post-event time frame, it could be compared with a later electroencephalogram, which would probably be normal in case of pseudoseizure (5). If an electroencephalogram is normal during or immediately after this event, it is major evidence against epilepsy and for pseudoseizures. In view of the above, and with a history of 2 previous normal electroencephalograms, obtaining an outpatient electroencephalogram or one during subsequent admission would most likely be a waste of time and money. If a video camera were available, video recording the patient during a spell, especially during electroencephalography, may provide excellent evidence or even lead to a definitive diagnosis. A serum prolactin level is a very good test in differentiating seizure from pseudoseizure; most generalized seizures, and many complex partial seizures, result in elevated levels during this initial 30-minute period following the event (1). It should be done but the results may not be back for several days. In a study by Holtkamp et al., serum creatine kinase (CK) in psychogenic status had a mean of 39 U/L (range 16-90), but in true epileptic status the mean was 699 with a range of 57-2,625 (1). In this study, creatine kinase began to rise after a delay of 3 hours and peaked after more than 36 hours. Therefore, it will not help in an emergency room setting, but it should be done and repeated (e.g., at 12, 24, and 36 hours after the event), as it can provide good evidence later on for true seizures (levels will rise) or against seizures, and for pseudoseizures if levels do not rise or rise only minimally. Through the use of 24-hour electroencephalography and video monitoring, comprehensive epilepsy centers can be extremely valuable and even definitive in differentiating pseudoseizures from true epileptic seizures. However, they are tertiary centers for evaluation of patients who have not received definitive diagnoses or have not responded to treatment for either. Also, these centers are not that common, and referral can be prolonged and complicated. As valuable as psychiatric referral could be if the patient is having pseudoseizures, a consult at this time would not be helpful. The patient's condition would preclude any meaningful interview. Also, pseudoseizure is a diagnosis of exclusion, and a full evaluation for other causes of her spells has not yet been done. Furthermore, premature referral could negatively impact cooperation from the patient and family. Psychiatric referral should be considered only after a firm diagnosis of pseudoseizures has been made, or where it becomes apparent that behavioral input is necessary to help in this differential. Summary of objective diagnostic testing that can be done to differentiate seizures from pseudoseizures: Electroencephalograms, if available, can be very helpful during the acute phase; with tonic and/or clonic movements, one would expect to see epileptiform abnormalities. Psychomotor seizures may show in the electroencephalogram, but not necessarily. A negative electroencephalogram is strong evidence of pseudoseizures, but not proof. An encephalogram done within 90 minutes after an epileptic seizure frequently shows transient postictal changes that will not be seen after a psychogenic seizure (5). Prolactin levels, done within 30 minutes of a seizure, may be helpful since most generalized seizures, and many complex partial seizures, result in elevated levels (1). Serum creatine kinase (CK) in psychologic status had a mean of 38 U/L (range 16-90), but in true epileptic status, the mean was 699 with a range of 57-2,625. In the study cited in (1), with true epileptic seizures, creatine kinase begins to rise after a delay of 3 hours, and it peaks after more than 36 hours. EEGs, if they can be obtained on a stat basis, can be most valuable in picking up the presence or absence of epileptiform abnormalities during the spells or the presence or absence of postictal changes in the 30-90 minute period following true seizures.Video recording of the patient, if a video camera is available, can be most helpful in later efforts at diagnosis. A combination of EEG and video recording could be of major importance. At a later time, continuous video and electroencephalography (vEEG) may be arranged if necessary, and it constitutes the gold standard for diagnosis.

A 5-year-old girl was hit by a slow moving car when she ran into the street while playing tag. She hit her head and had a brief period of loss of consciousness along with a few superficial scrapes and bruises. She was placed in cervical spine precautions and brought to the ED where cervical spine radiographs were read as negative. What transient symptoms immediately after injury suggest the presence of spinal cord injury without radiographic abnormality? Answer Choices 1 Burning hands syndrome 2 A see-saw respiratory pattern 3 Weakness in the arms greater than that in the legs 4 Lightning sensation down the spine with neck movement 5 Complete paralysis with loss of pain sensation ANS:4

4 Lightning sensation down the spine with neck movement Spinal cord injury without radiographic abnormality (SCIWORA) occurs almost exclusively among children younger than 8 years of age. Increased elasticity of the pediatric spine allows injury to the cord without obvious bony disruption. Up to 27% of these children may experience delayed onset of neurological signs. This delayed period may range from 30 minutes to 4 days, with a mean of 1.4 days. Clues to this condition may be the presence of certain transient symptoms immediately after the injury. These include, in addition to a lightning sensation down the spine associated with neck movement, paresthesia in the hands and legs or a subjective feeling of generalized weakness. The chief measures to improve outcome include injury prevention, prompt recognition through examination, search for such transient symptoms, use of MRI with electrophysiological verification, and timely bracing of SCIWORA patients. A burning sensation in the hands, fingertips, and occasionally feet after an injury, also known as burning hands syndrome, is probably due to hyperextension of the cervical cord with contusion of the spinothalamic tract. It is more commonly seen in football players. A see-saw respiratory pattern where the chest retracts and the abdomen distends may indicate paralyzed intercostal muscles due to cervical spine injury. Incomplete injuries include the central cord syndrome in which there is weakness in the arms that is greater than that in the legs, as well as variable bladder involvement. Another incomplete injury is the anterior cord syndrome that leads to complete paralysis below the level of injury, as the voluntary motor impulses carried by the corticospinal tracts are anteriorly located. There is also loss of pain sensation and /or temperature sensation. Preservation of proprioception through light touch remains intact.

You are moonlighting in a clinic that provides family planning services for low-income women. The following choices represent the only pertinent historical findings for individual patients. Who would be an appropriate candidate for oral contraceptives? Answer Choices 1 23-year-old Laotian immigrant taking rifampin for tuberculosis 2 25-year-old African-American woman with familial dyslipidemia (HDL 35, LDL 190) 3 35-year-old Caucasian woman with a history of classic migraines with aura 4 36-year-old Caucasian woman who smokes 1 pack of cigarettes per day 5 43-year-old African-American woman taking valproic acid for seizure control ANS:5

43-year-old African-American woman taking valproic acid for seizure control The correct response is a 43-year-old African-American woman taking valproic acid for seizure control. Some anticonvulsants (phenytoin and carbamazepine for example) decrease steroid levels in women taking birth control pills by inducing hepatic enzymes. It is not clear that this results in ovulation, but caution is advised. This effect is not seen with valproic acid.

A 44-year-old man starts to notice that his eyelids are drooping. Some time afterwards, his jaw becomes weak. He has difficulty swallowing and also experiences weakness in his limbs. He is quite embarrassed when he eats because he must use his hand to help support his jaw. His weakness gets progressively worse. Finally, he seeks medical attention. His physical examination demonstrates the weakness in his limbs; however, no sensory defects are present. A Tensilon test is done and is positive. His doctor is concerned about an associated malignancy. What is the underlying pathology of this disease? Answer Choices 1 Inhibition of acetylcholine release 2 Blockage of the sodium channels 3 Demyelination 4 Subacute combined degeneration of the spinal cord 5 Antibodies to the acetylcholine receptor ANS: 5

5 Antibodies to the acetylcholine receptor Antibodies directed towards the acetylcholine receptor at the neuromuscular junction are seen with myasthenia gravis. This man has myasthenia gravis. Ocular muscle weakness, ptosis, dysphagia, and limb weakness can all be seen with myasthenia gravis. When the initial symptom is ocular weakness, Eaton Lambert Syndrome is extremely unlikely. Eaton Lambert Syndrome tends to not involve the extra-ocular muscles or the muscles involving chewing, swallowing, or speech. The Tensilon test is used in the diagnosis of myasthenia gravis. The Tensilon test consists of the administration of edrophonium. Edrophonium is a quick acting anticholinesterase. Thymic tumors are associated with myasthenia gravis. Thymic tumors are also referred to as thymomas. Approximately 10 - 15% of patients with myasthenia gravis have an associated thymoma. The majority of patients with myasthenia gravis have hyperplasia of their thymus. Botulinum toxin inhibits acetylcholine release. The site of action is at the neuromuscular junction. Botulinum toxin is an enterotoxin produced by Clostridium botulinum. Botulism can result from incorrectly canned foods. Tetrodotoxin is a toxin produced by puffer fish. The sodium channels are blocked by tetrodotoxin. The blockage of the sodium channels interferes with the inflow of sodium. As a result, the propagation of nerve and muscle action potentials is affected. Demyelination refers to the loss of myelin around the axon. Several disorders result in demyelination. An example of a demyelinating disease is multiple sclerosis. Subacute combined degeneration of the spinal cord is also called combined systems disease. Subacute combined degeneration of the spinal cord is a neuropathy secondary to B12 deficiency. It is seen in patients with pernicious anemia, especially pernicious anemia that has been present for quite some time. Symptoms include paresthesias and a loss of proprioception.

A 12-year-old girl presents with a 3-day history of progressive dysarthria, dysphagia, and weakness. The patient was well until 3 days prior to admission to the hospital; at that time, she developed the onset and subsequent gradual worsening of dysarthria. She attributed the dysarthria to a sore throat that she had had about 2 weeks earlier. 3 days prior to admission, she also had the onset of mild dysphagia; it mostly occurred with liquids. 24 hours prior to admission, she developed weakness in both upper extremities, which increased and began to involve the lower extremities. This limb weakness was neither worsened by activity nor improved by rest. She also developed tingling in her toes 24 hours prior to presentation. When she became unable to walk without assistance on the day of admission, she decided to seek medical attention and was admitted to the hospital. Past medical history is significant for measles and mumps. Because of family religious beliefs, she has not had any immunizations. She is very athletic, and frequently plays soccer with friends and siblings in the fields on her grandfather's horse farm. Physical examination reveals a well-developed, well-nourished girl. She is awake, alert, cooperative, and in no acute distress. Temperature is 98.7 F by mouth, blood pressure of 140/80 mm Hg, heart rate is 84/min and regular, and respirations are 22/min and unlabored. There are multiple scratches and abrasions in varying stages of healing over most of her extremities. Her speech is moderately dysarthric. She experiences some mild choking when she tries to drink a glass of water. She can smile weakly, but she cannot raise her eyebrows against resistance. She shows mild bilateral weakness of eye adduction. Pupillary responses are normal. There is mild to moderate upper extremity and mild lower extremity weakness, greater distally than proximally. Her motor strength is sustained over at least 30 seconds without fatigue. Her gait is ataxic, and she cannot walk without assistance. Reflexes are hypoactive to absent, and the response to plantar stimulation is downgoing bilaterally. Sensation is intact, except for mildly impaired position and vibratory sensation in both feet. A complete blood count, chemistry profile, chest X-ray, and EKG are all normal. Computed tomography of the brain, with and without contrast, is negative. A nerve conduction study reveals a moderate degree of mostly motor demyelinating peripheral neuropathy, highly suggestive of Guillain-Barre. Question What statement best describes the patient's prognosis? Answer Choices 1 With a predominantly demyelinating rather than axonal neuropathy, her prognosis is especially bad 2 Mortality is 25-30% 3 Full functional recovery is expected in 45% in several months to a year 4 Whether recovery is full or partial, relapses do not occur during the recovery phase 5 Her rapidly evolving clinical course indicates a poor prognosis ANS:

5 Her rapidly evolving clinical course indicates a poor prognosis This patient must be watched very closely for the very real possibility of respiratory failure and the need for ventilatory support (2). Mortality is expected to be less than 5% with good medical support (1). With a demyelinating pattern on EMG, her prognosis is better. A consistent indicator of residual muscle weakness is an EMG pattern of axonal damage, with the more severe degrees of damage suggesting the worse prognosis (1). About 85% of patients with GBS have a full functional recovery within a year; however, some may be left with minor residuals such as areflexia on exam (2). Between 5-10% of patients with Guillain-Barre have 1 or more relapses; these cases are referred to as chronic inflammatory demyelinating peripheral neuropathy (CIDP) (1).

Ethosuximide is a primary treatment for what type of epilepsy? Answer Choices 1 Febrile seizures 2 Absence seizures 3 Generalized tonic-clonic seizures 4 Status epilepticus 5 Simple partial seizures ANS:2

Absence seizures The term epilepsy refers to a group of central nervous system disorders. These disorders are characterized by repeated occurrence of sudden and transitory seizures, which may be of motor, sensory, autonomic, or psychic origin. Seizures are almost always accompanied by abnormal and excessive discharges in the brain. The many types of seizures are classified into 10 different categories. Partial seizures, also known as focal or local seizures, are broken down into 3 groups: simple partial seizures, complex partial seizures, and partial seizures secondarily generalized. The generalized seizures (convulsive and nonconvulsive) are grouped into absence seizures, atypical absence seizures, myoclonic seizures, clonic seizures, tonic seizures, tonic-clonic seizures, and atonic seizures. Ethosuximide and valproic acid are the primary agents for the treatment of absence seizures. These drugs are generally ineffective against generalized tonic-clonic seizures and simple partial seizures, which are best treated with carbamazepine or phenytoin. Febrile seizures, if treated at all, are best treated with phenobarbital. Status epilepticus is a life-threatening disorder that must be treated promptly. Intravenous diazepam and/or phenytoin is indicated for control of this disorder.

A 68-year-old female presents to the emergency department with signs and symptoms of an acute ischemic stroke. The initial CT scan is normal. Her blood pressure is 164/105. What is the most appropriate treatment for the blood pressure of this patient? A Atenolol PO B Clonidine PO C Close monitoring D Labetolol IV E Nicardipine IV ANS:C

C Close monitoring Aggressively lowering blood pressure may decrease blood flow to the ischemic tissue, thus decreasing the chances of recovery or increasing the risk of further infarction. In the setting of an acute ischemic stroke, blood pressure elevation should be monitored closely, with some elevation expected. This elevation is expected to decline without medication in the first few hours to days, but if elevation continues to a systolic blood pressure greater than 220mmHg, or mean arterial pressure greater than 120mmHg, medication is advised. Medications may include intravenous labetolol or nicardipine, with close monitoring of the patient. After the acute phase following a stroke, appropriate oral medications may be considered for outpatient hypertension management.

A 66-year-old woman is brought to the emergency room by her daughter; the patient is becoming incoherent and confused. She was in good health until early that morning when she reported having a headache. She is not a known hypertensive or diabetic, and she is not on any medications. On examination, she is well-nourished, has a fever, and is disorientated to time, space, and person. Brudzinski's sign is positive. What would you expect to find on examination of her cerebrospinal fluid (CSF)? Answer Choices 1 CSF pressure 250 mm H2O, cells 2000/uL with predominant neutrophils 2 CSF pressure 150 mm H2O, cells 500/uL with predominant lymphocytes 3 CSF pressure 150 mm H2O, cells 250/uL with predominant lymphocytes 4 CSF pressure 150 mm H2O, cells 150/uL with predominant lymphocytes 5 CSF pressure 250 mm H2O, predominantly red blood cells ANS:1

CSF pressure 250 mm H2O, cells 2000/uL with predominant neutrophils The correct response is CSF pressure 250 mm H2O, cells 200/µL with predominant neutrophils. The clinical picture is suggestive of acute bacterial meningitis; this is an inflammatory disease involving the arachnoid layer of the meninges and the cerebrospinal fluid (CSF). It is a medical emergency and antibiotic treatment should be started immediately. Patients with acute bacterial meningitis usually have an increased CSF pressure, cells 500 - 5000/uL, and neutrophils are the predominant cells. The CSF glucose is decreased and the protein level is usually over 100 mg/dL. A Gram stain should be done on the CSF smear. Serologic latex agglutination tests for meningococcus, H. influenzae type B, and pneumococcal antigens in the CSF can provide a diagnosis rapidly. The CSF should be cultured. Other relevant cultures (e.g., blood, nasopharyngeal and respiratory secretions, urine, and skin lesions) should also be done. A lumbar puncture should be done to obtain CSF samples after a CT scan has excluded a mass lesion. Normal values of CSF parameters are as follows: A. CSF pressure 100 - 200 mm H2O B. Cells 0 - 3/uL C. Predominant cell type - lymphocyte D. Glucose 50 - 100 mg/dL E. Protein 20 - 45 mg/dL The following are the common bacterial causes of acute meningitis in the various age groups: Escherichia coli and Streptococci agalactiae are found in neonates and infants. Haemophilus influenzae type B, Streptococcus pneumoniae, and Meisseria meningitidis are found in early childhood. Neisseria meningitidis is found in adolescents and young adults. Meningococci exist in the nasopharynx of about 5% of the population and can spread by respiratory droplets through kissing, sneezing, or coughing. They tend to cause epidemics among closed populations (e.g., military barracks and boarding schools). Streptococcus pneumoniae is the most common cause of acute meningitis in people over 25 years of age. Especially at risk are alcoholics and persons with chronic otitis, sinusitis, pneumococcal pneumonia, sickle cell disease, and asplenia. Staphylococcus aureus is a common cause of acute meningitis in immunocompromised patients after penetrating head wounds or neurosurgical procedures. Gram-negative bacteria, such as Escherichia coli, Klebsiella, and Enterobacter, are common causes of nosocomial meningitis and meningitis after neurosurgical procedures, trauma, and genitourinary manipulation in the elderly. Listeria monocytogenes meningitis is common in patients with immunosuppression, carcinomas, and those receiving corticosteroids or cytotoxics. Patients usually present with headache, fever, chills, photophobia, vomiting, seizures, confusion, drowsiness, stupor, and coma (especially in the elderly). There may be a history of a prodromal upper respiratory tract infection, especially in patients with pneumococcal meningitis. Dehydration and vascular collapse due to adrenal insufficiency may lead to shock in Waterhouse-Friderichsen syndrome in meningococcal septicemia. Complications include cranial nerve palsies, cerebral infarction, convulsions, and mental retardation. On examination, patients may be febrile and have a stiff neck. A petechial or purpuric rash suggests meningococcal meningitis. Patients with increased intracranial pressure may have papilledema. The following signs may be positive: Brudzinski's sign ( flexion of the neck in a supine patient results in involuntary flexion of the hips and knees), and Kernig's sign (attempts to extend the knee from the flexed thigh position are met with strong passive resistance). There also may be evidence of extracranial infections (e.g., sinusitis, otitis, and mastoiditis). The differential diagnoses include viral meningitis, Lyme disease, tuberculous meningitis, and cerebral hemorrhages; all have different CSF findings. In tuberculous meningitis, the presentation is subacute or chronic and the CSF protein and pressure are increased. Lymphocytes are the predominant cell types with cell counts of 100 - 700/uL. In Lyme disease, the CSF protein is normal or increased; lymphocytes are the predominant cell types with cell counts of 0 - 500/uL. In viral meningitis, CSF protein and pressure may be normal or increased; lymphocytes are the predominant cell types with cell counts of 100 - 200/uL. In cerebral hemorrhages, the CSF is bloody, the pressure is increased, and red blood cells are the predominant cell types. The CSF glucose is within the normal range. Penicillin G is used for Streptococcus pneumoniae, and Neisseria meningitidis is sensitive to the drug. Ceftriaxone and chloramphenicol are used for Haemophilus influenzae and Gram-negative bacilli. Oxacillin is used for Staphylococcus aureus. Ampicillin is used for Listeria monocytogenes. Early use of corticosteroids, such as dexamethasone, may avert the development of major neurologic sequelae; they do so by inhibiting the release of pro-inflammatory cytokines triggered by bacterial fragments (which result from antibiotic activity).

A 15-month-old toddler presents with sudden onset of generalized tonic and clonic convulsions for the last 30 minutes. There is no history of trauma, fever, vomiting, or irritability prior to the onset of convulsions. This is the first episode of seizure, and there is no history of convulsions in the family. Birth history, neonatal period, and developmental milestones are normal. Question After initial stabilization by securing the airway, oxygenation, and controlling the acute convulsion, what is the investigation of choice for the above child? Answer Choices 1 Cerebrospinal fluid examination 2 CT scan of the head 3 Electroencephalogram 4 Cranial ultrasonography 5 MRI scan of the head ANS:2

CT scan of head CT scan of the head is the best diagnostic imaging study in a previously asymptomatic child who presents with status epilepticus, particularly if there is suspicion of intracranial hemorrhage, a space-occupying lesion, or midline shift. It is a non-invasive and rapid procedure and is widely available. It has a short imaging time and requires less sedation in young children. Though CT imaging is largely replaced by MRI, it still remains the most sensitive modality to detect intracranial hemorrhage and calcifications during the first 24 hours. Other indications are head injury, hypoxic ischemic encephalopathy, inflammatory disorders like neurocysticercosis, CNS tuberculosis, basal exudates, pyogenic abscess, and malformations like hydrocephalus and porencephaly. Myelination, posterior fossa, and brain stem structures are not well visualized by CT scan. As the child presented with his 1st episode of convulsion with status epilepticus without any prior symptoms, a CT scan of the head would be the first choice of investigation. Cranial ultrasonography (USG) is mainly used in the diagnosis and follow-up of newborns with intraventricular hemorrhage and for assessment of ventricular size. It can be performed through the open anterior fontanel, and it can also be performed at the bedside of the patient. It can visualize the ventricles, periventricular tissue, and a part of cerebral cortex. As the above child is 15 months old and the anterior fontanel is either closed or is very small at this age, it cannot be performed in this patient. Electroencephalogram (EEG) is commonly used to confirm a clinically doubtful convulsion, to characterize the type of convulsion, to locate an epileptic focus, or to distinguish a simple febrile convulsion from epilepsy. It can help to distinguish between a seizure and a non-seizure state, such as a fainting spell, hypoxic episode, or a breath-holding spell. It is especially helpful in the diagnosis of absence attacks, herpes encephalitis, and myoclonic seizures. It is not the investigation of first choice in the above patient. Lumbar puncture (LP) and examination of cerebrospinal fluid (CSF) is usually indicated for the diagnosis of conditions like meningitis, encephalitis, subarachnoid hemorrhage, demyelinating illnesses, and slow virus infections of the CNS. As the above child was asymptomatic before the episode of convulsion, lumbar puncture is not the investigation of first choice in this child. MRI scan of the head can demonstrate myelination abnormalities, edema, and infarcts more clearly as compared to CT. MRI is the study of choice for evaluating the spine, spinal cord, and spinal canal in children. It is more sensitive than CT scan for most brain tumors involving the brainstem, posterior fossa, and spinal column. It is not the investigation of first choice for acute intracranial hemorrhage during the first 24 hours. Also, it is not widely available for use during emergency.

A patient presents to the office. His gait is wide-based, staggering, and unsteady. He appears to have difficulty with turns and hangs on to his wife as he crosses your waiting room. What type of gait does he have? Answer Choices 1 Scissors gait 2 Steppage gait 3 Sensory ataxia 4 Cerebellar ataxia 5 Myopathic gait

Cerebellar Ataxia: Cerebellar Ataxia= The gait characterized by cerebellar ataxia is associated with a disease of the cerebellum or its affiliated tracts. The gait is staggering, unsteady, and wide-based. The patient has difficulty with turns, and cannot stand steady when feet are together and eyes closed (Romberg's sign). Scissors gait is apparent when each leg is advanced slowly and the thighs tend to cross each other. The gait is stiff and the steps are short. This is associated with bilateral spastic paresis of the legs. Steppage gait is associated with a foot drop, often secondary to lower neuron disease. The patient drags his foot, or lifts the knee high, and slaps the foot against the ground. This gait may be unilateral or bilateral. Sensory ataxia gait is unsteady and wide-based. The feet are thrown forward, out, then down. There is a distinct 2-slap sound, secondary to the heel striking first then the forefoot. This is associated with polyneuropathy or posterior column damage. A myopathic gait is where there is weakness on one side. This displays a a drop in the pelvic area on the contralateral side of the pelvis while walking (Trendelenburg sign). When there is a bilateral weakness, it will display as a dropping of the pelvic girdle on both sides of the body while walking that results in a waddle. This is seen in patients whom are afflicted with myopathies, such as muscular dystrophy.

A 53-year-old man presents with a 2-week history of severe headaches. The headaches occur primarily at night. The patient is pacing while he is talking. The pain surrounds 1 eye and lasts for 30 to 90 minutes. He also states that there is ipsilateral lacrimation, conjunctival injection, and nasal congestion during the attacks. The patient states that he's had these headaches 2 or more times in a day over a period of several weeks. He cannot point to any one thing that causes them. On examination, the patient has features of partial Horner's syndrome. His vital signs are temperature: 97.0 °F, heart rate: 80/min, respiration: 16/min, and blood pressure: 126/80mmHg. Question What type of headache is the patient experiencing? Answer Choices 1 Common migraine 2 Classic migraine 3 Cluster headache 4 Stress headache 5 Chronic daily headache

Cluster HA Cluster headaches come in groups and attacks may be separated by months or years. There are 2 types: episodic and chronic. The patient is often a middle-aged man. Cluster headaches are characterized by excruciating, unilateral pain (stabbing sensation) often occurring in the ocular, frontal, or temporal areas. These headaches generally last for a period of 15 minutes to 3 hours, recurring at the same time of day. They typically occur during the night. Pain often radiates to the upper teeth, jaw, and neck. The pain is normally accompanied by one or more of the following: ipsilateral conjunctival injection or lacrimation, ipsilateral nasal congestion or rhinorrhea, ipsilateral eyelid edema, ipsilateral forehead and facial sweating, ipsilateral miosis or ptosis, and a sense of restlessness or agitation. Restlessness is one of the characteristic features of cluster headaches noted in patients; behaviors such as pacing or rocking the head and trunk with head in hands are common. Other symptoms of cluster headache include facial flushing or pallor, tenderness on palpation of the ipsilateral carotid artery, bradycardia, and tenderness of the scalp and facial areas. The absence of aura, nausea, or vomiting helps in distinguishing cluster headaches from migraines. The treatment of choice for acute cluster headaches includes the administration of oxygen (7 L per minute for 15 minutes), sumatriptan, or a combination of both. Other therapeutic measures include intranasal dihydroergotamine, intranasal capsaicin, or intranasal lidocaine. Prednisone and verapamil are often used in prophylaxis. Common migraine is an intermittent syndrome characterized by hemi-cranial pulsatile cephalic discomfort, including nausea, vomiting, photophobia, anorexia, and phonophobia. Classic migraine typically presents with an aura. It also includes features of nausea, vomiting, photophobia, anorexia, phonophobia, and a sense of just wanting to hide in a quiet dark room. Stress/tension headaches are a generalized type of headache precipitated by stress. They are not one-sided. They do not involve photophobia, nausea, vomiting, aura, or phonophobia. Chronic daily headaches occur on a daily basis and are often disabling. This type of headache may be challenging to diagnose and treat. Headaches caused by medication overuse or drug rebound are the most treatable forms of refractory daily headache.

A 32-year-old woman presents several hours after a grand mal seizure. Her husband states that a few days before the seizure she experiences headache, nausea, vomiting, fever, and "was not herself". She is on glucocorticoid therapy due to a kidney transplant 1 year ago. On examination, you find a lethargic, febrile (39 C), dysphasic patient; there is right-sided hemiparesis, and meningeal signs are present, but detailed examination cannot be performed because of lack of cooperation. Cerebrospinal analysis (CSF) shows normal opening pressure, xanthochromia, mononuclear pleocytosis (100 white blood cells), elevated protein (100 mg/d), and a normal serum to glucose ratio; EEG shows focal abnormalities over temporal lobes, and her CT is normal. While waiting for the results of polymerase chain reaction, you start supportive care, anticonvulsants, diuretics, and acyclovir. Question What factor guides you to the choice of acyclovir? Answer Choices 1 EEG changes 2 CSF findings 3 Glucocorticoid therapy 4 Focal neurological deficit 5 Psychiatric symptoms

EEG In an immunocompromised patient with this clinical picture, you should think about herpes encephalitis. Its presentation is often atypical: fever, headache, and alteration of consciousness, focal neurological deficits, seizures, and other signs of acute encephalitis. When you suspect encephalitis, the workup must be initiated rapidly, but the workup should not delay treatment. There are no pathognomonic clinical findings in herpes simplex encephalitis, but electroencephalography (EEG), with a specificity of 32% and sensitivity of 84%, should guide your decision about the therapy. Usual abnormal EEG patterns are focal abnormalities (periodic complexes and periodic lateralizing epileptiform discharges, spike and slow- or periodic sharp-wave patterns) over temporal lobes or diffuse slowing. The diagnosis will be confirmed only by polymerase chain reaction or brain biopsy, but those results will not be available immediately, and they should not delay therapy. In this case, the CT excluded a space-occupying lesion so that a lumbar puncture could be performed. CSF findings are not specific; they will only raise the suspicion of viral cause of the disease. Patients on glucocorticoid therapy and consequent immune suppression are prone to different infections, not only herpes simplex encephalitis. A focal neurological deficit is a non-specific finding. Focal neurologic deficits, cerebrospinal fluid pleocytosis, and abnormalities on computed tomography may initially be absent in herpes simplex encephalitis. Psychiatric symptoms are not specific for herpes simplex encephalitis; they are part of clinical picture of a variety of CNS disorders.

A 42-year-old woman presents with a tremor that has become more bothersome lately. She thinks it originally began as a mild tremor twelve years ago. It affects her hands more than her head. It usually occurs when she moves her extremities. She drinks 1 or 2 alcoholic beverages every day because alcohol seems to lessen the tremor. Her mother suffered from a similar disorder. Her neurological exam is essentially normal other than a tremor of the hands and head. What movement disorder does this patient have? Answer Choices 1 Alcoholic withdrawal tremor 2 Cerebellar tremor 3 Essential tremor 4 Parkinsonian tremor 5 Psychogenic tremor

Essential tremor The patient has essential tremor, which is the most common movement disorder. This postural tremor may have its onset anywhere between the second and the sixth decades of life. Its prevalence increases with age. It is slowly progressive over a period of years. It may be familial with autosomal dominant penetrance. The frequency of the tremor is typically 5 Hertz. In addition to the hands, it also affects the head, voice, tongue, and legs. It may be alleviated by alcohol. Alcoholic withdrawal tremor affects only the hands, unlike the multiple sites of involvement of essential tremor. Cerebellar tremor is an intention tremor or goal-directed. It increases in severity as the extremity approaches a target. Parkinsonian tremor occurs at rest. Psychogenic tremor includes an abrupt onset, a static course, spontaneous remission, and unclassified tremors. They often increase in frequency and amplitude with attention, and they decrease with distraction.

A 32-year-old woman presents due to "I'm feeling odd." Further questioning reveals the patient experiencing a significant amount of involuntary movements and a severe lack of coordination; she recently had a significant decrease in cognitive abilities. Because she is an established patient, you recall that the patient got married about a year ago; past family history is unknown due to the fact that the patient was adopted as a child. Physical examination reveals rapid, involuntary movements of fingers bilaterally, impaired ability to concentrate, minor slurred and disorganized speech, as well as difficulty responding with the appropriate words or phrases when prompted. Question What would be the recommended testing at this time to confirm the patient's diagnosis? Answer Choices 1 Magnetic resonance imaging (MRI) of the head 2 Computed tomography (CT) of the head 3 Acid fast bacilli (AFB) smear 4 Genetic testing 5 Rapid plasma reagin (RPR) testing ANS:4

Genetic testing The correct response is genetic testing. The patient in the above scenario is displaying signs of Huntington's disease. This fatal progression of chorea, dementia, and behavioral changes is an autosomal dominant disorder that occurs in all ethnic groups with a prevalence rate of 5 per 100,000. More commonly than not patients will have a strong family history of this disease state; the unknown family history in our patient makes this information, although missing, extremely valuable. Onset of symptoms typically occurs between the ages of 30 - 50 years old and usually leads to death within 15 - 20 years after symptoms begin occurring. Early symptoms and signs include abnormal movements or intellectual changes, both of which are evident in the patient above. Severe progression of rigidity, chorea, akinesia, and severe psychiatric changes will accompany the progression of the disease. Diagnostic testing at this time to confirm diagnosis of Huntington's disease should be via genetic testing with ultimate guidance from a genetic counselor. The gene responsible for causing Huntington's disease has been identified exclusively on chromosome 4. CT scanning or MRI imaging will not confirm the diagnosis, but rather support it; cerebral atrophy as well as caudate nucleus atrophy will be found as the disease progresses. Acid fast bacilli smear is the inappropriate choice for this scenario; this diagnostic testing may be utilized in a patient to support the diagnosis of tuberculosis. Rapid plasma reagin testing is also the incorrect choice; this diagnostic study along with the serologic test for syphilis (STS), venereal disease research laboratory (VDRL), and the fluorescent treponemal antibody (FTA) assist health care providers in diagnosis as well as successful treatment of syphilis.

A 32-year-old man presents with a 2-day history of high fever and progressive, severe, headaches. It is associated with blurred vision and increasing confusion. The patient is normally healthy, and he does not have a remarkable past medical history. He is married. He does not drink alcohol, smoke, or use illicit drugs. He has not had any blood transfusions, and he takes no medications. On physical examination, he appears ill and disorientated to time, place, and person. His pulse is 110/minute, temperature 39 degrees Celsius (102 degrees Fahrenheit), respirations 22/minute and blood pressure is 115/70 mmHg. He is well-hydrated. He has no scleral icterus or oral candidal infection. Pupils are equal and reactive. There are no palpable adenopathy or rashes. His cranial nerve examination is intact. His fundi are normal. He can move all of his limbs. Deep tendon reflexes are normal; plantar reflexes are equivocal. His neck is supple. His lab work includes Hemoglobin = 13. 1 g/dL; hematocrit = 39%; white blood cell count (WBC) = 18x109/L with 70% polymorphonuclear neutrophils and 30% lymphocytes; platelets = 200x109/L; erythrocyte sedimentation rate = 90 mm/hour; sodium = 138 mEq/L; potassium = 4 mEq/L; chloride = 102 mEq/L; blood urea nitrogen (BUN) = 29 mg/dL; creatinine = 1mg/dL; glucose = 101mg/dL; calcium = 9. 2 mg/dL; total bilirubin = 1mg/dL; alanine transaminase (ALT) = 26 units/L; aspartate transaminase (AST) = 22 units/L; alkaline phosphatase = 112 units/L. Results of drug screening are negative and his urinalysis is normal. The lumbar puncture results are as follows: Cerebrospinal pressure Increased Protein level 180 mg/dL Glucose 92 mg/dL White blood cell count 116 per microliter (all lymphocytes) Red blood cell count 80 per microliter Gram-staining results White blood cells with no organisms Oligoclonal bands Absent IgG index Normal Question An MRI scan of the brain demonstrates low-density lesions in the brain; they represent areas of gross demyelination. What is the most likely diagnosis? Answer Choices 1 Acute multiple sclerosis 2 Central nervous system lymphoma 3 Herpes simplex virus (HSV) encephalitis 4 Multifocal leukoencephalopathy 5 Viral meningitis ANS:3

HSV encephalitis The patient has a fever, severe headache, altered mental status and has abnormal levels of lymphocytes in the CSF. These findings, along with the results of the MRI scan, suggest herpes simplex virus, encephalitis. HSV encephalitis occurs in 10% of all cases of encephalitis in the United States. There is a high mortality rate and morbidity rate. The latter is normal in the form of neurological damage. Herpes simplex type 1 is the most common cause and type 2 causes neonatal encephalitis (acquired by mother). The illness typically occurs over several days. The CSF findings in this patient typify the condition. Specifically, the CSF pressure is increased with pleocytosis. The red blood cells reflect the hemorrhagic nature of the CNS lesions. The protein level is increased; the glucose level may be normal or decreased. Tuberculosis or fungal meningitis is associated with decreased glucose levels, which can create confusion. Multiple sclerosis (MS) is a chronic CNS disease affecting young adults. There is the presence of demyelination and T-cell predominant perivascular inflammation in the brain white matter. Some axons may be spared from these pathological processes. Early symptoms may include numbness and/or paresthesia, mono- or paraparesis, double vision, optic neuritis, ataxia, and bladder control problems. Subsequent symptoms include upper motor neuron signs. Vertigo, incoordination and other cerebellar problems, depression, emotional lability, abnormalities in gait, dysarthria, fatigue and pain are also commonly seen. The presence of oligoclonal bands and/or elevated IgG index constitutes the basis for diagnosis. The most typical presentation of a primary CNS lymphoma is in an immunocompetent patient with progressive symptoms indicative of a mass lesion. Seizures may occur. Patients with AIDS are more likely to present with an encephalopathy than other patients with a primary CNS lymphoma. A careful sexual and drug abuse history is necessary. Isolated, ocular, or meningeal tumor may occur in the absence of any focal abnormalities on MRI. Multifocal leukoencephalopathy is a lethal secondary viral infection; it primarily occurs in AIDS patients with advanced immunodeficiency. The JC virus is a ubiquitous virus, able to enter the brain and lytically infect oligodendrocytes (the cells making myelin in the brain). This causes demyelination of the brain, which results in a wide variety of focal neurologic symptoms, including weakness, loss of sensation, visual loss, and changes in balance and coordination. Because it is usually relentlessly progressive, severe neurologic disability develops over a period of 2 to 6 months; death follows rapidly from general disability. A small percentage of persons stabilize or spontaneously improve. The lack of meningeal signs and the presence of demyelination on the MRI scan would not typically back up the diagnosis of viral meningitis.

A 23-year-old woman presents with increasing fatigability and weakness. On further questioning, she reveals that, in the past month, she has experienced difficulty swallowing; there have also been episodes of double vision as the day progresses. Several of her professors have expressed concern because she looks sleepy during the day. A CT of the chest is advised. It reveals a mass that, once removed surgically, significantly lessens symptoms. The pathology slide made from the removed tissue can be found below. Based on the combined aspects of her history, findings upon investigation, and her recovery, what did the patient likely have? Answer Choices 1 Myasthenic syndrome 2 Myasthenia gravis 3 Amyotrophic lateral sclerosis 4 Botulism 5 Hyperthyroidism 6 Inclusion body myositis

Myasthenia Gravis Explanation Myasthenia gravis is an autoimmune disorder in which patients present with the cardinal symptoms of weakness and fatigue.This condition is more common among young women. Clinical features of this condition include: Ptosis and diplopia Chewing and swallowing difficulties Respiratory difficulties Asymmetrical proximal limb weakness with normal deep tendon reflexes The initial symptoms are often related to muscles that are controlled by cranial nerves, as these are affected first. Confirmation of this condition and the elimination of differential diagnosis can be done using the edrophonium test. The administration of edrophonium, a short-acting anticholinesterase, results in a marked increase in the muscle strength for a short duration in patients with myasthenia gravis. Investigations done in patients with myasthenia gravis are: The acetylcholine receptor antibody test CT scan of the thorax to detect a thymoma if present Electromyogram (EMG) The patient in this question was found to have a mediastinal mass, which was found to be a mixed thymoma. (See image) Treatment of myasthenia gravis involves: Neostigmine and other anticholinesterase drugs, improving symptoms. Avoidance of drugs such aminoglycosides. Thymectomy(removal of the thymus), which produces an improvement in over 85% of the patients between puberty and 55 years of age. It is usually performed in cases of moderate-to-marked generalized myasthenia gravis, and in some, it can produce complete remission of the disease. A thymoma is an indication for surgery. Steroids provide for symptom management as well; improvement is the most dramatic with high-dose corticosteroids. Plasmapheresis and IV immunoglobulins. Amyotrophic lateral sclerosis is a type of motor neuron disease. This is a mixed upper and lower motor neuron deficit, which is found in the limbs and is often associated with Parkinsonism and dementia. This condition does not present with ptosis, and it does not have the characteristic variation of muscle power seen in myasthenia gravis. Myasthenic syndrome, otherwise referred to as the Lambert-Eaton Syndrome, is the result of the defective release of acetylcholine in response to a nerve impulse. The resulting weakness is more prominent in the proximal muscles of the limbs. Muscle contraction strength increases with sustained contraction. Diagnosis of the disease is by electrophysiological studies. It is usually associated with small cell carcinoma of the lung. Treatment is aimed primarily at the tumor, but plasmapheresis and immunosuppression have been successful. Botulism is due to consumption of the toxin of Clostridium botulinum, which prevents the release of acetylcholine at the neuromuscular junctions and the autonomic synapses. This is an acute condition. Symptoms appear within 72 hours of ingestion, and they require immediate medical attention. Inclusion body myositis is a disorder of middle age. It results in weakness of the proximal muscles of the lower and then upper limbs. The diagnosis of this condition is also by means of a muscle biopsy, and it does not respond to corticosteroids. This patient does not show any of the other clinical signs of hyperthyroidism, such as weight loss, increased appetite, preference for cold weather, nervousness, increased bowel movements, an enlarged thyroid, etc.

GCS values

The GCS is correlated with degree of brain injury: GCS ≥13, minor injury; GCS 9 -12, moderate injury; and GCS 3 - 8, severe injury (coma). A GCS ≤ 8 generally indicates the need for airway protection via endotracheal intubation.

A 76-year-old woman presents because her children are concerned that that she might have dementia. She states that she is doing reasonably well, except that she sometimes sleeps less deeply and wakes up more often than she did several years ago. According to her children, she is slower than before, and her memory has been getting worse over the last 3 years; she has difficulties recalling the specific date of an event (although she can describe the event itself). She also has a great deal of trouble with names, but she can easily recognize people. She always says: "It is on the tip of my tongue, but..." Aside from hypertension that is under control, she does not have any other health problems. She has been a widow for about 10 years. Her older brother was diagnosed with dementia. Physical examination today is within normal limits for the age, and neurological examination is nonfocal. Question What should be your next diagnostic step? Answer Choices 1 EEG 2 Stanford-Binet Intelligence Test 3 Mini-mental status examination 4 MRI of the brain 5 TSH and T4

The mini-mental state examination (MMSE) is a brief 30-point screening test for cognitive impairment. In persons older than 70, the ability to memorize, acquire, and retain new information and recall names is diminished. Memory function may be disturbed in this way despite the relative retention of other intellectual abilities. It particularly applies to episodic memory, and it is manifested by difficulties with recalling names and/or specific dates. This disturbance is known as benign senescent forgetfulness or age-associated memory impairment. It progress very little or not at all over a period of many years, and it does not interfere significantly with the person's work performance or activities of daily living. Your patient most probably has this sort of age related cognitive decline. MMSE can be used also as follow-up of her condition. Electroencephalography (EEG) is recording of spontaneous electrical activity of the brain; it is primarily used in the diagnosis of epilepsy, and it is sometimes used in the diagnosis of coma, encephalopathies, sleep disorders, and brain death. Your patient's sleep complaints are normal in advanced age. Stanford-Binet Intelligence Scales cannot be used to score adults. MRI of the brain is indicated when structural changes are suspected; however, they are not likely in this patient due to her normal neurological examination. You should perform a thyroid function test if you find out that a patient has dementia - slowness is a part of normal aging.

A 55-year-old man with a history of heavy alcohol use presents for treatment of an infected gash in his hand. He is not currently intoxicated. His clothes fit loosely, and he mentions that he thinks he has lost weight since becoming homeless earlier in the year. In preparation for giving him antibiotics, he receives an intravenous infusion of 5% dextrose solution. 2 hours later, he is confused and ataxic, with bilateral 6th nerve palsies (intranuclear ophthalmoplegia) and nystagmus. What is the patient most likely exhibiting? Answer Choices 1 Alcoholic ketoacidosis 2 Wernicke encephalopathy 3 Friedreich ataxia 4 Sepsis 5 Malnutrition ANS:2

Wernicke encepahlopathy Ablation or destruction of the limbic areas highlights the role of limbic areas in the processing and storage of memory. Because of the protection afforded by the redundancy of having 2 hemispheres with overlapping functions, tumors, injuries or infarctions affecting limbic areas are often silent unless they cause pressure effects or electrical excitation. Toxic or metabolic damage, which typically affects symmetrical areas, can have a dramatic impact by contrast. The most common, clinically significant example is that of thiamine deficiency, which occurs in malnutrition, especially among alcoholics. Acute thiamine deficiency produces Wernicke encephalopathy. The cardinal features of this condition are delirium with ataxia, oculomotor dysfunction (especially 6th nerve palsy- intranuclear ophthalmoplegia), and sometimes anisocoria. Because thiamine is a cofactor for the cerebral metabolism of glucose, administering glucose to someone on the verge of thiamine deficiency can precipitate acute encephalopathy. Wernicke encephalopathy is prevented or rapidly reversed by administration of thiamine. Chronic thiamine deficiency causes alcohol-induced persisting amnestic disorder (Korsakoff's syndrome). The main sites of damage are paraventricular, symmetrical lesions in the mammillary bodies, the thalamus, and the hypothalamus. Lesions may also occur in the midbrain, pons, medulla, fornix, and cerebellum. Cerebellar and ocular signs may not be present. Diagnosis depends on a careful mental status examination. Patients with Korsakoff's syndrome have relatively preserved long-term memory, are socially appropriate, use language normally, and can do simple calculations. Their profound recent memory loss seems so disproportionate to the rest of their mental status examination that observers may misdiagnosis malingering, i.e., willfully faking abnormality for personal gain. Patients with Korsakoff's syndrome will occasionally try to conceal their deficits by confabulation, making up or agreeing to untrue scenarios given by the examiner. This is not the same thing as lying, but it is often mistaken for it. The mild macrocytosis is a common incidental finding in chronic alcoholics, but it does not account for the observed behavioral changes.

A 5-month-old male infant presents after a seizure involving all 4 limbs. His mother tells you that he was born full term without any complications, and he was well until 2 days ago when he developed a fever. He vomited multiple times yesterday and was irritable. He has not had diarrhea or a cough. He was given antipyretic medication for his fever. He has no known allergies. His immunizations are up-to-date. His developmental milestones have been in accordance with his age. On physical exam, his temperature is 102.7 F, and his pulse is 154/min; BP is 90/50 mmHg, and RR is 20/min. He is lethargic, pale, and focal neurological deficits are present. His anterior fontanel is bulging. You suspect that he has bacterial meningitis. Question After drawing blood samples for investigations, what is the most appropriate next step? Answer Choices 1 Intravenous phenytoin 2 Intravenous empirical antibiotics 3 MRI of the head 4 Lumbar puncture 5 Intravenous glucose ANS:2

intravenous emiprical antibiotics The infant in the vignette appears to have bacterial meningitis. The initial approach to the patient should be the "ABCs." After assessing and stabilizing the patient's airway and obtaining IV access, intravenous antibiotics should be given immediately. As bacterial meningitis is associated with high morbidity and mortality, prompt initiation of empirical antibiotics is crucial for better prognosis. The choice of antibiotics is dependent on the patient's age and specific predisposing conditions. Use of broad-spectrum cephalosporins, such as ceftriaxone or cefotaxime with vancomycin, may be used in infants more than 1 month old. Ideally, serum glucose, blood culture, complete blood count, and serum chemistries should be drawn when IV access is obtained; however, drawing labs should not delay beginning antibiotics. Intravenous glucose is necessary if the patient is found to be hypoglycemic; bedside serum glucose is mandatory in any patient that presents with a seizure. Intravenous phenytoin and an MRI of the head might also be necessary for a patient such as the one in the vignette, but would not emergently precede antibiotics. The diagnosis of bacterial meningitis rests on CSF examination performed after lumbar puncture. However, LP is deferred in patients with evidence of increased intracranial pressure, new onset seizure, cardiorespiratory compromise, or focal neurological deficits. Antibiotics should be given, and CT scan of the head should be performed. If CT scan is negative, LP can be performed.

A 23-year-old woman was diagnosed with myasthenia gravis a few years ago, and she now presents for a routine check-up. She is currently being treated with 15 mg of neostigmine taken orally every 4 hours. In myasthenia, there is autoantibody production against the acetylcholine nicotinic post-synaptic receptors. This results in the reduction of the number of acetylcholine receptors. The nicotinic acetylcholine receptors are composed of a, b, g, and d subunits in the molar ratio of what sequence? Answer Choices 2 : 1 : 1 : 1 3 : 1 : 1 : 1 4 : 1 : 1 : 1 5 : 1 : 1 : 1 6 : 1 : 1 : 1

2:1:1:1 Nerve cells and their target cells abut but do not touch each other. An electrical signal traveling along the nerve cell is transmitted across the gap that is called the synapse. The cell that sends the signal across the synapse is called the presynaptic cell; the cell receiving the signal that traveled across the synapse is called the postsynaptic cell. Receptors on the membrane of the presynaptic cell are called presynaptic receptors; receptors on the postsynaptic cell are called postsynaptic receptors. The electrical signal that traveled along the presynaptic cell is passed across the synapse to the postsynaptic cell as a chemical signal. The chemicals that travel across the synaptic gap are called neurotransmitters. Typically, neurotransmitters are released from vesicles in the presynaptic cells. The vesicles fuse with the presynaptic membrane and release their contents into the cleft. The vesicles are called presynaptic vesicles. In vertebrates, acetylcholine is an excitatory neurotransmitter. The neurotransmitter has a postsynaptic receptor that has been visualized by electron microscopy. This shows that the receptor is composed of 4 different polypeptides subunits labeled a, b, g, and d. These subunits occur in a molar ratio of 2 : 1 : 1 : 1, and they have a total mass of 255,000 daltons. Each of the subunits spans the membrane forming a cylinder with a water-filled channel. In addition, each of the subunits exhibits sequence similarity implying that each arose as a gene duplication and diverged from a single gene.

A 46-year-old man recently recovered from a bout of influenza, and he now presents due to an 8-hour history of right-sided facial paralysis. He is having trouble closing his right eye, cannot raise his right eyebrow, and cannot smile with the right side of his mouth. Question What medication(s) should this patient be started on? Answer Choices 1 Ibuprofen 800mg PO q 6 hours plus acyclovir 400mg PO 5 times daily 2 Prednisone 60mg PO daily alone 3 Prednisone 60 mg PO daily plus acyclovir 400mg PO 5 times daily 4 Acyclovir 400mg PO 5 times daily alone 5 Prednisone 60mg PO daily plus augmentin 500mg PO 2 times daily ANS: 3

3 Prednisone 60 mg PO daily plus acyclovir 400mg PO 5 times daily The correct answer is prednisone 60mg PO daily plus acyclovir 400mg PO 5 times daily. Either medication alone can be used with fairly good results, but the most recent research has shown that patients treated within 3 days of onset with both a glucocorticoid and an antiviral have a better outcome than patients treated with either prednisone or acyclovir alone. Ibuprofen 800mg PO q 6 hours plus acyclovir 400mg PO 5 times daily is not the correct answer. Acyclovir is an appropriate choice for treatment, as it is an antiviral, and herpes simplex virus is the likely etiologic agent in the pathogenesis of Bell's palsy. However, a nonsteroidal anti-inflammatory is not the correct type of anti-inflammatory to treat Bell's palsy, so ibuprofen is an incorrect choice. Prednisone 60mg PO daily alone is not the correct answer. Prednisone is an appropriate treatment choice for Bell's palsy. It is typically given 1mg/kg for 5-7 days and then tapered over the next 5-7 days. This appears to shorten the duration of symptoms and improve the functional outcome. It should be started as early as possible, but not at all if it has been 7 days or more since the onset of symptoms. However, outcomes are even better if this treatment is used in conjunction with an antiviral, such as acyclovir. Acyclovir 400mg PO 5 times daily alone is not the correct answer. Acyclovir 400mg PO 5 times per day for 10 days is a good treatment option for Bell's palsy. It is best if started within 72 hours of symptoms, but can be started up to 7 days after the onset. However, outcomes are even better if this treatment is used in conjunction with a glucocorticoid, such as prednisone. Prednisone 60mg PO daily plus augmentin 500mg PO 2 times daily is not the correct answer. Prednisone is an appropriate treatment choice for Bell's palsy. It is typically given 1mg/kg for 5-7 days and then tapered over the next 5-7 days. This appears to shorten the duration of symptoms and improve the functional outcome. However, because it is an antibiotic used to treat bacterial infections, augmentin would not be used to treat Bell's palsy. Bell's palsy is caused by a virus, not a bacterium; therefore, an antibiotic would not help the outcome.

What is the therapeutic window for using recombinant tissue plasminogen activator in acute ischemic stroke? Answer Choices 1 1 hour 2 2 hours 3 3 hours 4 4 hours 5 6 hours

3 hours Tissue plasminogen activator (TPA) catalyzes the conversion of plasminogen to plasmin, which promotes fibrinolysis. Ischemic stroke is caused by sudden occlusion of a cerebral artery by thrombus or embolus, producing a focal neurologic deficit. Timely reperfusion of ischemic brain tissue has been shown to limit irreversible neuronal injury in both humans and animals. The benefit of TPA in acute ischemic stroke was demonstrated in a large, multicentered national study. Patients whose symptoms began within 3 hours of treatment with TPA had significantly less disability at the end of 3 months than those given placebo. Although treated patients had higher incidence of secondary brain hemorrhage (6.4% vs. 0.8 %), there was no difference in mortality between the 2 groups. TPA must be given no later than 3 hours of symptom onset in patients with acute ischemic stroke. Prior to treatment, it is essential to exclude cerebral hemorrhage on a head CT scan. The presence of a bleeding diathesis, uncontrolled hypertension, recent prior stroke, and past history of cerebral hemorrhage are important contraindications for using TPA in acute ischemic stroke. TPA given later than 6 hours of stroke onset is associated with unacceptable risk of cerebral hemorrhage; it is not known whether fibrinolytic therapy can be given between 3 and 6 hours of stroke onset.

A 32-year-old man presents with a severe headache; he has had 2 similar headaches within the past week. He describes a burning, 'hot poker'-type of pain located primarily behind his right eye. He notes that his eye waters profusely with the headache; in addition, his nose is initially congested, then it starts running. Only his right side is affected. The headache is so severe that he cannot work or sleep through it, and he is unable to concentrate on anything else. The headaches have been unresponsive to over-the-counter pain medications. The episodes seem to last about 1 hour. He denies any other symptoms. This patient has no chronic medical conditions, and he takes no regular medications. Question What is the most likely underlying pathophysiology of this patient's condition? Answer Choices 1 Antigen binds to IgE, triggering the release of histamines and other inflammatory substances 2 Growth of an intracranial tumor near the base of skull, leading to pressure on the cerebellum 3 Increased emotional stress, leading to contractions of head and neck musculature 4 Increased trigeminal nerve and parasympathetic activity, leading to vasodilation 5 Reactivation of varicella zoster virus, leading to inflammation in a ganglion ANS:4

4 Increased trigeminal nerve and parasympathetic activity, leading to vasodilation This patient presents with a history indicating cluster headaches. While the exact mechanism is unclear, it is known that there is increased trigeminal nerve and parasympathetic activity, leading to vasodilation in the intracranial vasculature. Key features of cluster headaches include unilateral pain, involvement of the eye and nose (autonomic/parasympathetic system), an episodic pattern of attacks, and excruciating pain. Antigen binding to IgE, leading to release of histamines, leukotrienes, and prostaglandins is the typical type I hypersensitivity reaction of allergies. Seasonal allergies could cause a headache, along with nasal and ophthalmic symptoms; however, this patient's severity of headaches exceeds a usual allergic sinus headache. If this patient's symptoms had been caused by an allergic response he would have presented with a history of allergies, chronic lower-grade headaches, and sinus symptoms; he would not have acute, episodic attacks of high intensity. While the growth of an intracranial tumor could present with headaches, this patient's history is not suggestive of cerebellar dysfunction. If there were a tumor in this patient's cerebellar region, some motor, and possibly language, dysfunction would be expected. Headaches with lacrimation and rhinitis are not typically seen from a tumor. Increased emotional stress and tight neck musculature is associated with tension-type headaches, which are the most common form of headaches. Typically, these headaches are more chronic; they are associated with psychosocial stressors and respond to analgesics. Nasal and ophthalmic symptoms are not seen with tension-type headaches. The reactivation of the varicella zoster virus (shingles) can produce severe throbbing, stinging, or other pain symptoms along the affected dermatome. Shingles would be a disorder to consider on this patient's differential, but the patient's young age, lack of skin lesions, and the episodic nature of the attacks fit much better with a diagnosis of cluster headaches.

A 35-year-old man presents with unusual sensations in his legs. He states that his symptoms began in college and were mild, but they have been steadily worsening over the last 4 years. His symptoms begin whenever he is lying down attempting to go to sleep. He describes the sensation as an uncomfortable crawly sensation deep within his legs. If he tries to ignore them, the feeling worsens. He has brief relief with movement, but the feelings return as soon as he is still again. He was given a trial of a benzodiazepine to take at bedtime, but it did not really help. In addition, he complains of being sleepy during the day as a result of the medication. He also tried leg massages and exercise without much relief. His past medical history is significant for depression and anxiety. The patient denies snoring, vivid dreams, or cataplexy. He states that he has a depressed mood and reduced energy and concentration, but he denies suicidal ideation. His physical exam is normal. Complete blood cell count and chemistry panels are normal. Question What is true regarding the patient's condition? Answer Choices 1 Men are affected more than women 2 Opioids are the drug of choice for this condition 3 The condition is caused by a deficiency in calcium 4 Symptoms usually resolve when underlying depression is successfully treated 5 An abnormal electrical impulse conduction study is frequently seen ANS:5

5 An abnormal electrical impulse conduction study is frequently seen An abnormal electrical impulse conduction study is frequently seen in restless legs syndrome (RLS), which is a neurological disorder characterized by unpleasant sensations in the legs and an uncontrollable urge to move when at rest in an effort to relieve these feelings. RLS sensations are often described by people as burning, creeping, tugging, or like insects crawling inside the legs. Often called paresthesias or dysesthesias, the sensations range in severity from uncomfortable to painful. Needle electromyography and nerve conduction studies should be considered if polyneuropathy is suspected on clinical grounds, even if results of the neurologic examination are apparently normal. The most distinctive aspect of the condition is that lying down and trying to relax activates the symptoms. Many patients with RLS have difficulty falling asleep and staying asleep. The disorder is diagnosed clinically by evaluating the patient's history and symptoms. Women may be slightly more affected than men. Calcium deficiency causes mainly muscle spasm and twitching, and is usually associated with other symptoms of arm and leg numbness, numbness around the mouth, nervousness, heart palpitations, and an inability to sleep, which are all not found in this case. In most cases, the cause of RLS is not known; in some cases, it may be caused by iron deficiency, folic acid deficiency, vitamin B12 deficiency, or arthritis. Sometimes, RLS can be associated with the use of antidepressant medications, necessitating the use of an alternative treatment.

Which of the following medication classes has been shown to improve the short-term rate of cognitive preservation in mild to moderate Alzheimer's dementia patients? A Acetylcholinesterase inhibitors B Decarboxylase inhibitors C Dopamine receptor agonists D Monoamine oxidase inhibitor E N-methyl-D-aspartate receptor antagonist ANS:A

A Acetylcholinesterase inhibitors Alzheimer's dementia is a chronic, progressive, neurodegenerative disorder. Acetylcholinesterase inhibitors have been associated with a modest decrease of cognitive decline and increased functioning. These medications increase the levels of acetylcholine, a neurotransmitter, and increase cholinergic activity within the affected brain regions. They have been approved for use in mild to moderate Alzheimer's disease. Memantine, an N-methyl-D-aspartate receptor antagonist, is indicated for use in moderate to severe disease, has been associated with decreased destruction of cholinergic neurons, and may slow cognitive decline. It is often added to acetylcholinesterase inhibitor therapy for progressed disease, but may be first line in a patient unable to tolerate the acetylcholinesterase inhibitor medications. Decarboxylase inhibitor medications inhibit the metabolism of dopamine; one example, carbidopa, is used in conjunction with L-dopa in the treatment of Parkinson's disease. Dopamine receptor agonists activate dopamine receptors in the absence of dopamine, and are useful in conditions with low dopamine levels, such as Parkinson's, prolactinomas, and restless leg syndrome. Selegiline, a monoamine oxidase inhibitor, is selective for monoamine oxidase-B and inhibits catabolic dopamine breakdown, with a potential neuroprotective effect. Use in moderately advanced Alzheimer's disease may slow progression, but remains controversial.

Which of the following etiologies is the most common cause of nontraumatic subarachnoid hemorrhage (SAH)? A Cerebral aneurysm B Poorly controlled hypertension C Anticoagulant use D Arteriovenous malformation ANS:A

A Cerebral aneurysm Congenital cerebral aneurysms or Berry aneurysms account for 75% to 80% of nontraumatic subarachnoid hemorrhages (SAHs). Poorly controlled hypertension and anticoagulant use are more commonly associated with intracerebral hemorrhages (ICH). AVMs can cause either SAH or ICH

A 72-year-old man presents with low-grade fever, nausea, confusion, and lethargy. His past medical history is significant for hypertension, hypercholesterolemia, and diabetes. He had sinusitis approximately 1 week prior to presentation; otherwise, he has been healthy. Laboratory workup shows a CSF with elevated opening pressure and blood glucose levels. They also show a low CSF glucose. A Gram stain on the CSF shows Gram-positive coccus. CBC shows an elevation of PMNs, but it is otherwise normal. What is the most appropriate treatment in this case? Answer Choices 1 Antibiotic therapy 2 Steroids 3 Supportive care 4 Transfusion 5 Antiviral treatment

ABX therapy This patient is suffering from meningitis, which is an infection of the meninges that causes inflammation. It may be acute or chronic, and many different agents can cause it. Acute meningitis develops in a few hours to a few days, and chronic meningitis may develop weeks to months after exposure. Symptoms may be different in children or in older adults with underlying health conditions, such as diabetes or other immunocompromised states, IV drug users, those on a shunt for hydrocephalus, and extremes of age. Instead of presenting with headache and nuchal rigidity, they may present with lethargy and confusion. Both infectious and non-infectious agents can cause this disease. There are many infectious agents, including bacteria, viruses, fungi, and parasites. The Gram-positive cocci in the patient's CSF points to Streptococcus pneumoniae as the organism responsible. Bacterial meningitis is treated mainly with antibiotic therapy. The treatment usually consists of ampicillin plus ceftriaxone or cefotaxime plus vancomycin when meningitis is suspected and no other data is available. Vancomycin is added to cover penicillin-resistant S. pneumoniae, which may be present. If a Gram stain has been done and shows Gram-positive cocci, the treatment of choice is penicillin G or ceftriaxone. Meropenem is the drug of choice for cephalosporin-resistant isolates. When deciding on an antibiotic regimen, it is important to keep in mind the person's age and health status. Steroids are not considered first-line treatment. However, they can be used as an adjuvant therapy. They are thought to help lessen the severity of symptoms. Dexamethasone may be considered to reduce cerebral edema. Supportive care is important, especially if the patient's disease is advanced. However, antibiotic therapy should be initiated as soon as possible to avoid complications or death. On rare occasions, anticonvulsants may be required. Transfusion is not considered the treatment of choice for bacterial meningitis. If the patient has an underlying anemia or lacks certain clotting factors, transfusions would be supplied as part of his care. However, there is no indication of this in this case. Since he has a Gram stain showing Gram-positive cocci in his CSF, antiviral therapy is not indicated in this patient. Antibiotic therapy should be initiated immediately, and supportive care should be given as needed.

A 56-year-old man presents with weakness of his grip. On exam, the problem is bilateral. During the next few office visits, you note the development of hyperactivity of his deep tendon reflexes (DTRs), extensor plantar reflexes, and dysarthria. The patient's sensory system remains normal, and he denies any urinary symptoms. Question What is the most likely diagnosis? Answer Choices 1 Multiple sclerosis 2 Alzheimer's dementia 3 Huntington's disease 4 Amyotrophic lateral sclerosis 5 Myasthenia gravis

ALS The clinical picture is suggestive of amyotrophic lateral sclerosis (ALS). ALS is a progressive bilateral muscle disease that causes fasciculations and hyperreflexia, plantar reflexes, and dysarthria. Sensation and bladder function remain normal. The common clinical presentation seen in multiple sclerosis is weakness, numbness, tingling, or unsteadiness in a limb; spastic paraparesis; retrobulbar neuritis; diplopia; disequilibrium; or a sphincter disturbance such as urinary urgency or hesitancy, which is not present in this patient. Alzheimer's patients present with memory impairment and at least 1 of the following: language impairment, apraxia, agnosia, and impaired executive function. Rigidity and bradykinesia are primary signs; tremors are rare. These symptoms are not present in this patient. Huntington's disease is characterized by chorea and dementia. Clinical onset is usually between 30 and 50 years of age. Patients present with writhing choreiform movements of the body, which are not present in this patient. In myasthenia gravis, small motor units (such as ocular muscles) are most often affected, producing the nystagmus. Weakness that improves after a period of rest and fluctuating fatigue are the major clinical features of this disease, and these symptoms improve after administering acetylcholinesterase inhibitors. These symptoms are not present in this patient.

A 9-year-old boy presents after an episode that his mother describes as "zoning out". He was eating dinner when he looked up and began to stare at the wall. He would not respond to her verbal questioning. Almost immediately, he lost control of his bladder. She stated that he then looked at her and realized he had wet himself; he then began to cry. Question What is the most likely diagnosis? Answer Choices 1 Grand mal seizure 2 Simple partial seizure 3 Complex partial seizure 4 Absence seizure 5 Atypical absence seizure

Absence Seizure The correct answer is absence seizure; they are characterized by an impairment of consciousness, occasionally with mild tonic, clonic, atonic, or autonomic symptoms. The patient is typically unaware of the attacks. They almost always begin in childhood, and they end by age 20. Atypical absence seizures are those characterized by more striking changes in tone; seizures may be more gradual in onset and completion than typical absence seizures. Patients with simple partial seizures do not experience a loss of consciousness. Complex partial seizures typically have impaired consciousness with focal motor symptoms or somatosensory symptoms, and they do not have the 3 phases. Grand mal seizures are characterized by a tonic phase, which includes a sudden loss of consciousness and rigidity that typically lasts less than a minute. A clonic phase follows; it is characterized by jerking motions, and they usually last for a few minutes. The clonic phase is followed by a flaccid coma state.

A mother brings her 10-year-old daughter to the pediatrician's office. Over the past few weeks, the girl's teacher has noticed the child staring off into the distance and pausing in the middle of a sentence. Yesterday, the child started twitching at her desk, but it only lasted a few seconds. The child does not seem to remember any of these claims. The mother states that her pregnancy with the child was without complications, and you note that the child has achieved all growth milestones uneventfully. The child is not currently on any medications. The mother has not noticed any of these behaviors at home. She adds that she and her husband are separating. What is the most likely cause of this child's behavior? Answer Choices 1 Simple partial seizure 2 Absence seizure 3 Emotional stress 4 Attention deficit disorder 5 complex partial seizure

Absence Seizures Absence seizure is correct because this child seems to be exhibiting episodic impairment in her consciousness, as evidenced by staring off into the distance and breaks in communication. She also had some twitching, which could be mild clonus. Absence seizures also typically present in children. Simple partial seizure is incorrect because she would not be exhibiting impaired consciousness during the episodes. The clonic activity would also be more apparent. Emotional stress is incorrect because this patient is exhibiting characteristic activity of an absence seizure. Attention deficit is incorrect because this patient is exhibiting characteristic activity of an absence seizure. Complex partial seizure is incorrect. Even though her consciousness is impaired, her clonic activity is minimal.

A 66-year-old English teacher is in the hospital. Her neurologist gives her a magazine to read, and she is unable to do so. What is this phenomenon called? Answer Choices 1 Paraphasia 2 Echolalia 3 Alexia 4 Apraxia 5 Agraphia ANS:3

Alexia When a patient is able to hear things and repeat them, it is called echolalia. With echolalia, the patient does not understand what he has heard. This is also referred to as echophrasia. Paraphasia is a type of aphasia. The substitution of a similar sounding word for another word is called paraphasia. With paraphasia, the words can also be jumbled. Alexia is a type of aphasia. An aphasia where there is a problem with reading is called alexia. Alexia is word blindness or text blindness. Alexia is also called optical aphasia or visual aphasia. Apraxia refers to the condition where a patient has difficulty performing motor acts, despite having the muscular capacity and coordination to do so. A patient with apraxia cannot execute the intended movement. A writing disturbance is called agraphia. There are various forms of agraphia. With absolute agraphia, even simple letters cannot be written. This is also referred to as literal agraphia.

A patient with early onset of hypertension, hematuria, and palpable kidneys was diagnosed to have adult polycystic kidney disease, and he is being evaluated for associated berry aneurysms. What investigation is considered the "gold standard" for the detection of aneurysms of the cerebral blood vessels? Answer Choices 1 MRI 2 Angiogram 3 PET scan 4 CAT scan 5 Electroencephalogram ANS:2

Angiogram Adult polycystic kidney disease is associated with cysts of the liver in 30 to 40% of patients. Other associations include berry aneurysms of the cerebral blood vessels, colonic diverticula, abdominal wall hernias, and mitral and aortic regurgitation. APKD is an autosomally dominant disorder associated with the PKD 1 gene on chromosome 16. Angiography involves taking several X-rays after the injection of a radio opaque dye in the blood vessel thus making the outline of the vessel visible. From this outline, it would be discernible as to whether there is or is not an aneurysm present. Computer-assisted tomography (CAT), like conventional radiography, produces X-ray images of various structures. The CAT scan, however, is able to reveal subtle differences that can distinguish gray and white matter, the ventricles, the thalamus, the basal ganglia, and others. It can produce an image of a single plane or section of the brain. (The term tomography comes from the Greek word meaning, "cut".) Positron emission tomography (PET) provides live, real time images of the functioning of the brain. PET scanning measures the differential rates of glucose metabolism in the active brain. Both CAT and PET images are obtained by a detector that rotates around the patient's head. However, CAT images reflect the absorption of radiation, whereas PET images are produced by the emission of radiation. Electroencephalography measures the electrical potentials of the surface layers of the cerebral cortex by means of electrodes placed on the skull.

A 31-year-old woman presents with a purpural rash covering her arms, legs, and abdomen. She also has fever, chills, nausea, abdominal tenderness, tachycardia, and generalized myalgias. Prior to the development of the rash, the patient noted that she had a headache, cough, and sore throat. Laboratory studies were positive for Gram-negative diplococci in the blood, along with thrombocytopenia and an elevation in PMNs. Urinalysis showed blood, protein, and casts. Vital signs are as follows: PB 92/66, P 96, RR 14, T 39. The patient denies any foreign travel and does not have any sick contacts. However, she does work part time as a nurse in a local hospital. Question The patient is diagnosed with Meningococcemia; she is admitted to the hospital and placed in respiratory isolation. What major course of therapy should this patient receive? Answer Choices 1 Steroids 2 Supportive care 3 Antibiotics 4 Transfusion 5 Bactericidal/permeability-increasing protein ANS:3

Antibiotics Antibiotics are the treatment of choice for meningococcemia. The preferred drug for active infection is penicillin G. For those allergic to penicillin, chloramphenicol and cephalosporins (ie, cefotaxime, cefuroxime) may be used as alternatives. Patients will also receive supportive care, but antibiotic therapy must be initiated quickly if the patient is to survive. Intensive care placement may be necessary if organ failure is imminent. Ventilatory support, inotropic support, and IV fluids are necessary in some. If adrenal insufficiency occurs, corticosteroid replacement may be considered. A central venous line helps to provide large amounts of volume expanders and inotropic medications for adequate tissue perfusion. Steroids have not been shown to play a major role in the treatment of meningococcemia. However, they have been used in addition to antibiotic therapy. In the case of adrenal insufficiency, for example, steroid replacement has been shown to be beneficial. Transfusion does not generally play a major role in treatment. If the patient suffers from a devastating coagulopathy, blood or blood products may be replaced as necessary. Bactericidal/permeability-increasing protein is a protein stored in the granules of neutrophils. It binds to endotoxin in vitro and neutralizes it. This technique is experimental, and it is not used in everyday treatment of meningococcemia.

A 45-year-old female presents with a sudden onset of vertigo, nausea, and vomiting. Upon physical exam, you note that she is holding on to the rails of the bed, and her pain gets worse when you attempt any movement of her head. Neurologic exam is grossly normal. Which combination of the following medications is indicated to treat the patient's symptoms? A Hydrochlorothiazide, lorezapam, and gentamycin B Lorezapam, prochlorperazine, and diphenhydramine C Scopolamine, aspirin, and cisplatin D Metoclopromide, hydrochlorothiazide, and cyclobenzaprine E Diazepam, hydrocodone, and hydrochlorothiazide ANS B

B Lorezapam, prochlorperazine, and diphenhydramine All other combinations include an ototoxic medication: furosemide, gentamycin, aspirin, and cisplatin. Treatment of acute vertigo is more effective using a combination of vestibular suppressants (benzodiazepines), anti-emetics (prochlorperazine), and anticholinergics (diphenhydramine or scopolamine).

A 37-year-old woman presents to her GP surgery with a history of right-sided facial weakness and peri-auricular discomfort since she awoke this morning. She is afebrile. Question What is the most likely diagnosis? Answer Choices 1 Trigeminal neuralgia 2 Bell's Palsy 3 Multiple sclerosis 4 Myasthenia gravis 5 Primary lateral sclerosis ANS: 2

Bell's Palsy Explanation The correct answer is Bell's palsy, which is a condition typically with sudden onset that affects the facial nerve, causing unilateral facial weakness. Trigeminal neuralgia presents with sharp pain on one side of the mouth that radiates to the ipsilateral ear, eye, or nostril. Multiple sclerosis is a demyelinating disorder, causing a multitude of symptoms that typically include diplopia or blurred vision early on, then an insidious onset of progressive weakness, numbness, and/or tingling in the extremities. Myasthenia gravis commonly presents with ptosis and diplopia, as well as difficulty swallowing, fatigue, and muscle weakness. Primary lateral sclerosis is an upper motor neuron disease that causes limb weakness, stiffness, and fasciculations.

A 37-year-old woman presents with a history of right-sided facial weakness and periauricular discomfort since she awoke this morning. She is afebrile. Question What is the most likely diagnosis? Answer Choices 1 Trigeminal neuralgia 2 Bell's Palsy 3 Multiple sclerosis 4 Myasthenia gravis 5 Primary lateral sclerosis ANS: 2

Bell's Palsy The correct answer is Bell's palsy, which is a condition with typically sudden onset that affects the facial nerve, causing unilateral facial weakness. Trigeminal neuralgia presents with sharp pain on 1 side of the mouth that radiates to the ipsilateral ear, eye, or nostril. Multiple sclerosis is a demyelinating disorder; it causes a multitude of symptoms that typically includes diplopia or blurred vision early on, followed by an insidious onset of progressive weakness, numbness, and/or tingling in the extremities. Myasthenia gravis commonly presents with ptosis and diplopia, as well as difficulty swallowing, fatigue, and muscle weakness. Primary lateral sclerosis is an upper motor neuron disease that causes limb weakness, stiffness, and fasciculations.

Radiculopathy due to nerve root compression occurs most commonly at which nerve root within the brachial plexus? A C5 B C6 C C7 D C8 E T1 ANS:C

C7 the C7 nerve root is affected the most often (approximately 45-60%). This radiculopathy can result from foraminal encroachment of the spinal nerve, cervical disk herniation, tumor, and multiple sclerosis. C7 radiculopathy can present with weakness in the triceps, which cause elbow extension, and finger flexion and extension. C6 is another common site of radiculopathy. C6 radiculopathy can present with weakness in the biceps, brachioradialis, and wrist extensor muscles. Cervical radiculopathy at the C5, C8, and T1 are less common, but still possible. C5 radiculopathy can present with deltoid and biceps muscle weakness. C8 radiculopathy can present with finger flexor weakness and T1 radiculopathy with finger abduction weakness.

A 28-year-old man presents following a road traffic accident 20 minutes prior. He states that it was just a "minor bump from behind," but feels he might have whiplash. His neck is stiff and sore, and he has developed numbness and tingling on the lateral surface of his right arm extending into his right fourth and fifth digits. On physical examination, his bicep strength is +5/5 on the left and +5/5 on the right. His biceps tendon reflex is 2+ on the left and 2+ on the right. In addition, his triceps tendon reflex is 2+ on the left and 2+ on the right. His grip strength on the right is diminished as compared to the left. The remainder of the physical examination is normal. Question Based on the above presentation, what is the cervical nerve root most likely affected? Answer Choices 1 C4 2 C5 3 C6 4 C7 5 C8

C8 Impingement of C8 may cause numbness and tingling primarily on the medial surface of the arm and into the lateral hand into the fourth and fifth digits. It may also cause dysfunction of the hand as it innervates the small hand muscles. Impingement of the C4 nerve root may cause neck and upper shoulder numbness and pain. Impingement of C5 nerve root may cause deltoid and shoulder numbness and pain, and biceps tendon reflex may be diminished. Impingement of the C6 nerve root can cause numbness and tingling down the arm into the thumb, with weakness in the bicep muscle and diminished brachioradialis tendon reflex in the affected extremity. Impingement of C7 also causes numbness and pain down the affected arm but into the middle finger, and the triceps reflex may be diminished on examination.

A 28-year-old man presents following a motor vehicle accident 20 minutes prior. He states that it was just a "fender bender," but he feels he might have whiplash. His neck is stiff and sore, and he has developed numbness and tingling on the medial surface of his right arm and into his right fourth and fifth digits. On physical examination, his bicep strength is +5/5 on the left and +5/5 on the right. His biceps tendon reflex is 2+ on the left and 2+ on the right. In addition, his triceps tendon reflex is 2+ on the left and 2+ on the right. His grip strength on the right is diminished as compared to the left. The remainder of the physical exam is normal. Question Based on the above presentation, what is the cervical nerve root most likely affected? Answer Choices 1 C4 2 C5 3 C6 4 C7 5 C8 ANS:5

C8 The most likely nerve affected in this case is the C8 nerve root. Impingement of C8 may cause numbness and tingling primarily on the medial surface of the arm and into the lateral hand into the fourth and fifth digits. Impingement may also cause dysfunction of the hand, as it innervates the small hand muscles. Impingement of the C4 nerve root may cause neck and upper shoulder numbness and pain. Impingement of C5 nerve root may cause deltoid and shoulder numbness and pain, and biceps tendon reflex may be diminished. Impingement of the C6 nerve root can cause numbness and tingling down the arm into the thumb, with weakness in the bicep muscle and diminished brachioradialis tendon reflex in the affected extremity. Impingement of C7 also causes numbness and pain down the affected arm but into the middle finger, and the triceps reflex may be diminished on exam.

A 35-year-old man presents with headaches. He describes them as only occurring on one side of his head; there is a "sharp stabbing pain just above his left eye". Each headache lasts for about 2 hours at a time; they occur daily for spans of 2 -3 weeks, and they have had the same pattern every 3 - 4 months for the past couple years. During these periods, the patient's nose feels congested and his left eye appears red and waters excessively. The patient has deduced that aggravating factors include alcohol and stressful situations. He denies any known family history of headaches similar to these; there is also no family history of migraines. The physical examination is within normal limits. Question Based on the given history and physical examination, what is an effective preventative medication that might prevent these headaches in the future? Answer Choices 1 Triptans 2 Calcium channel blockers 3 Local anesthetics 4 Oxygen 5 NSAIDs

CCB (like Verapamil) The patient in the above scenario is experiencing a cluster headache. Cluster headaches are headaches that occur more commonly in middle-aged men. Patients will complain of severe unilateral periorbital pain that occurs daily for several weeks and potentially are associated with 1 or more of the following symptoms: ipsilateral nasal congestion, rhinorrhea, lacrimation, or redness of the eye. Patients are restless and agitated during the acute phase of these headaches. Episodes vary in length from around 15 minutes up to 3 hours; at the same time, these occurrences may last for several weeks and then resolve for many weeks at a time. Any time a healthcare provider or a patient approaches a headache of any type, treatments for the acute signs/symptoms as well as preventative treatments should be considered. Fast acting treatments to give the patient immediate relief from the signs and symptoms of a cluster headache include oxygen, triptans, octreotide, local anesthetics, and dihydroergotamine (intravenous only). Various NSAIDs may also be attempted for symptoms, but not for prevention of cluster headaches. Preventative therapies are used at the onset of the cluster episode with the overall goal of suppressing the severity of the headaches. Various options may be attempted as preventative therapy; corticosteroids, lithium carbonate, nerve blocks, ergots (sublingual), calcium channel blockers, and even melatonin usage have been effective. At times, the anti-seizure medications divalproex and topiramate may also be considered. The calcium channel blocker verapamil is very often the first choice for preventing cluster headaches.

A 4-month-old febrile infant presents with loss of appetite, irritability, seizures, focal sensory and motor deficits, and an acute petechial rash. On physical examination, a bulging fontanelle is noted; rectal temperature is 102.8°F. What study would be most important in this child's evaluation? Answer Choices 1 CBC 2 Urinalysis 3 Chemistry panel 4 Serum glucose 5 CSF analysis ANS: 5

CSF analysis The clinical picture is suggestive of meningitis, which is diagnosed after obtaining and evaluating the cerebrospinal fluid (CSF). Although a bulging fontanelle is an indication of increased intracranial pressure (ICP), it is not an automatic contraindication for a lumbar puncture (LP) unless the patient displays additional signs of increased ICP. Tonic seizures would be an example of an indication of increased ICP. Even though this child has had seizures, an LP to run a CSF evaluation is still the best answer. This will be the most important study and diagnostic tool for this child. It is likely that a CT or MRI would be done to look at the level of swelling in the brain, and possibly determine the level of ICP. It would be likely for either of these tests to be done prior to a lumbar puncture. Normally, meningitis causes fever, inactivity, and mental status changes; however, these symptoms are often hard to detect in young children. In infants, signs and symptoms may include appearing to be slow or inactive (lack of alertness), irritable, vomiting, or feeding poorly. When CSF infections are suspected, blood should be obtained for a CBC, general chemistry panel, and culture. However, obtaining CSF analysis is the most important.

A 22-year-old man presents after being hit on the right temple by a baseball 1 hour ago. He lost consciousness for 2 minutes after the impact, but he did not seek immediate medical attention. He has no other significant medical problems and family history is unremarkable. On presentation, he is comatose, with no response to painful stimuli. There is anisocoria with fixed dilatation of the right pupil. Vital signs are as follows: BP 100 / 66 mm Hg, RR 14 rpm, HR 62 bpm, temperature 36.6 °C (97.9 F). What is the most appropriate next step to confirm the suspected diagnosis? Answer Choices 1 MRI of the brain 2 Perform a lumbar puncture 3 Burr hole craniotomy 4 Computed tomography of the brain 5 Observation only ANS:4

CT The signs and symptoms described in this clinical scenario are consistent with an acute epidural hematoma. This lesion's pathophysiology is a dural tear with laceration of an artery or, less commonly, a vein. The most commonly injured vessel is the middle meningeal artery (66% of cases), but lesions of the anterior ethmoidal artery and the dural sinuses may also occur. About 85-95% of these cases are associated with a fracture of the skull. However, there is little injury to the underlying intracranial components; because of this, the prognosis is good. Other findings, such as intracranial air or vascular malformations, may also be present. Due to the progressiveness of this pathology if not treated, immediate diagnosis and surgical intervention is required. The most commonly used method to make this diagnosis to date is a non-contrast CT of the head. Not only will this help locate the hematoma, this will also help visualize any other abnormalities that may be present, such as skull fractures. A CT scan would also enable the patient to be observed during the entirety of the procedure to ensure his stability and safety. MRI of the brain may be considered appropriate, however should not be used in any patients who are unstable. Due to the patient being comatosed with no response to painful stimuli, this would not be an acceptable choice. Burr hole craniotomy is used primarily in the treatment approach in a rapidly deteriorating patients with signs of imminent herniation. Although the patient is in a comatose state, his vitals are acceptable. Knowing if there is indeed an epidural hematoma as well as where it is located will both be accomplished via a CT of the head. A lumbar puncture would not be an acceptable choice in this patient scenario and in fact may result in detrimental results. Observation only would also most likely have devastating circumstances

A 28-year-old man presents with a history of headache, vomiting, and seizure. On examination he is febrile; pulse is 110/min; and blood pressure is 110/70mmHg. On central nervous system (CNS) examination, he appears alert; he is oriented to time, place, and person. He has aphasia and weakness of the right hand and leg. You suspect brain abscess. Question What initial test could confirm your diagnosis? Answer Choices 1 Magnetic Resonance Imaging (MRI) of brain with diffusion weighting 2 Computed Tomography (CT) scan of brain with contrast 3 CT scan of the brain without contrast 4 Stereotactic aspiration and examination of abscess fluid 5 Ultrasonography

CT with Contrast!! The correct answer is CT scan of brain with contrast. It is an excellent tool for establishing diagnosis and can demonstrate evidence of a ring-enhancing lesion in a well-defined abscess, as well as features of cerebral edema in the stage of cerebritis1. CT aids in determining the location of the abscess, its size, number of abscesses, mass effect, shifts, and the presence of intraventricular rupture. It provides information regarding the cause, and the paranasal sinuses and mastoids are also imaged concomitantly1. CT scan of the brain without contrast does not form a ring around the lesion, making the diagnosis difficult. Hence, it is not the right choice. Although magnetic resonance imaging (MRI) obtained with diffusion weighting may be more sensitive in the differentiation of an abscess from other cystic brain lesions, as well as in detection of the cerebritis stage, it may not be useful in an acutely ill patient, and routine MRI is not recommend to diagnose patients with a suspected brain abscess1. Stereotactic aspiration and examination of abscess fluid is used to make a definitive microbiological diagnosis by collecting pus from the abscess for testing. It is not used as an initial test to establish the diagnosis. Ultrasonography is not used for diagnosing a brain abscess in adults because the skull causes bone artifact. In children with an open anterior fontanelle, a sonogram can be used to diagnose an abscess1.

A 75-year-old man is involved in a motor vehicle accident and strikes his forehead on the windshield. He complains of neck pain and severe burning in his shoulders and arms. His physical examination reveals weakness of his upper extremities. What type of spinal cord injury does this patient have? A anterior cord syndrome B central cord syndrome C Brown-Séquard syndrome D complete cord transection E cauda equina syndrome ANS: B

Central Cord Syndrome the central cord syndrome involves loss of motor function that is more severe in the upper extremities than in the lower extremities, and is more severe in the hands. There is typically hyperesthesia over the shoulders and arms. Anterior cord syndrome presents with paraplegia or quadriplegia, loss of lateral spinothalamic function with preservation of posterior column function. Brown-Séquard syndrome consists of weakness and loss of posterior column function on one side of the body distal to the lesion with contralateral loss of lateral spinothalamic function one to two levels below the lesion. Complete cord transection would affect motor and sensory function distal to the lesion. Cauda equina syndrome typically presents as low back pain with radiculopathy.

A 20-year-old college student describes his headaches as unilateral, a 5 out of 10 for pain, and with throbbing, associated with photophobia and often accompanied by nausea and vomiting. He states that the headaches occur two to three times a month and are associated with decreased sleep. He has taken OTC medication with relief and is trying to stay on a sleep schedule. He denies any other neurologic symptoms. Which of the following is the most appropriate next step for managing this patient? A Brain MRI B Hydrocodone-acetaminophen tablets C Physical therapy D Topiramate E Sumatriptan nasal spray ANS:

Classic migraines often present with episodic, unilateral, and throbbing headache pain, associated with photophobia and phonophobia. Nausea and vomiting may also be involved. Management of migraines includes nonpharmacologic therapies, such as healthy eating, sleep pattern stabilization, caffeine avoidance, and stress relief. If nonpharmacologic therapies are not sufficient, or if symptoms impact activities of daily living, pharmacologic management is indicated. Medications are available for migraine sufferers and are typically catergorized as abortive or preventive. Abortive therapy for this patient may include nonsteroidal anti-inflammatory medications. Additionally, studies have shown that stimulation of the 5-HT receptors can successfully stop a migraine, and the 5-HT1 receptor agonists, with selective agents (the triptans) often used successfully for headache improvement. Nasal spray formulations are useful for nausea and vomiting. Topiramate, an anticonvulsant, has received FDA approval for migraine prevention therapy in patients with increasing migraine frequency or poor response to abortive therapies. Narcotics are typically avoided for migraine management, and MRI is not warranted without additional clinical signs and symptoms, and other differential diagnoses.

A 45-year-old man goes to a party and enjoys several glasses of an alcoholic cocktail. His past medical history is significant for headaches. The drinks trigger a unilateral right-sided headache. The headache is behind his right eye and spreads to his forehead. He also notices that his right nostril has a watery discharge and his right eye is tearing. He describes the pain as if he were "being stabbed in my eye". Question What kind of headache does he have? Answer Choices 1 Tension headache 2 Hangover headache 3 Common migraine 4 Classic migraine 5 Cluster headache

Cluster HA This patient has a cluster headache. Typically, cluster headaches occur in the spring and fall, as the case here. During a cluster period, the headaches can be triggered by alcohol. During an attack, there can be discharge from the nose and eye on the same side of the face that the headache is on. The pain of a cluster headache begins rapidly. The pain is described as stabbing or knifelike, as the case here. The headaches last up to 2 hours. Tension headaches are associated with stress. This kind of headache would not be associated with water discharge from the nose and eye. A hangover headache would not occur while the man is still drinking. This kind of headache would not be associated with water discharge from the nose and eye. Common migraine and classic migraine would not be associated with water discharge from the nose and eye.

A 40-year-old man presents with a 3-week history of a daily headache that awakens him from sleep. The headaches occur around the left eye. He notes that the left eye waters and his left nostril gets stuffy with each headache. What is the most likely diagnosis? Answer Choices 1 Sinusitis 2 Tension headache 3 Cluster headache 4 Intracranial mass 5 Migraine ANS:3

Cluster HA The clinical picture is suggestive of a cluster headache. Cluster headaches are typically seen in middle-aged men. Unilateral periorbital pain can occur daily for several weeks. They are often accompanied by ipsilateral nasal congestion, rhinorrhea, lacrimation, and redness of the eyes. Episodes often occur at night, and they last between 15 minutes and 3 hours. Sinusitis commonly presents with unilateral face fullness, pressure, and tenderness over the cheeks. Tension headaches are caused by muscle contractions that usually elicit a 'bandlike' tightness rather than throbbing pain. Often brought on by stress, they are usually not associated with nausea or vomiting, and they more commonly cause bilateral rather than unilateral pain. Intracranial masses cause an increase in intracranial pressure, which can cause headaches with vomiting. Headaches related to intracranial masses are typically worst in the morning, as recumbency increases intracranial pressure. Headaches due to intracranial pathology are likely to worsen over a short period and be progressive in nature, rather than present as a cyclical course. Migraine headache pain is usually described as pulsatile in nature and is commonly unilateral. Other common manifestations include nausea, vomiting, and photophobia. They are more common in women than in men.

A 62-year-old man presents with vision problems and difficulty swallowing. Over the past week, he has had a constellation of symptoms, beginning with numbness and tingling in his feet that progressed to weakness that now affects both lower and upper extremities. Within the past day, he has started to notice difficulty swallowing and double vision. He also feels it is difficult for him to take a big breath. His past medical history is noncontributory, and he takes no medications. Exam reveals bilateral absence of patellar and ulnar reflexes. Question What is the most likely prognosis for this patient? Answer Choices 1 Complete resolution of symptoms 2 Gradual resolution of paralysis and residual paresthesias 3 Gradual resolution of paresthesias and residual lower extremity weakness 4 Progressive paralysis and premature death most likely due to respiratory failure 5 Waxing and waning polyarticular weakness and paresthesias

Complete Resolution The vast majority of patients (>80%) with Guillain-Barré syndrome (GBS) experience a full recovery with supportive care. In addition to supportive care, plasmapheresis and/or intravenous immune globulin are often used to reduce the duration and severity of symptoms once the diagnosis is recognized. Symmetrical ascending paralysis as described in this patient is indicative of Guillain-Barré syndrome (GBS). The cause of GBS is unknown, but it is generally thought to be an inflammatory autoimmune process. More than 50% of patients with GBS report an antecedent illness. The antibodies produced in response to antigens present in the infectious agent are thought to cross-react with components of human neurons, prompting an acute post-infectious demyelinating process. Lumbar puncture characteristically reveals elevated CSF protein content. Other results are normal, although the white blood cell count may be somewhat elevated. Decreased CSF glucose and increased polymorphonuclear cell counts are seen in acute bacterial meningitis. Decreased CSF glucose and elevated CSF lymphocyte counts are commonly seen with meningitis caused by fungi. Viral meningitis usually presents with elevated lymphocyte counts, normal CSF glucose, and normal or slightly elevated CSF protein levels.

A 23-year-old woman with epilepsy discovers she is pregnant. The epilepsy has been well controlled with phenytoin for several years. What is the best course of action? Answer Choices 1 Discontinue all anticonvulsants 2 Switch to phenobarbital 3 Switch to valproic acid 4 Continue phenytoin at the same dose 5 Continue phenytoin and monitor monthly levels

Continue phenytoin and monitor monthly levels Control of epilepsy is especially important during pregnancy. Generalized seizures or status epilepticus in a pregnant woman poses a significant risk to the fetus. Ultimately, the best course of action is prevention; women of childbearing potential who have epilepsy should be under optimal control, preferably with a single agent. Either phenobarbital or phenytoin is ideal, although both are teratogenic. Standard prenatal supplements and/or vitamins should be taken based upon the individual patient. During pregnancy, changing volume of distribution and protein binding require frequent monitoring of drug levels; dosage adjustments should be made as necessary. Although it used to be standard practice to give vitamin K to pregnant women on antiepileptics, it is no longer a universal practice.

A 48-year-old woman presents with new-onset headache that she describes as nonspecific, worse on awakening, intermittent throughout the day but can worsen with bending over or coughing. Her husband reports that she has not been herself since the headaches started about 4 to 6 weeks ago. Which of the following tests would be best for determining the etiology of her presenting symptoms? A noncontrast head CT scan B lumbar puncture C contrast-enhanced brain MRI D noncontrast brain MRI E cerebral angiography ANS:C

Contrast MRI this patient has an intracranial mass until proven otherwise. Headaches starting later in life and accompanied by other neurologic or cognitive problems should raise a high suspicion of a tumor. Obtaining a contrast-enhanced MRI of the brain will demonstrate an intracranial mass lesion. The contrast will follow blood flow distribution and help in determining possible tumor type. Some lesions are difficult to see without contrast enhancement. MRI scans have a much higher resolution for soft tissue over CT scans and are preferred for looking at brain parenchyma. Prior to the advent of CT and MRI, cerebral angiography was used to look for intracranial masses. Vascular tumors have characteristic blush patterns, and if a mass effect is present, it will distort the position of the blood vessels.

A 50-year-old woman with Hodgkin's disease has been treated with cytotoxic drugs and steroids. She is admitted to the hospital with a 5-week history of headache, low-grade fever, visual disturbances, and confusion. The patient also has a 2-month history of progressive memory loss. Physical examination reveals a stiff neck. After lumbar puncture, initial studies reveal a WBC of 100 cells/mm3, glucose of 20 mg/dL, and protein of 100 mg/dL. CSF testing with India ink is positive. What is the cause of the CNS disease? Answer Choices 1 Bacterial meningitis 2 Aseptic meningitis 3 Herpes simplex virus encephalitis 4 Tuberculous meningitis 5 Cryptococcal meningitis

Cryptococcal Meningitis

Which of the following diagnostic studies is indicated for a patient with amaurosis fugax? A CT of the head B Intraocular pressure C Temporal artery biopsy D Carotid ultrasound E Ocular fluorescein angiogram ANS:D

D Carotid Ultrasound Amaurosis fugax is a monocular vision loss that appears like a curtain passing over the eye, and comes from carotid artery disease. A CT of the head is indicated for lateralizing stroke symptoms. Intraocular pressure is taken for evaluation of chronic or acute glaucoma. A temporal artery biopsy is taken if giant cell arteritis is suspected. An ocular fluorescein angiogram is done to evaluate retinal disorders.

Which of the following findings is consistent with a lower motor neuron deficit? A aphasia B dysdiadochokinesia C sensory loss D weakness E hyperreflexia ANS D

D Weakness Weakness is one potential finding of a lower motor neuron process. Hyporeflexia is also common. Aphasia results from injury to the speech pathways within the brain. Sensory loss arises from many causes, but it is not a motor issue. Dysdiadochokinesia is consistent with cerebellar pathology. Hyperreflexia is typically a signal of upper motor neuron disease.

A 70-year-old woman presents with a 1-year history of short-term memory loss. She did not seek medical attention earlier because she believed the memory loss to be age-related. Her husband has also noticed that she is unable to perform her regular daily activities and is often misplacing things. Recently, she was unable to get back home after her evening walk, and neighbors brought her home. The husband also feels she has been acting strange lately, and she is suspicious of her own son. Her past medical history is significant for hypertension and she has been on medication for the past 3 years. On clinical examination, she appears conscious and alert. Her vitals are PR- 70/min, BP- 120/74 mmHg. She is afebrile. Question What is your provisional diagnosis? Answer Choices 1 Dementia of the Alzheimer's type 2 Dementia due to a medical condition 3 Vascular dementia 4 Major depression 5 Substance induced dementia

Dementia of the Alzheimer's Dementia of the Alzheimer's type involves multiple cognitive deficits including 1 or more of the following: aphasia or language disturbance; apraxia, or the inability to carry out motor activities despite intact motor function; agnosia, or the inability to recognize familiar objects; and the disturbance of executive activities, such as planning, organizing, sequencing, or abstracting. There can be changes in personality and behavior, in addition to the inability to learn and recall new information. Long-term memory is affected. The disease also has a gradual onset and continuing decline. Focal neurological signs and symptoms are usually absent. If the disease occurs before age 65, this is early-onset. If it occurs after age 65, the disease is late-onset. Vascular dementia has the same general symptoms and signs as Alzheimer's disease, except that the onset is usually more sudden and acute. In addition, it is picked up easily by clinical or laboratory evidence of a vascular cause, a history of cerebrovascular disease, or multiple infarctions. Dementia due to medical conditions may present with similar or the same symptoms as other dementias, but are due to other medical or neurological diseases such as head trauma, Parkinson's disease, Pick's disease, Huntington's disease, HIV, multiple sclerosis, vitamin deficiencies, and numerous other medical conditions. Many elderly suffer from major depression. Symptoms include short-term or recent memory loss, fatigue, insomnia or hypersomnia, low appetite, weight loss, overeating, and an inability of handling activities of daily living. In major depression, deficits in various cognitive domains may be present. Temporal lobe magnetic resonance imaging (MRI) is helpful in differentiating dementia of the Alzheimer's type from depression and other disorders that may cause diagnostic difficulties in clinical practice. Substance-induced dementia includes cognitive defects caused by inhalants, sedative drugs, hypnotic drugs, prescription side effects, overdoses of prescription drugs, or other substances. This is common in the elderly.

You have completed comprehensive testing on an 80-year-old woman, and the diagnosis is dementia of the Alzheimer's type. You believe she is in the very early stages of the disease and want to try a medication to possibly slow disease progression. What drug may be an effective treatment? Answer Choices 1 Cognex (Tacrine HCL) 2 Vitamin E 3 Risperdal (Risperidone) 4 Donepezil (Aricept) 5 Haldol (Haloperidol) ANS:4

Donepezil (Aricept) Alzheimer's dementia accounts for over 60% of dementia in older adults. It develops with multiple cognitive deficits, including memory impairment and 1 or more of the following: aphasia (language disturbance), apraxia (difficulty in carrying out motor activities), agnosia (the inability to recognize familiar objects or people), and/or disturbance in organizing and planning. These symptoms usually develop over a gradual period of time and represent a distinct decline from previous functioning. Another mental illness or serious medical problem does not account for the symptoms. To make an accurate differential diagnosis, the physician must first complete a history and physical exam in order to rule out physical causes of the dementia. A complete mental status exam must then be done, along with initial lab testing. All other additional testing, such as neurological or psychological testing, should be done at a later stage after all other physical causes have been ruled out. Also, it is very important to refer the patient and family to a medical social worker at some point in order to assist the family in community resources and emotional support, especially if the diagnosis becomes dementia of the Alzheimer's type. There are no current cures for Alzheimer's disease and no one really knows what causes it. There are 2 major drugs being used to treat some of the symptoms: tacrine (Cognex) and donepezil (Aricept). Donepezil has become the treatment of choice because it has once-a-day dosing; there is also no need for close monitoring, and it causes fewer side effects. Tacrine has multiple side effects and requires close liver monitoring. Vitamin E is being used to reduce the effects of drugs, but it is not a definitive treatment of choice. Risperidone (Risperdal) and haloperidol (Haldol) are being used as later choice drugs to treat later-stage patients who have developed psychotic or agitated reactions. Both have side effects, have to be more closely monitored, and are usually prescribed by a psychiatrist/liaison. Normal pressure hydrocephalus is characterized by gradual onset of dementia, gait disturbance, and usually urinary incontinence. Pick's disease is very rare and presents itself with changes in personality, deterioration in social skills, emotional flatness, and significant language problems. Vascular dementia usually has a more sudden onset, and there is evidence of cerebrovascular disease with focal neurological signs and symptoms and usually a history of a previous stroke. Pseudo dementia is the term used to describe false dementia or major depression in the elderly; it usually consists of depressed mood, crying episodes, hopelessness, helplessness, and fatigue. Patients may also present with insomnia or hypersomnia.

Your patient is a 66-year-old female who has been dropping her coffee cup and concurrently slurring her speech. The episodes last for approximately 15 minutes. Her blood work, carotid dopplers, and MRI of the brain are normal and you suspect recurrent transient ischemic attacks (TIAs). Which of the following is NOT approved or recommended for the prevention of stroke in this patient? A aspirin B extended-release dipyridamole plus aspirin C clopidogrel D prasugrel E warfarin ANS:E

E Warfarin Aspirin, aspirin plus extended-release dipyridamole, and clopidogrel are all antiplatelet agents and approved for use to reduce recurrent TIAs and ischemic cerebrovascular accidents (CVAs). Prasugrel is not FDA-approved for this indication. Warfarin is an anticoagulant and has no role in prevention of either recurrent TIA or ischemic CVA.

A 62-year-old man presents with vision problems and difficulty swallowing. Over the last week, he has had a constellation of symptoms; they began with numbness and tingling in his feet and progressed to weakness that now affects both lower and upper extremities. Within the last day, he has started to notice difficulty swallowing and double vision. He also feels it is difficult for him to take a big breath. His past medical history is noncontributory, and he takes no medications. Exam reveals bilateral absence of patellar and ulnar reflexes. A lumbar puncture is performed to confirm the diagnosis. Question What cerebrospinal fluid (CSF) finding is most likely? Answer Choices 1 Decreased CSF glucose content 2 Decreased CSF protein content 3 Elevated CSF polymorphonuclear cell count 4 Elevated CSF protein content 5 Elevated CSF lymphocyte count

Elevated CSF proteins Correct response is elevated CSF protein content. Symmetrical ascending paralysis as described in this patient is indicative of Guillain-Barré syndrome (GBS). The cause of GBS is unknown, but it is generally thought to be an inflammatory autoimmune process. More than half of patients with GBS report an antecedent illness. The antibodies produced in response to antigens present in the infectious agent are thought to cross-react with components of human neurons, prompting an acute postinfectious demyelinating process. Lumbar puncture characteristically reveals elevated CSF protein content. Other results are normal, although the white blood cell count may be somewhat elevated. Decreased CSF glucose and increased polymorphonuclear cell counts are seen in acute bacterial meningitis. Decreased CSF glucose and elevated CSF lymphocyte counts are commonly seen with meningitis caused by fungi. Viral meningitides usually presents with elevated lymphocyte counts, normal CSF glucose, and normal or slightly elevated CSF protein levels.

A 3-year-old girl presents with terrible head pain, increasing fever, and irritability. On examination, you find the child febrile, 103o F, and she cries when you lift her head from the examining table. The neck appears to be quite stiff when manipulated. You perform a lumbar puncture. What findings would you expect if this child had bacterial meningitis? Answer Choices 1 Elevated protein 2 Elevated glucose 3 Decreased number of WBC 4 Decreased opening pressure 5 Negative Gram stain

Elevated Protein Bacterial meningitis is usually an acute infection of the subarachnoid space and meninges caused by a variety of organisms. In the neonatal period, infection with group B. streptococcus, E. coli, Listeria monocytogenes, and other Gram-negative enteric bacteria are the most likely. In children, the most important pathogens are Haemophilus influenzae (most common), Neisseria meningitidis (meningococcus), and Streptococcus pneumoniae (pneumococcus). In adults, pneumococcus and meningococcus are the most important causes of meningitis. Older individuals are at increased risk for infection with Listeria and Gram-negative enteric bacteria. The classic triad of symptomsof meningitis include fever, headache and stiff neck. Respiratory distress, increased response to auditory stimulus, and increased Babinski reflex are not characteristic of presenting symptoms of this disease. Pallor is not characteristic, either. Once the diagnosis of meningitis is suspected, immediate examination of the spinal fluid is indicated with lumbar puncture. Spinal fluid abnormalities usually include elevated protein, low glucose (hypoglycorrhachia), elevated opening pressure, and elevated white cells in the 100 - 10,000 cells/cubic mm range with a predominance of polymorphonuclear cells. The Gram stain is usually positive in more than 90% of patients.

A 37-year-old man fell from a ladder as he finished hanging the Christmas lights on his house. The right side of his head hit the alley cement, and he lost consciousness for about 1 minute; he woke up with a headache, but he had no other complaints. A few hours later, the patient is brought to the emergency room by his neighbor because of an intense headache, confusion, and left hand hemiparesis. On examination, the patient has a bruise located over the right temporal region, mydriasis, and right deviation of the right eye, papilledema, and left extensor plantar response. An emergency CT scan of the head without contrast reveals a lens-shaped hyper-density under the right temporal bone with mass effect and edema. What is the most likely diagnosis? Answer Choices 1 Epidural hematoma 2 Subdural hematoma 3 Subarachnoid hemorrhage 4 Intracerebral parenchymal hemorrhage 5 Acute meningitis ANS: 1

Epidural Hematoma Epidural hematoma most often results from a traumatic tear of the middle meningeal artery. Although a lucid interval ranging from minutes to hours followed by altered mental status and focal deficits is typical for epidural hematoma, this clinical picture is only encountered in up to 1/3 of the patients. The collection of blood between the skull and dura mater causes an evident mass effect with ophthalmic nerve palsy and the contralateral hemiparesis. Surgical evacuation of the clot via burr holes is the treatment of choice. Subdural hematoma results from a traumatic rupture of the bridging veins that connect the cerebrum to the venous sinuses within the dura. This venous hemorrhage will result in a gradual increase of the hematoma, with a progressive clinical picture over days or weeks. The CT scan will show a concave, crescent-shaped hyper-density compared to the convex, lens-shaped hyper-density in epidural hematoma. Subarachnoid hemorrhage is the result of an aneurysm rupture; the most common is the congenital berry aneurysm. The clinical picture is of a sudden, severe headache with meningeal irritation. A CT scan will show blood in the subarachnoid space, and a lumbar puncture will reveal xanthochromia CSF. Intracerebral parenchymal hemorrhage is most likely caused by hypertension complicated with Charcot-Bouchard aneurysms. The blood accumulates into the brain substance and most commonly involves the basal ganglia. Acute meningitis is not associated with trauma. Fever and signs of meningeal irritation dominate the clinical picture. Lumbar puncture, indicated if there are no focal neurological signs on clinical examination, will be the diagnostic procedure. The CT scan of the patient presented in this case is characteristic for epidural hematoma, and there is no indication for a lumbar puncture.

A 52-year-old woman presents with a 1-year history of bilateral shaking in her hands. The shaking tends to worsen when she is using her hands, and her symptoms improve when she drinks a glass of wine on the weekends. Her 82-year-old mother also has a similar shaking in her hands which developed when she was in her 50s. Question What is the most likely diagnosis? Answer Choices 1 Alcohol withdrawal 2 Parkinson's disease 3 Essential tremor 4 Huntington's disease 5 Multiple sclerosis ANS:3

Essential tremor (or benign tremor) typically involves the hands or head, but it spares the legs. The cause is unknown, but it may be inherited. Patients note that the tremor develops when moving the hands. The symptoms may worsen in times of stress, and they may be alleviated by alcohol intake. Alcohol withdrawal may be associated with a tremor, but is unlikely given the duration of the tremor. The tremor of Parkinson's disease is typically a resting tremor; this patient is not experiencing other symptoms of Parkinson's disease (e.g., rigidity, bradykinesia, and postural instability). Huntington's disease may be associated with abnormal movements and is inherited in a familial matter; however, Huntington's disease is typically fatal within 20 years of onset. This patient notes that her mother has been suffering from her tremor for close to 30 years. Multiple sclerosis is not typically associated with tremor.

A 62-year-old woman presents with excruciating pain, burning, and swelling in her left forearm and wrist. She reports that symptoms initially began with a fracture 4 months ago. The fracture was casted, and the patient was told it had healed well with cast removal at 8 weeks. She is frustrated because her symptoms have persisted and worsened, rather than improving, as she was told they would. The patient has continued to use a sling and limit use of the left arm so she doesn't worsen her condition. She is unable to wear a jacket or long sleeves, as even the fabric touching her skin causes pain. She denies fevers, pain in other areas, and new trauma. She denies polyuria and polydipsia. Her past medical history is unremarkable; she is a menopausal woman, with no known medical conditions, no history of surgeries, no regular medications and no allergies. She lives with her husband, and she is a homemaker. She denies drug, alcohol, and tobacco use. On physical exam, she is a small, thin, pleasant woman, and she is fully oriented. Vitals are normal. No gait or balance abnormalities are noted when she walks or gets onto the exam table. Her left forearm has some mild edema and erythema, as well as tenderness with even light touch. Distribution of findings includes the region from the elbow to wrist, both the anterior and posterior surfaces. Left wrist strength and range of motion are decreased compared to the right. Distal pulses, capillary refill, and reflexes are normal. The remainder of the exam, including mental status, is normal. Question What medication would be most appropriate for this patient's likely diagnosis? Answer Choices 1 Gabapentin 2 Heparin 3 Methadone 4 Probenecid 5 Vancomycin ANS:1

Gabapentin This patient's diagnosis is most likely a complex regional pain syndrome (CRPS). CRPS most often develops after a minor trauma and classic characteristics include pain out of proportion with findings and history, allodynia (pain sensation with normally non-painful stimuli) and motor and sensory disturbances in the affected extremity. The mechanism for CRPS development is not well understood. Prolonged immobilization following injury is a risk factor for development of CRPS. Diagnosis is clinical, and testing is done to rule out other disorders. Treatment is multi-modal and primarily consists of physical therapy, focusing on mobilization and desensitization. Other treatments are often off-label and targeted at chronic pain relief; they include corticosteroids, bisphosphonates, tricyclic antidepressants, anticonvulsants, and topical anesthetics. Gabapentin would be a reasonable choice for this patient. Heparin would be used if this patient's symptoms were attributed to a deep venous thrombosis (DVT). DVT is rare in the upper extremities; it is not associated with the hyperesthesia and allodynia shown in this patient. Methadone, a longer-acting opioid medication, can be used for chronic pain. However, for several reasons, this is not the most appropriate medication to use in CRPS. Opiates carry addiction potential and increase fall risk in the elderly. CRPS is often categorized as early (<6 months' duration) versus late (>6 months). For early CRPS, opiates should be avoided and even with late CRPS, other medications options should be utilized before initiating opiates. Probenecid is a medication which inhibits urate resorption. It is used for gout, a painful inflammatory condition which may cause acute pain, erythema and swelling in the affected joint(s). Gout tends to be more acute and affect joints (not the forearm, as with this patient). Vancomycin is an IV glycopeptide antibiotic; it is used for severe bacterial infections, including cellulitis. Cellulitis, which commonly presents with erythema, edema and tenderness in an extremity, may be considered on the differential for this patient. Cellulitis tends to occur more acutely. This patient had a normal 8 week exam with symptoms of CRPS, but no findings of cellulitis at that time.

A 32-year-old man is admitted to the hospital following loss of consciousness. The patient had been ill with fever and headache for several days, then developed double vision, confusion, and loss of consciousness. He has been otherwise healthy with no past medical conditions. Imaging reveals edema of the frontal lobe. Question Which virus most commonly causes the patient's infection? Answer Choices 1 Herpes simplex virus 2 Adenovirus 3 Influenza A 4 West Nile Virus 5 Epstein-Barr Virus ANS:HSV

HSV Herpes simplex virus is correct. The patient is suffering from encephalitis, which is typically viral in nature. Herpes simplex virus is the most common viral cause of encephalitis. Adenovirus is a rare cause of encephalitis. Adenovirus more commonly causes respiratory and eye infections. Encephalitis is a rare complication of these conditions. Influenza A is a rare cause of encephalitis. Influenza A typically causes respiratory infection, which is infrequently complicated by encephalitis. West Nile Virus is a rare cause of encephalitis. While about half of all patients who are diagnosed with West Nile Virus have meningitis and encephalitis, the overall rate of West Nile Virus is low. Epstein-Barr Virus is incorrect. Epstein-Barr virus causes infectious mononucleosis. It is estimated that only 1-5% of cases of infectious mononucleosis have nervous system involvement.

A 22-year-old engineering student presents after a major motor seizure. The patient was well until 3 days prior to presentation, when he developed generalized headache and a fever of 101.2 F (by mouth), for which he took aspirin. He later complained that he could not concentrate on his studies, and he added that all his teachers wanted to flunk him anyway. On more than 1 occasion, he accused his roommate of stealing his food from their shared refrigerator. He complained that he could not smell his food. Several times, he was noted to be having staring spells and lip smacking; during these episodes, he did not respond to his roommate. The spells lasted less than 1 minute. On the day of presentation, the roommate returned to the dorm to find the patient standing naked in the hallway outside their room; he was screaming "We're going to settle this now!" He then fell to the floor and sustained a major motor seizure. The patient's mother denies a history of major illness, but reveals he had measles and chicken pox as a child. The roommate states that the patient drinks a lot of beer at weekend parties and smokes marijuana occasionally, but he denies knowledge of other drug use. There is no history of seizures. Exam shows a well-developed, lean, muscular young man. He is lying on a gurney and fighting his restraints. He is alert but mute, and he is unresponsive to the spoken word. Temperature is 101.2 F (taken rectally). Heart rate is 110 and regular; respirations are 22 and unlabored. Blood pressure is 140/85 mm Hg. Neurological exam is limited by patient agitation and lack of cooperation, but it reveals normal pupils and eye movements, full symmetric facial movements, a mild right hemiparesis, and right extensor plantar response. Labs include complete blood count with a white blood cell count of 8,500; there is a normal differential. The chemistry profile, chest X-ray, and electrocardiogram are normal. Stat computed tomography of the head with contrast reveals left inferomedial temporal and frontal confluent areas of hypointensity with surrounding edema as well as scattered areas of hemorrhage in the same regions. These areas enhance densely. Cerebrospinal fluid exam reveals moderately elevated pressure, mildly elevated protein, and 267 white blood cells (80% lymphs, 20% polymorphs). Glucose is normal. Question What is the most likely cause of this patient's illness? Answer Choices 1 Herpes simplex encephalitis 2 Acute hemorrhagic leukoencephalitis 3 Subdural empyema 4 Cerebral abscess 5 Cerebral venous thrombosis 6 Septic embolism 7 Ruptured saccular aneurysm ANS:1

HSv encephalitis This is a rather classic presentation for focal encephalitic form of herpes simplex encephalitis. A prodrome of fever and headache is common. Anosmia, apparent temporal lobe seizures, and personality changes point to the inferomedial temporal and frontal regions. A convulsion is frequently the event that precipitates admission. The computed tomogram is classic. The cerebrospinal fluid is typical, but it can be normal early in the clinical course. This is the most common and serious of the acute encephalitides. This is a very grave illness, and it must be recognized and treated with antiviral agents (acyclovir) as soon as possible. Acute hemorrhagic leukoencephalitis is seen mostly in children, but it can also be seen in young adults. However, it is almost always preceded by symptoms of respiratory infection of 1 - 14 days duration. These patients may also develop headache and confusion, but neck pain and signs of meningeal irritation are more common and pronounced. Anosmia and temporal lobe seizures would be less common or pronounced. In this condition, white blood cell counts as high as 20,000 - 30.000 can be an important point of differential. Cerebrospinal fluid may be identical, but pleocytosis of up to 3,000 can help to differentiate it from herpes simplex encephalitis. The most helpful information at this point in differentiating this condition is the computed tomogram and magnetic resonance imaging scan, which would be expected to show diffuse asymmetric, large, and confluent edematous lesions in the white matter, with multiple small hemorrhages in both white and gray matter. Subdural empyema is a suppurative condition between the inner surface of the dura and the outer surface of the arachnoid. Since it occurs in a preformed space, it is properly termed an empyema, although it is frequently and erroneously called subdural abscess. It most commonly follows a worsening of chronic sinusitis, in which cases it most commonly involves adolescent and young men. There is usually a history of worsening of brow or mastoid pain and nose or outer ear canal purulent discharge. Fever, malaise, and headache appear. The headache pain worsens, and then becomes generalized; vomiting is frequent. Focal neurological signs may appear, and stupor and coma may ensue. Fever, leukocytosis, and stiff neck are almost universal. Cerebrospinal fluid is rather similar to that seen in herpes simplex encephalopathy. However, computed tomography scan of the head shows ear or sinus lesions and possible bone erosion. Enhanced meninges around the pus may be seen. Magnetic resonance imaging is more likely to visualize the empyema. About 40% of cerebral abscesses are related to diseases of the paranasal sinuses, middle ear, and mastoid air cells. Other conditions include bronchiectasis, lung abscess, acute bacterial endocarditis, congenital heart disease, pulmonary arterial venous malformation (AVM), abscessed teeth, osteomyelitis, or other sites of bacterial infection. The clinical course may be as seen in this patient, but onset and progression are much more variable depending on where the abscess is and the stage of development it has reached. The main concern here would be in differentiating herpes simplex encephalopathy from temporal or inferior frontal abscess. Spinal fluid is usually quite similar, and while finding bacteria might resolve the issue, this is rarely the case. Here, contrast studies are most helpful. Computed tomography would show a central hypodensity, a surrounding ring of enhancement, and hypodensity beyond the enhancing ring. MRI T1-weighted images show capsular enhancement with central hypodensity. T2-weighted images show capsular hypodensity, and surrounding edema is obvious. A septic embolism usually produces sudden neurological deficit, with the clinical picture depending on the site of ischemia. It might be mild and brief, its history being obtained only later when the patient presents with a clinical picture of expanding intracranial lesion (abscess), or it may present as a major acute typical appearing ischemic cerebrovascular accident. If it formed a temporal or frontal abscess, magnetic resonance imaging and computed tomography would probably be definitive. They may also form mycotic aneurysms of the more distal cerebral arteries; in turn, they may rupture, producing a picture of acute subarachnoid hemorrhage with or without intracerebral hemorrhage. Elevated sedimentation rate, leukocytosis, fever, malaise, fatigue, and known heart disease should prompt a thorough search for a septic embolic source. Ruptured saccular aneurysms are not usually a problem. There should be no prodrome (preceding headache, fever, stiff neck, malaise, abnormal behavior, etc.). The patient would be expected to have sudden severe headache, nausea, vomiting, and possibly loss of consciousness. There would be no fever or leukocytosis (except for a possible stress pattern). Focal neurological deficits are the exception, at least initially. Optic disc hemorrhages may be seen. Computed tomography generally shows blood (90%) in the subarachnoid spaces, which can be dramatic, or show only a subtle shadow along the tentorium or in the Sylvian or adjacent fissures. Rarely, the bleed may extend into brain tissue, producing an intracerebral hematoma. In the 5 - 10% of cases where no blood is seen on computed tomography or magnetic resonance imaging, a lumbar puncture generally reveals red blood cells in the thousands to a million, frequently with xanthochromia after several hours.

A 10-year-old boy is referred to you for being fidgety at school, though he makes good grades. Prior history is unremarkable, and there has been no recent illness. The father had a similar history as a child. While speaking with the patient, you notice that he clears his throat several times per minute. Examination is otherwise normal, except for rapid, nonrhythmic jerking movements of the face, neck, and shoulders while at rest. Question What is true of this patient's case? Answer Choices 1 Does not have Tourette syndrome due to the absence of coprolalia 2 Has a factitious disorder 3 Has a condition that clusters in families 4 Must start a regimen of levodopa 5 Suffers from Sydenham chorea

Has a condition that clusters in families The combination of vocal tics (here, throat clearing) and motor tics should suggest Gilles de la Tourette (Tourette) syndrome, which is a common tic disorder that clusters in families. Once thought to be a single gene, autosomal dominant condition, it appears that multiple genes and nongenetic factors may be at play. Coprolalia is but one of many vocal tics seen in Tourette syndrome (throat clearing, humming, whistling) and, although a notorious one, is not a necessary feature. The positive family history makes it unlikely that the condition described is factitious. Pimozide, SSRIs, and other agents have been used in the treatment of Tourette syndrome, but levodopa is not used. The movement disorder of Sydenham chorea, as seen after a streptococcal infection, is choreiform - that is, tends to be slower and rhythmic, as opposed to the rapid, nonrhythmic jerks seen in Tourette syndrome. Also, vocal tics are not typically seen with Sydenham chorea.

A 24-year-old man is brought in by his roommates for a seizure. The patient has alternative episodes of consciousness and is not currently seizing. The roommates give a history of the patient being in previously excellent health and studying exercise science in college. He first complained of feeling ill about 3-4 days ago, with headache, fever, and malaise. By last night, the roommates noted the man was not making sense in conversation, having trouble swallowing, and appeared more lethargic. Early this morning, he also had a witnessed seizure. Vitals are shown in the table. Weight 182# Height 72" Pulse 106 Blood pressure 102/58 Temperature 104.1°F/40.1°C On physical exam, the patient appears lethargic and intermittently responds to loud voice stimulus. He does not verbalize any answers and appears to have a hemiparesis. Other than his abnormal neurologic status and mild tachycardia, the remainder of his physical exam is normal. Multiple tests are done and results are shown in the table. Urinalysis Normal Complete blood count Mildly elevated white blood cell count (WBC) Comprehensive metabolic panel Normal C-reactive protein Elevated Toxicology screen Negative Cerebrospinal fluid analysis Increased lymphocytes and protein, normal glucose MRI of head Some areas of increased signal, no masses Brain biopsy Edema, inflammation, Negri bodies Question If the patient was able to supply an accurate, complete history by himself, which of the following aspects in this patient's history would best explain his current condition? Answer Choices 1 He had experienced multiple concussions while playing football as a teen. 2 He had recently started a new oral medication for acne. 3 He was the victim of a bat bite while spelunking. 4 His father had Huntington disease. 5 His mother consumed large amounts of alcohol throughout his fetal gestation.

He was bit by a bat This patient presents a very concerning clinical picture for encephalitis. Encephalitis classically presents with fever, headache, altered mental status, and a variety of other neurologic signs and symptoms. It is caused by a variety of viral organisms, ranging from herpesvirus, influenza, HIV, adenovirus, and many others. It can be transmitted by a variety of vectors worldwide, and history is vital to establishing possible etiologies. If this patient was able to report he was the victim of a bat bite while spelunking, that history would be most supportive of a diagnosis of rabies encephalitis. Encephalitis is a complicated condition with a variety of findings, but the Negri bodies on brain biopsy are pathognomonic for rabies. If he had experienced multiple concussions while playing football as a teen, the patient may be suspected of having postconcussion syndrome and/or mild traumatic brain injury (TBI). Both are associated with headaches and possible seizures, but the timing would be much different. Neither is associated with fever, as with this patient. Some medications, such as isotretinoin, could produce neurologic and psychiatric adverse effects ranging from psychosis, seizures, and stroke. If this patient had recently started a new oral medication for acne, medication side effects would not explain his fever and inflammatory response or the Negri bodies. If his father had Huntington disease, an autosomal dominant neurodegenerative disorder, this patient would be expected to have a much less acute presentation. While seizures and cognitive deficits are possible with Huntington disease, this patient's presentation is much more suggestive of encephalitis. If his mother consumed large amounts of alcohol throughout his fetal gestation, fetal alcohol syndrome may be suspected. Cognitive deficits and seizures are associated with this syndrome, but abnormalities would have been present since infancy and not with an acute onset.

A 48-year-old woman presents after a seizure. Prior to the seizure, she experienced confusion and disorientation preceded by nausea, vomiting, and blurred vision. Symptoms appeared after working for several hours in the garden under the sun. Her medical history is significant for the presence of schizophrenia, for which she takes chlorpromazine at bedtime. Her temperature is 41 C; BUN and creatinine are elevated; and there is neutrophilia, hemoconcentration, and lactic acidosis. You think that the event is possibly drug-related. Question What is the most likely diagnosis? Answer Choices 1 Heat cramps 2 Neuroleptic malignant syndrome 3 Heat stroke 4 Malignant hyperthermia 5 Heat exhaustion ANS:5

Heat stroke Heat disorders can be exertional and nonexertional. Both can be drug-related. Neuroleptics (e.g., phenothiazines, thioxanthenes) may impair thermoregulation due to both anticholinergic and antidopaminergics effects. Anticholinergics inhibit sweating, therefore disturbing thermoregulation during exercise or under conditions of environmental heat stress. Antidopaminergics elevate the set point of the temperature regulation center in hypothalamus. Your patient most probably suffered heat stroke. It is a life-threatening condition characterized by elevated body temperature with nausea, blurred vision, confusion, disorientation, and seizures. Hemoconcentration, anuria, rhabdomyolysis, kidney dysfunction, lactic acidosis, and even disseminated intravascular coagulation may result. Heat cramps are a mild disorder characterized by painful muscle contractions due to temporary fluids and electrolytes depletion. There are no signs and symptoms of neurological dysfunction, and body temperature is normal. Neuroleptic malignant syndrome is an idiosyncratic reaction to neuroleptics, most commonly phenothiazines and butyrophenones, and it is characterized by rigidity, fever, and autonomic instability. It is not connected with the exposure to heat and exertion. Men under 40 are at greatest risk. Malignant hyperthermia is a nonexertional idiosyncratic reaction to the anesthetic. Heat exhaustion is a condition with a severity that lies between heat cramps and heat stroke. Body temperature might be slightly elevated, and there may be neurological signs like headache, but there will be neither severe confusion nor seizures.

A 4-month-old infant presents for a well child check up. She was a spontaneous vaginal delivery at 39.5 weeks without complications. The mother notes the baby has been more irritable and has not been eating well. No cough or fever is described. There are no smokers at their home. On exam, you note impaired extraocular movements, especially in the upward gaze, and a bulging anterior fontanel. There is increased tone of the legs. Skin exam is normal. Similar to her last visit, the length and weight are at the 50th percentile. The head circumference was formerly at the 75th percentile and is now at the +99th percentile. What clinical diagnosis do you make for this child? Answer Choices 1 Positional plagiocephaly 2 Craniosynostosis 3 Hydrocephalus 4 Neurofibromatosis 5 Catch-up growth

Hydrocephalus The clinical diagnosis of hydrocephalus can be made by macrocephaly, impaired upward gaze, and hypertonia of the extremities. These signs would warrant further workup for definitive diagnosis. Craniosynostosis results when there is premature closure of the cranial sutures, usually resulting in elongation of the head. Positional plagiocephaly is occipital flattening, most often from infants lying on their back to sleep. The skin exam is normal in this baby, while neurofibromatosis will often display café au lait spots. Catch-up growth can be seen in babies born prematurely, which is not the case with this patient. Also, only the head percentile is changing, not the length and weight.

What is the most accurate statement concerning the treatment of migraine headache with sumatriptan? Answer Choices 1 Its effectiveness is related to its action as a serotonin antagonist 2 Its most common side effect is irritability 3 The recommended oral dose is 6 mg 4 It is indicated for both treatment and prophylaxis of migraine headache 5 If it is effective in the initial therapy, it is often effective in aborting a recurrence of symptoms

If it is effective in the initial therapy, it is often effective in aborting a recurrence of symptoms f it is effective in the initial therapy, it is often effective in aborting a recurrence of symptoms. Sumatriptan is a serotonin agonist; it is believed that its effectiveness is related to the importance of serotonin in the regulation of cranial and intracranial vasoactivity. The most common side effect is nausea with or without vomiting; however, these are frequent symptoms of a migraine headache itself. It may also cause sedation and drowsiness, but irritability is not common. The recommended oral dose is 25 mg, and the recommended subcutaneous dose is 6 mg. Sumatriptan is indicated for the treatment of a migraine headache, but it is not recommended for prophylaxis. Clinical trials have shown that sumatriptan is effective in reducing the severity of a migraine headache in approximately 70 - 80% of patients, but recurrence of the headache is seen in as many as 35 - 45%; a 2nd dose of sumatriptan is effective in treating the recurrence in the majority of patients.

A 42-year-old schoolteacher presents with a hand tremor; the tremor has been present for several years, but it is getting more bothersome. The patient would like treatment for her tremor. It appears to be present only in her right hand, and she tells you that it "comes and goes". It is especially embarrassing for her because she writes on the board in front of her students. She denies pain, motor weakness, and abnormal sensation in the right hand. She reports that her father's hand used to "shake" when he drank his coffee. She otherwise feels well, and she denies other symptoms. She has not noticed memory problems, incoordination, or balance problems. Her past medical history is unremarkable. She had 3 uncomplicated pregnancies and deliveries; she has no chronic medical conditions. Except for a mild tremor on the patient's right hand when held extended, the physical exam is unremarkable. Her neurological exam is otherwise normal. Question What is the most appropriate intervention for this patient's condition? Answer Choices 1 Advise the patient her condition is benign and no treatments are available 2 Initiate carbidopa/levodopa 3 Initiate methylphenidate 4 Initiate propranolol 5 Referral for physical therapy

Initiate Propranolol Based on her history and exam, this patient has a benign essential (familial) tremor, for which it is appropriate to initiate propranolol. Propranolol is a non-selective beta-blocker. This patient has no contraindications to the therapy; she is troubled by the worsening tremor, which interferes with her daily activities. Essential tremor is indeed a benign condition. When the tremor is mild and does not cause embarrassment or interfere with the patient's activities, it would be reasonable to avoid medications, but it is inappropriate to tell the patient that no treatments exist. Especially in this case, the patient desires treatment. Carbidopa/levodopa would be an appropriate medication for an individual with tremor due to Parkinson's disease; however, this patient has no other neurological complaints and no history to suggest she currently has Parkinson's disease. Methylphenidate is a stimulant; it is typically used for the treatment of attention deficit hyperactivity disorder. Not only would it be ineffective in this patient's case, it would also likely make her tremor worse. Physical therapy has not been proven to have a role in treating essential tremor. It would not be helpful in this case.

What condition is characterized by fine movements that are absent at rest, occur with activity, and worsen as the target is neared? Answer Choices 1 Postural tremor 2 Asterixis 3 Intention tremor 4 Chorea 5 Static tremor

Intention Tremor Intention tremors are fine movements of the hands that are absent at rest, occur with activity, and worsen as the target is neared. They occur with multiple sclerosis and cerebellar disease. Choreiform movements are brief, rapid, jerky, unpredictable motions of the hands. They occur both at rest and during normal actions. They are associated with both Huntington's and Sydenham choreas. Asterixis is non-rhythmic flapping movements of the hands, especially if the wrists are dorsiflexed. It is associated with liver failure, renal failure, and pulmonary insufficiency. Static, or resting, tremors are coarse movements of the hands that are present at rest, disappear with movement, and may involve alternation of the fingers in a "pill-rolling" manner. They are associated with Parkinson's syndrome and extrapyramidal dysfunction. Postural tremors are fine movements of the hands that occur when the hands are held in an active position, usually against gravity. Examples of postural tremors are those resulting from anxiety, fatigue, and hyperthyroidism, as well as familial postural tremors.

You are on duty in the ER when a young man is brought in by ambulance. The paramedics report that they were called by this man's roommate who reported the patient was confused and dazed, and obviously very ill. The patient is acutely febrile and delirious; there is evidence of meningeal involvement. Subsequent diagnosis reveals viral encephalitis. What is the most common cause of viral encephalitis worldwide? Answer Choices 1 La Crosse 2 Herpes simplex 3 Japanese B 4 Eastern equine 5 Western equine

Japanese B Viral encephalitis is secondary to viral infection of the brain tissue itself, causing widespread death of nerve cells and glia. Approximately 1,000-5,000 cases per year are reported in the U.S. Worldwide, Japanese B encephalitis is the most common cause of epidemic encephalitis. However, herpes simplex is the most common cause of sporadic encephalitis in the U.S. Arthropod-borne viral encephalitides are transmitted through mosquitoes (most often), and ticks and are named for the region of the US affected. These included LaCrosse, St. Louis, Venezuelan equine, Western equine, and Eastern equine encephalitides. LaCrosse, a type of California encephalitis, is the most common arthropod-borne encephalitis in the U.S. Eastern equine encephalitis (EEE) is a summertime disease that affects children in the east, south and on the Gulf Coast. The mortality is 50-75%, with 80% of patients having neurological deficits. Western equine encephalitis (WEE) occurs also in the summer months in states west of the Mississippi River. The mortality is 5-15%, with infants being the most susceptible. Venezuelan equine encephalitis (VEE) occurs in the southwestern U.S. and Central and South America and has very low mortality and morbidity rates. This infection has been linked to epidemics in tens of thousands of cases, however. St. Louis encephalitis occurs in the central, western and southern U.S. and affects older adults in the autumn months.

A 40-year-old man presents with drooping of eyelids and difficulty getting up from a sitting position. He also gives a history of impotence and frequent dryness of his mouth. On examination, he is found to have ptosis, depressed reflexes, and loss of power in the proximal muscle groups of the lower extremity. On electrodiagnostic testing, he is found to have incremental response on repetitive nerve stimulation. What is the likely diagnosis in this patient? Answer Choices 1 Neurasthenia 2 Botulism 3 Lambert Eaton syndrome 4 Progressive external ophthalmoplegia 5 Myasthenia gravis ANS:3

Lambert eaton syndrome The above history and findings are suggestive of Lambert-Eaton syndrome. Lambert-Eaton myasthenic syndrome (LEMS) is a presynaptic neuromuscular disorder that causes weakness of the muscles. LEMS is caused by antibodies directed against the calcium channel on the motor nerve terminal, leading to the decreased release of acetylcholine. 1/2 of the patients have an autoimmune form, the other half have a tumor-associated form (usually small-cell lung cancer) of the disease. Hence a thorough investigation has to be done to detect occult malignancy. Clinical features include: Weakness of proximal muscle groups initially (especially the lower limb) and later other muscles. Ptosis and diplopia. Depressed or absent reflexes. Autonomic changes such as dry mouth, impotence, etc. Worsening of symptoms in the morning and improvement with exercise, as opposed to myasthenia gravis Diagnostic data includes incremental response of the muscle groups on repetitive nerve stimulation, as opposed to decreased response in case of myasthenia gravis. Treatment includes plasmapheresis and immunosuppression. A significant number of patients have clinically benefited from 3, 4-diaminopyridine (DAP). Immunotherapy should be considered if severe weakness persists despite DAP. Plasma exchange and high-dose immunoglobulin induce a transient improvement in many patients, but function rarely becomes normal. Botulism is a paralytic disease involving the cranial nerves. It progressively involves the extremities in a rostrocaudal manner. It is caused by neurotoxin produced by the bacteria Clostridium botulinum (anaerobic gram positive organism) and is primarily associated with home canned foods. Cranial nerve involvement marks the onset of the disease with diplopia, dysarthria, dysphagia, and muscular weakness/descending paralysis. Ptosis is frequently seen. Diagnostic features include absence of fever, ptosis, depressed pupillary reflexes, suppressed gag reflex, and normal or depressed deep tendon reflexes. Paralytic ileus, severe constipation, and urinary retention may also be seen. Myasthenia gravis is a neuromuscular disorder characterized by weakness and fatigability of the skeletal muscles. The pathophysiological cause is the decrease in the number of acetylcholine receptors in the postsynaptic membrane. Due to the decreased receptors, the amount of acetylcholine bound to the postsynaptic receptors are decreased, leading to inadequate depolarization and ultimately resulting in weakness of the muscles. Clinical features include muscular weakness and generalized fatigue. The weakness increases on repetitive use of a group of muscles, and it reduces on rest. Neurasthenia usually presents as tiredness and weakness, and it mimics neuromuscular junction disorder. Muscle testing in these patients reveals a jerky release. The complaint of muscular weakness in these patients is due more to tiredness than any real decrease in the muscular power. Progressive external ophthalmoplegia is a rare condition usually involving the extraocular muscles and proximal muscle groups. This is usually caused by some mitochondrial disorder, and it is usually diagnosed by muscle biopsy.

A 64-year-old man presents with difficulty speaking, trouble swallowing, and right arm weakness. Physical examination reveals dysarthria, tongue deviation to the right, paralysis of the left soft palate, left facial droop, left facial sensory loss, and right arm weakness. These findings suggest involvement in what area? Answer Choices 1 Medulla 2 Left brain stem 3 Left cerebral hemisphere 4 Pons 5 Right midbrain

Left Brain Stem Although precise localization of neurologic deficits is often difficult, certain patterns are important to recognize. Brain stem involvement results in 'crossed findings' (e.g., right facial weakness and left arm weakness) because the lesions affect the brain stem nuclei directly (uncrossed), and the corticospinal tract as it is crossing to the opposite side of the body. Because of the unique anatomic arrangement of the brainstem, a unilateral lesion within the structure can cause "crossed findings" in which ipsilateral dysfunction of 1 or more cranial nerves is associated with hemiplegia and hemisensory loss on the contralateral side of the body, as described in this case. Choice B is the only option mentioning the brain stem; therefore, it is the correct choice. Medullary syndromes, such as lateral medullary syndrome, are associated with vertigo, nausea, vomiting, nystagmus as well as hiccups, and diplopia; they were not observed in this patient. Medial medullary syndrome is associated with tongue deviation toward the lesion, which is opposite to what is described in this case. Cerebral (cortical) lesions produce contralateral motor and sensory findings in the limbs and contralateral cranial nerve deficits (e.g., right arm and right facial paralysis). Pontine lesions result in coma, miosis, gaze paresis, and altered respiratory patterns. Cerebellar lesions produce nystagmus, dizziness, nausea and vomiting, and the inability to stand or walk if midline cerebellar areas are involved.

A 74-year-old woman presents for management of an ischemic stroke. She reports difficulty seeing objects on her right side. You perform confrontational visual field testing as part of your neurological examination, and you discover she has a right inferior homonymous quadrantanopsia. This lesion localizes to what part of the body? Answer Choices 1 Left optic tract 2 Optic chiasm 3 Left optic nerve 4 Left parietal lobe 5 Right optic tract ANS:4

Left parietal lobe Quadrantanopsia (visual loss in a quadrant of the visual field) may result from several causes, of which stroke is the most common. Accurate visual field testing can often localize the region of ischemia. An inferior homonymous quadrantanopsia localizes to the contralateral parietal lobe where the superior optic radiation fibers run. Other symptoms such as hemiplegia or hemisensory loss may also be seen. An optic tract lesion would cause a contralateral homonymous hemianopsia. An optic chiasmal lesion would cause a temporal non-homonymous hemianopsia. A left optic nerve lesion would cause unilateral, not bilateral visual field loss.

A 19-year-old woman presents to the emergency department complaining of headache. The headaches are generalized and increasing in intensity. They have not responded to over-the-counter (OTC) medications. She complains of approximately 1 week of blurred vision, intermittent diplopia, and vague dizziness. Her medical history includes obesity and acne. She takes Accutane and oral contraceptives. She is found to have bilateral papilledema, visual acuity of 20/30 on physical examination, and a normal MRI of the brain. The next most appropriate step would be ANS: LUMBAR PUNCTURE

Lumbar Puncture The presence of headache associated with papilledema raises the concern for a brain tumor. The MRI excluded a mass lesion, raising a strong suspicion of pseudotumor cerebri. This is also known as benign intracranial hypertension. It is not a benign condition, however, since it causes severe headache and may result in visual loss. It is particularly frequent in obese adolescent girls and young women. The etiology is unknown but may be associated with the use of oral contraceptives, vitamin A, and tetracycline. The presentation consists of headaches caused by an increase in intracranial pressure and blurring of vision. There may be diplopia, but the remainder of the neurologic examination is unremarkable. Papilledema is virtually always part of the presentation. The mental status is normal. The differential diagnosis includes venous sinus thrombosis, sarcoidosis, and tuberculosis or carcinomatous meningitis. The last two are excluded by lumbar puncture. An abnormal cerebrospinal fluid is not consistent with pseudotumor cerebri. The diagnosis is made by excluding mass lesions with CT scan or MRI and demonstrating markedly increased intracranial pressure by lumbar puncture. The treatment involves weight loss, diuretics, and steroids. Repeat lumbar punctures to remove cerebrospinal fluid and decrease intracranial pressure are effective. In cases that are unresponsive to these measures, lumbar-peritoneal shunting is effective, as is unilateral optic nerve sheath fenestration. Effective treatment can improve headaches and prevent vision loss.

A 32-year-old man presents with muscle weakness, lack of coordination, and tingling in his fingertips; he states that he has not been able to control his urinary function for the past few weeks. He also states that he has double vision sometimes. What is the most likely diagnosis? Answer Choices 1 Parkinson's disease 2 Multiple sclerosis 3 Multiple myeloma 4 Transient ischemic attack 5 Stroke

MS The clinical picture is suggestive of multiple sclerosis. The common clinical presentation is weakness, numbness, tingling, or unsteadiness in a limb, spastic paraparesis, retrobulbar neuritis, diplopia, disequilibrium, or a sphincter disturbance such as urinary urgency or hesitancy. Common findings in Parkinson's disease are tremor, rigidity, bradykinesia, and postural instability. Parkinson's disease is commonly seen in older patients. Multiple myeloma is a malignancy of plasma cells characterized by replacement of bone marrow, bone destruction, and paraprotein formation with a median age-of-onset of 65 years old. Most patients complain of lower back pain. Transient ischemic attacks (TIA) are characterized by focal ischemic cerebral neurological deficits that last for less than 24 hours. Symptoms vary from patient to patient, but they include abrupt or sudden onset; recovery usually occurs within minutes. Some common symptoms include weakness and heaviness in the contralateral arm, leg, or face. Strokes are similar to TIA, but symptoms last longer than 24 hours and most patients have a history of hypertension, diabetes mellitus, valvular heart disease, or atherosclerosis.

A 22-year-old woman presents with an 8-hour history of headache and vomiting. She resides in a college dormitory; she has no remarkable travel history, and reports eating food from a street vendor 24 hours before becoming ill. The patient is lethargic and disoriented. On examination, she is found to have a temperature of 38.5° C and a petechial rash on her legs. Gram stain of exudate from a skin lesion shows Gram-negative diplococci. A lumbar puncture is performed; the cerebral spinal fluid (CSF) reveals a leukocyte count of 5/μL consisting of primarily neutrophils. A Gram stain of CSF is negative for bacteria. What is the most likely diagnosis? Answer Choices 1 Rocky Mountain spotted fever 2 Neisseria gonorrhoeae bacteremia 3 Listeria monocytogenes meningitis 4 Meningococcemia 5 Viral meningitis ANS:4

Meningococcemia Meningococcemia may evolve rapidly, with death ensuing before the development of meningitis. The presence of Gram-negative diplococci and the clinical picture are consistent with a diagnosis of bacteremia caused by Neisseria meningitidis. While also a Gram-negative diplococcus, N. gonorrhoeae is an unlikely cause of this clinical presentation. Listeria monocytogenes, transmitted by contaminated food, is also a common cause of meningitis. This organism is not typically associated with a petechial rash; it appears as a Gram-positive coccobacillus in the CSF, where mononuclear cells predominate. Rocky-Mountain spotted fever is caused by the Gram-negative obligate intracellular pathogen Rickettsia rickettsii, which is not detectable by Gram stain. Infection with R. rickettsii results in a macropapular rash. Viral meningitis is usually accompanied by monocytes in the CSF, although neutrophils may appear early in the disease.

A 16-year-old boy presents with acute onset of stiff neck, fever, headache, and vomiting. On exam, he appears lethargic, has limited range of motion of his neck, and a petechial rash is noted. Fever is 103 °F. The patient's medical history is non-contributory. What is the most likely etiology of the suspected diagnosis? Answer Choices 1 Enterovirus 2 Varicella virus 3 Syphilis 4 Pneumococcus 5 Meningococcus

Meningococcus Meningitis is an inflammation of the membranes covering the brain and spinal cord. The most common causes of meningitis are bacterial infections that start in other parts of the body and spread to the brain or spinal cord via the bloodstream. Meningitis can be caused by bacteria, viruses, chemical irritation, or tumors. Symptoms include fever, severe headache, nausea and vomiting, stiff neck (nuchal rigidity), photophobia, and mental status changes. Complications include hearing loss, brain damage, loss of vision, and possible deafness. Meningococcal meningitis is a life-threatening infection caused by the Neisseria meningitidis bacterium. Most cases occur in children and young adults. The onset is often abrupt, and early treatment is essential to achieving reduced morbidity and mortality. In addition to the usual symptoms, a petechial rash (non-blanching, red-brown spots on all parts of the body) is characteristic and caused by capillary leak and platelet dysfunction. Routine vaccination against meningococcus is recommended for adolescents in the United States; the first dose at age 11 or 12, and a booster dose at age 16. If this patient has had not routine medical care in many years, then he will not have been vaccinated against this disease. Enterovirus is a term given to cases of meningitis caused by viruses. Enteroviruses, such as the Coxsackie and echovirus, account for the majority of cases of viral meningitis. Other viruses that can cause meningitis include mumps, varicella, and the human immunodeficiency virus. Viral meningitis is usually milder than that caused by bacteria; symptom onset is more gradual, and patients usually recover with conservative management. Syphilitic meningitis is a progressive, life-threatening complication of untreated syphilis infection. In addition to the usual symptoms of meningitis, patients may present with prominent mental status changes and focal neurologic deficits. Serum VDRL or RPR along with CSF (cerebrospinal fluid) examination is diagnostic. Pneumococcal meningitis is caused by the bacteria Streptococcus pneumoniae. Meningitis occurs as a consequence of bloodstream infection with subsequent invasion of the central nervous system. Patients often present with a history of recent pneumonia, upper respiratory infection, infective endocarditis, or ear infection.

A 28-year-old woman presents with a 6-hour history of severe pulsatile hemicranial headache. She had numbness of the right upper limbs and lower limbs prior to the onset of the headache. The numbness lasted for 45 minutes. The headache worsens with exertion. She had a similar episode of headache, with nausea and vomiting, 2 weeks prior to presentation. She does not have a history of head injury. She has been married for 2 months, and she is on oral contraceptives. She gives a history of febrile convulsions in her childhood, and she has a sister with epilepsy. On examination, her blood pressure is 150/84 mm Hg, and neurological examination is normal. Her serum cholesterol level is 384 mg/dl. Question What is the most likely diagnosis? Answer Choices 1 Sensory epilepsy 2 Migraine 3 Cluster headache 4 Thromboembolic transient ischemic attack 5 Hypertension

Migraine The correct answer is migraine. The patient presents with a headache which is hemicranial and pulsatile, which are characteristics commonly seen with migraines. In addition, the association of nausea/vomiting and thepresence of a precipitating factor (i.e., being on oral contraceptives) increases the likelihood of it being a migraine. The numbness lasted for less than 60 minutes, which is probably an aura of sensory symptoms. Sensory epilepsy is an incorrect response. The numbness lasted 45 minutes and disappeared; however, in cases of sensory epilepsy, there is usually a distinct progression of symptoms. Cluster headache is an incorrect response. Cluster headaches are usually accompanied by at least 1 of the following ipsilateral autonomic symptoms: conjunctival injection or lacrimation, nasal congestion or rhinorrhea, eyelid edema, forehead and facial sweating, miosis or ptosis, and restlessness or agitation. Thromboembolic transient ischemic attack is an incorrect response. In cases of transient ischemic attacks, the maximum deficit is present immediately, and headache is unusual. Hypertension is an incorrect response. Systolic blood pressure of 150 mm Hg may probably be due to pain (headache). Mild hypertension {(140 - 159)/ (90 - 99) mm Hg} is usually asymptomatic.

A 45-year-old woman presents with a 1-month history of double vision, tripping, urinary incontinence, and restless legs; the symptoms are accompagnied by weakness, numbness, and tingling sensations. She denies any headaches, difficulty with speech, or memory impairment. Her physical examination does not demonstrate impairments in mental status, speech, or memory. Her muscular strength is 2+ bilaterally in the upper and lower extremities, although it is diminished from a previous exam 1 year ago. Deep tendon reflexes are also diminished compared to 1 year ago. What is the most likely diagnosis? Answer Choices 1 Multiple sclerosis 2 Vitamin E deficiency 3 Vitamin B12 deficiency 4 Friedreich's ataxia 5 Charcot-Marie-Tooth Disease ANS:1

Multiple sclerosis Multiple sclerosis is correct because this patient demonstrates most of the distinguishing characteristics; spasticity (what she describes as restless legs), diplopia, unsteadiness, numbness, tingling, and urinary urgency/hesitancy (sphincter abnormalities). Vitamin E deficiency is incorrect because its main features are ataxia, slurred speech, sensory loss, and absent deep tendon reflexes. This patient doesn't report or exhibit speech impairment. Vitamin B12 deficiency is incorrect because its main features are polyneuropathy, mental status changes, and optic neuropathy. These patients do not exhibit or report mental status issues. Friederich's ataxia is incorrect because its main features are clumsy hands, ataxia, pes cavus, and sensory disturbances. Charcot-Marie-Tooth Disease is incorrect because its main features are polyneuropathy, distal weakness, impaired deep tendon reflexes, and sensory loss.

A 5-year-old boy presents with an altered gait. His mother reports that he first walked independently at 15 months of age, but that for the past 3 months his gait has become waddling and he has been taking a less active part in physical activities with his friends. He has even begun to fall on occasion. Past medical history is unremarkable. The mother recalled that a maternal uncle died in his late teens of an unknown condition. Physical examination reveals normal vital signs and growth parameters. Gait is waddling. Running is awkward, and hopping on either foot can only be performed with great difficulty. Strength of the deltoids and hip flexors was 3/5, with normal strength in the distal muscle groups. The calves appear enlarged bilaterally. He needs to climb up upon himself to arise from a sitting position on the floor. Question What test is most likely to yield a specific diagnosis? Answer Choices 1 Aldolase level 2 Creatinine kinase level 3 Electromyography 4 Muscle biopsy 5 Nerve conduction velocities

Muscle Biopsy Male gender, age of onset, gait alteration, proximal weakness, pseudohypertrophy of the calves, and positive Gower sign (needing to climb up upon himself), as well as the family history, strongly suggest Duchenne muscular dystrophy, which is an X-linked recessive condition caused by any of a number of mutations of the dystrophin gene. The condition is progressively debilitating. In some cases, it is associated with mental retardation and myocardial disease. Muscle biopsy will usually show muscle necrosis with replacement of muscle tissue by fat and fibrous tissue (contributing to the pseudohypertrophy of the calves). An elevated aldolase level is nonspecific for any number of muscle disorders. Similarly, an elevated creatinine kinase is also nonspecific for any number of muscle disorders. Electromyography will help to distinguish between neuropathy and myopathy, but it rarely provides an exact diagnosis. Nerve conduction velocities will help distinguish between neuropathy and myopathy, but they will not provide a specific diagnosis.

A 65-year-old woman presents with general weakness. She had the weakness for the past few days, and it gets worse as the day progresses. She says she has been having double vision, and you notice she has poor posture. A blood test reveals presence of antibodies to acetylcholine receptor. A repetitive nerve stimulation test showed a decremental response. What is the most likely diagnosis? Answer Choices 1 Marfan's syndrome 2 Myasthenia gravis 3 Duchenne's muscular dystrophy 4 Rheumatoid arthritis 5 Systemic lupus erythematosus

Myasthenia Gravis Myasthenia gravis, a disorder more common in women, is characterized by chronic weakness of voluntary muscles. This weakness improves with rest and worsens with activity. The body produces an immune response (autoantibodies) against the nicotinic acetylcholine receptors at the neuromuscular junction. The most common presentation is muscle weakness that progressively worsens as the day progresses. The clinical features include ocular manifestations such as ptosis and diplopia, extra-ocular manifestations including limb and trunk weakness, dysphagia, generalized weakness, depressed gag reflex, limp body, slackening of the jaw, and respiratory depression. The diagnostic tests are anti-ACh receptor antibody assay, repetitive nerve stimulation (shows a decremental response), and electromyography. Treatment consists of the administration of acetylcholinesterase inhibitors, immunosuppressive agents like corticosteroids, cyclosporine, azathioprine, plasmapheresis, and intravenous gamma globulin. Thymectomy can also be done. Marfan syndrome occurs due to mutation in the fibrillin-1 gene, located on chromosome 15. The cardinal features occur in 3 systems--skeletal, ocular, and cardiovascular. Skeletal manifestations include an increase in height, disproportionately long limbs and digits, joint laxity, anterior chest deformity, vertebral column deformity such as scoliosis and thoracic lordosis, and high-arched palate with crowding of the teeth. Ocular manifestations include myopia and subluxation of the lenses (ectopia lentis). Cardiovascular features are mitral regurgitation, mitral valve prolapse, dilatation of the aortic root, and aortic regurgitation. Molecular studies of fibrillin-1 gene and imaging studies should be carried out in patients suspected of Marfan syndrome. Duchenne muscular dystrophy is an X-linked recessive disorder. It is characterized by delay in gross motor milestones, symmetrical proximal progressive muscle weakness, pseudohypertrophy of calf muscles, and lordotic posture. Positive Gower sign and waddling gait are diagnostic of Duchenne muscular dystrophy. Confirmation of diagnosis is by elevated levels of creatine kinase, DNA testing, and muscle biopsy. Rheumatoid arthritis is a chronic inflammatory systemic disease that primarily involves the articular surfaces and synovial membranes of multiple joints. It is characterized by the presence of rheumatoid nodules and rheumatoid factor. Assay for rheumatoid factor and imaging studies should be carried out in patients suspected of having rheumatoid arthritis. Systemic lupus erythematosus (SLE) is a chronic, inflammatory, multi-system disorder. SLE is an autoimmune disorder associated with the production of antinuclear antibodies (ANA). The manifestations include polyarthralgias, polyarthritis, malar rash, discoid lupus erythematosus, alopecia, aphthous stomatitis, anemia, and lupus nephritis. The clinical presentation and the investigations are indicative of myasthenia gravis and not Marfan syndrome, Duchenne muscular dystrophy, rheumatoid arthritis, or systemic lupus erythematosus.

A 64-year-old woman presents for evaluation of myxedema. She is experiencing constipation, headaches, weakness, fatigue, lethargy, somnolence, cold intolerance, decreased sweating, paresthesias, arthralgias, and muscle cramps. She has dry, coarse skin. Her face and extremities are swollen, and her voice is coarse. Question What gait abnormality would you most likely expect to see in this patient? Answer Choices 1 Astasia-abasia 2 Ataxic 3 Myopathic 4 Steppage 5 Limping 6 Magnetic ANS:3

Myopathic Myxedema is the accumulation of hydrophilic mucopolysaccharides in subcutaneous tissues in severe hypothyroidism. Neuromuscular symptoms are present in 30-80% of patients with hypothyroidism, and they usually improve or disappear with correction of the hypothyroidism. The most common complaints are of muscle cramping, proximal symmetrical muscle weakness, muscle stiffness, and exercise intolerance. These manifestations can occur at any time during the presentation of hypothyroidism. Proximal muscle weakness presents as dystrophic or circumduction gait. These patients usually complain of difficulty climbing stairs because they are actually using the handrails or their own hands to pull themselves up with their arms. Thyroid myopathy is also characterized by slowness of both muscle relaxation and muscle contraction. This is due to the decreased amount of myosin ATPase, leading to the slowing of release and reaccumulation of calcium in the endoplasmic reticulum. In thyroid disease, segmental demyelination of peripheral nerves with decreased nerve conduction velocities may also be observed; there may also be a decrease in vibration, joint-position, and touch-pressure sensations. Astasia-abasia is seen most commonly in the very late stages of normotensive hydrocephalus. Patients may demonstrate the inability to walk, inability to stand, sit, rise from a chair, or turn over in bed; this advanced stage is called "hydrocephalic astasia-abasia." Ataxic gait is a wide-based gait with difficulty or inability to tandem walk due to the midline cerebellar diseases. Patients sway from side to side as they walk. Typically, patients who have midline cerebellar degeneration have difficulty with tandem gait, but they have normal coordination in their arms. Unilateral cerebellar lesions often cause incoordination of the arm and leg on the same side as the lesion. A cerebellar abnormality is implicated in ataxia only if weakness, spasticity, and position-sense abnormality are absent. A steppage gait occurs when the patient has either a foot drop from weakness of the foot dorsiflexors or poor position sense in the feet. With a foot drop, the patient flexes the hip so that the foot will clear the floor. Patients with poor position sense have a wide-based gait; their feet slap the floor; and they require visual assistance to walk. Limp is a gait abnormality usually due to pain ("antalgic gait") or skeletal deformities, e.g., hip deformity with or without associated muscular weakness ("Trendelenburg gait"), with the body shifted over the affect hip. In magnetic gait (when feet seem attached to floor as if by a magnet), each step is initiated in a "wresting" motion carrying feet upward and forward. Magnetic gait and gait disturbances are typically the initial and most prominent symptom of the triad of normotensive hydrocephalus (dementia, incontinentia, and gait disturbance) and may be progressive due to expansion of the ventricular system with a progressive traction of motor fibers in pyramidal tract. The gait disturbance in normotensive hydrocephalus can be classified as mild (unsteadiness, impaired balance, especially when encountering stairs and curbs, usually described as cautious gait), marked (evident difficulty walking or considerable unstable gait when the patient needs mobility aids to provide some stability), or severe (unaided gait not possible, as there is a constant tendency to fall backwards; gait is broad based in order to increase the base of support and provide some stability; and steps height is decreased, as well as foot-floor clearance and the speed of walk). This style may remind one of Parkinsonian gait, except there is no rigidity or tremor.

A 40-year-old woman presents with a 7-day history of pain in her right arm. The patient denies any trauma or injury to this extremity just prior to the pain starting, but she does admit to having a Colles' fracture in this arm about 2 months ago. She denies any injury to her back, neck, or other components of the musculoskeletal system prior to the event of pain. She describes the pain burning and throbbing with an extremely diffuse, uncomfortable ache accompanying it. She further states that this limb has become extremely sensitive to touch and cold; she adds that it appears somewhat more swollen than her left arm. The patient is very upset because she does not know why her arm is so painful when she has not done anything to it. She is a nonsmoker. She does not drink alcohol, and she exercises 3 times a week. Physical examination of the extremity reveals a slightly cyanotic hue and generalized pain of the entire right upper extremity. Pulses are strong (2+), but due to the pain, the patient states ROM is limited. Question Considering the most likely diagnosis, what is the initial pharmaceutical regimen that should be initiated? Answer Choices 1 Acetaminophen 500 mg 2 tablets BID 2 Acyclovir 800 mg TID 3 Naprosyn 500 mg 2 tablets BID 4 Cephalexin 500 mg BID

Naprosyn The correct response is Naprosyn 500 mg 2 tablets BID. The patient being described is most likely in stage I of complex regional pain syndrome (CRPS). This disorder is rare, causing instability of the vasomotor and autonomic components. CRPS is a disorder of a body region, most commonly the extremities, that is characterized by pain, swelling, limited range of motion, vasomotor instability, skin changes, and even patchy bone demineralization. Frequently symptoms will begin after the patient has sustained an injury, had surgery, or experienced a vascular event, such as a stroke. 35% of patients with CRPS report not having a distinct precipitating event. Stage 1 of CRPS characteristically has findings such as pain in the limb (burning/throbbing), diffuse, uncomfortable aching, sensitivity to touch or cold, or even localizededema. The distribution of pain is not compatible with a single peripheral nerve, trunk, or root lesion. There will also be signs of variable vasomotor disturbances that result in altered color and temperature. At this stage if a radiograph was completed, most likely it will be normal but could show signs of patchy demineralization. Early treatment gives the best prognosis for this patient. The accepted first line treatment would be NSAIDs (Naprosyn 250 - 500 mg 2 times daily). More severe cases that have significant edema (our patient above states there is swelling, but pulses are strong on PE) will be started on daily prednisone, 30 - 60 mg/day for a 4 week taper. Other options that may be considered for treatment include (but are not limited to) gabapentin, topical capsaicin, topical dimethylsulfoxide, IV bisphosphonates, IV immunoglobulin, and intrathecal baclofen. Acetaminophen, acyclovir, colchicine, and cephalexin are not indicated for treatment of CRPS.

A 19-year-old man presents for evaluation of excessive daytime sleepiness. Despite getting a full night's sleep, his coworkers have repeatedly noticed him suddenly dozing off at his desk and during meetings; sometimes, he falls asleep mid-conversation. Upon further questioning, the patient admits that he occasionally experiences nighttime episodes during which he has the sensation of being unable to speak or move. The remainder of the history and physical exam are unremarkable. Aside from the occasional ibuprofen for shoulder pain, he does not take any medications. Question The patient's symptoms are most consistent with what disorder? Answer Choices 1 Central sleep apnea 2 Insomnia 3 Medication side effect 4 Narcolepsy 5 Obstructive sleep apnea ANS:4

Narcolepsy Based on the symptoms described, the patient is most likely to have narcolepsy. Narcolepsy is marked by 4 major symptoms: 1) excessive daytime sleepiness; 2) cataplexy; 3) sleep-related hallucinations;and 4) sleep paralysis. It is a central sleep disorder that is attributed to abnormal neurotransmission in the brain's sleep centers. Daytime sleepiness can be so severe that it results in involuntary somnolence during activities such as driving or talking. Cataplexy describes the brief loss of muscle tone that can occur sporadically during waking hours. Patients also may suffer from visual, auditory, or tactile hallucinations at the onset of or upon awakening from sleep. In addition, they may experience inability to move on falling to or emerging from sleep (sleep paralysis). Obstructive sleep apnea is defined as partial or complete upper airway obstruction caused by anatomic collapse of pharyngeal structures. Patency of the upper airway is partially (hypopnea) or completely (apnea) lost during sleep. This results in the development of progressively increasing inspiratory effort against an obstructed glottic opening, leading to the build-up of increasingly negative intrathoracic pressure. At some point, brain stem reflexes result in an altered sleep state that prompts reopening of the airway. This may awaken the patient or be observed as a "gasping" respiratory effort by an onlooker. Snoring is indicative of obstructed airflow, and daytime somnolence is a function of disrupted sleep. Central sleep apnea is an uncommon neurological condition that involves impairment of a patient's normal spontaneous cue to breathe based on changes in blood oxygen and carbon dioxide levels. The symptoms are similar to those of obstructive sleep apnea. Insomnia is a generic condition marked by difficulty falling asleep or inability to remain sleeping for extended periods of time. It is a symptom, not a diagnosis in and of itself. Sleep derangement is a common medication side effect; it is especially observed with psychotropic medications. There is no reason to suspect medication-related sleep impairment in this patient because he reports no medication use.

A 27-year-old woman presents for the evaluation of hypersomnia. Her symptoms began approximately 5 years prior to presentation. The hypersomnia persisted despite the fact that she was sleeping 6 to 8 hours per night during the week and up to 10 hours on the weekends. She has a difficult time maintaining wakefulness while at work. It has been difficult developing a social life due to the fact that she often goes to bed early on weekends to catch up on her sleep. She often has vivid dreams and frequent hypnagogic hallucinations. She has sleep paralysis, which is long standing; it typically occurs 1 or 2 times a month. She has had episodes of cataplexy, with several episodes within a 1-week period. An episode would occur if she got excited or stressed. Her neck muscles would weaken, and she would have an irresistible urge to sleep. During these episodes, she was able to hear everything that was going on around her, but she could not respond. The episodes typically lasted a few minutes. She denies a history of head trauma or depression. She undergoes an overnight sleep study that demonstrates an apnea-hypopnea, and she slept for 7.4 hours. Question What is the most likely cause of the patient's symptoms? Answer Choices 1 Narcolepsy 2 Absence seizures 3 Simple partial seizures 4 Frontal lobe epilepsy 5 Delayed sleep-phase syndrome

Narcolepsy Narcolepsy is characterized by daytime sleepiness, cataplexy, hallucinations, and sleep paralysis. It is a chronic sleep disorder caused by the brain's inability to regulate sleep-wake cycles normally. Narcolepsy is thought to result from genetic predisposition, abnormal neurotransmitter functioning and sensitivity, and abnormal immune modulation. Human leukocyte antigen subtypes and abnormalities in monoamine synaptic transmission are thought to be involved. The cause of narcolepsy remains unknown. Men and women are affected equally, and children as young as 2 years old may also be affected. The age of onset distribution is biphasic; the highest peak occurs at the age of 15, and a smaller peak occurs at around age 36. Dysfunction and inappropriate regulation of rapid eye movement (REM) sleep are thought to cause narcolepsy. At various times throughout the day, people with narcolepsy feel strong urges to sleep. Patients may fall asleep anywhere from a few seconds to several minutes. In rare cases, some people may remain asleep for an hour or longer. Patients suffer from extreme daytime sleepiness, sudden loss of voluntary muscle tone (cataplexy), vivid hallucinations during sleep onset or upon awakening, and brief periods of total paralysis at the beginning or end of sleep. The condition may not be diagnosed until 10 to 15 years after the first symptoms appear. Absence seizures are brief episodes of staring. Awareness and responsiveness are impaired. Patients usually do not realize when they've had one. There is no warning before a seizure, and the person is alert immediately afterward. Simple partial seizures are seizures that usually affect the temporal lobes and/or hippocampi. Simple partial seizures are usually followed by larger seizures later on in life; the abnormal electrical activity spreads to a larger area of the brain, usually resulting in a complex partial seizure or a tonic-clonic seizure Frontal lobe epilepsy is either simple partial or complex partial, mostly with secondary generalization, and it is characterized by recurrent seizures. The clinical presentation ranges from postural, behavioral, tonic, to motor manifestations. Status epilepticus is usually associated with frontal lobe seizures. Delayed sleep-phase syndrome (DSPS), which is also known as delayed sleep-phase disorder or delayed sleep-phase type, is a circadian rhythm sleep disorder, a chronic disorder of the timing of sleep, peak period of alertness, hormonal, and other rhythms. People with DSPS tend to fall asleep well after midnight, and they also have difficulty waking up in the morning.

A 34-year-old man presents with a right-sided periorbital headache of 20 minutes duration. The pain is described as excruciating and bore-like, and it is associated with tearing and redness of the right eye, right eyelid swelling, and nasal congestion. The headaches have been occurring at multiple times nearly daily over the past 2 weeks, and each episode lasts approximately 30 to 60 minutes. The pain is not relieved with acetaminophen or ibuprofen. There is no nausea, vomiting, or photophobia. There is no history of amaurosis, visual scintillations, diplopia, focal weakness, numbness, neck stiffness, or other common neurologic complaints. The patient has had similar bouts of headaches over the past 5 years; the bouts typically last 3 to 4 weeks. The patient has an otherwise unremarkable medical history. General exam is remarkable for right eye conjunctival injection. The neurologic exam is significant for a mild right ptosis; the right pupil is 2 mm and reactive to light; the left pupil is 4 mm and reactive to light. There are no other neurologic findings. Question What is the most effective abortive therapy for this patient's headache? Answer Choices 1 Aspirin 2 Meperidine 3 Naproxen 4 Nasal oxygen 5 Oral sumatriptan

Nasal O2 This patient has cluster headaches (CH). CH is a relatively rare primary headache disorder characterized by recurrent periorbital or temporal headaches; they are associated with ipsilateral conjunctival injection, lacrimation, rhinorrhea, eyelid edema, and oculosympathetic signs (ptosis and miosis). The pain of CH is severe, typically characterized as "excruciating" in nature. CH occurs in cycles (the cluster period), usually lasting several weeks to months. The headaches are relatively brief in duration (15 to 180 minutes), but they can occur multiple times daily (up to 8) during the cluster period. Unlike other primary headaches, CH is more common in men. CH is classified as episodic or chronic, depending upon the duration and frequency of the cluster period. In episodic CH, the cluster period lasts from 7 days to 1 year; cycles are separated by pain-free remissions of at least 2 weeks. In chronic CH, the cluster period is longer than 1 year; remissions last less than 2 weeks. Abortive therapies for acute CH include nasal oxygen (5 to 10 L/minute), sumatriptan (subcutaneous), and dihydroergotamine (DHE, subcutaneous or intramuscular). Oral analgesics (acetaminophen, aspirin, ibuprofen, narcotics) are much less effective in aborting acute CH. A brief course of corticosteroids is often effective in shortening the CH cycle. Prophylactic agents in CH include lithium, methysergide, verapamil, and valproic acid.

A 75-year-old African-American man has a past medical history that is significant for severe and uncontrolled hypertension; he is brought into the emergency room by his family due to a 30-minute history of change of mental status. The patient had just climbed stairs when he first developed a headache that has become progressive; it is associated with nausea, non-bilious vomiting, and unilateral upper and lower extremity numbness. His physical exam reveals an alert and oriented times zero patient. He is afebrile, with a blood pressure of 185/108 mm Hg, and there is nuchal rigidity. His neurological exam notes contralateral sensory loss, contralateral hemiparesis, gaze paresis, homonymous hemianopia, and miosis. A CT scan without contrast was performed, and it is shown in the image. Question What medication would be most appropriate in this patient? Answer Choices 1 Heparin 2 Dobutamine 3 Nicardipine 4 Pyrimethamine 5 Methotrexate ANS:3

Nicardipine The correct response is nicardipine. This patient is experiencing an intracerebral hemorrhage, with underlying hypertension as a major contributory etiologies. Antihypertensive agents reduce blood pressure to prevent exacerbation of intracerebral hemorrhage. Pressure should be cautiously lowered to a mean arterial pressure (MAP) less than 130 mm Hg. Excessive hypotension should be avoided. Early treatment in patients presenting with spontaneous intracerebral hemorrhage is important as it may decrease hematoma enlargement and lead to better neurologic outcome. Nicardipine is a calcium channel blocker with a potent rapid onset of action, ease of titration, and lack of toxic metabolites. It (as is Labetalol) is an agent that reduces blood pressure in order to prevent exacerbation of intracerebral hemorrhage. Patients with intracerebral bleeding should not be given blood-thinning agents. Instead, vitamin K and protamine may be used to restore normal coagulation parameters. Furthermore, recombinant factor VIIa (rFVIIa) within 4 hours after the onset of intracerebral hemorrhage limits the growth of the hematoma, reduces mortality, and improves functional outcomes at 90 days. Dobutamine should be avoided; it is a sympathomimetic agent. It has beta and alpha-1 adrenergic effects in the body, causing tachycardia and vasoconstriction. Pyrimethamine is the most effective agent in the treatment of toxoplasmosis, and it is included in most drug regimens. Methotrexate is the single most effective chemotherapeutic agent for primary central nervous system lymphoma.

A 74-year-old man presents after his wife witnessed him grab his head in pain and fall to the floor. He has not regained consciousness. His current blood pressure is 150/96 mm Hg, and his heart rate is 65 bpm. Emergent head CT shows a subarachnoid hemorrhage. Question In addition to life saving interventions, what prescription medication will most benefit this patient at this time? Answer Choices 1 Furosemide (loop diuretic) 2 Prednisone (glucocorticoid) 3 Tranexamic acid (antifibrinolytic agent) 4 Labetolol (beta blocker) 5 Nimodipine (calcium channel blocker) ANS: 5

Nimodipine (CCB) Nimodipine, a calcium channel blocker, has been shown to improve outcomes in patients following aneurysmal SAH. The mechanism of action is thought to be prevention of ischemia. Glucocorticoids (e.g., prednisone) are often utilized in patients with SAH because they offer symptomatic relief of headache and neck pain, but they have not been proved to decrease cerebral edema. Antifibrinolytic agents (e.g., Tranexamic acid) can be utilized in patients with a diagnosed aneurysm who cannot undergo directed treatment, but they are not routinely used following aneurysmal rupture. Labetalol (a beta blocker) may be utilized to treat elevated BP, but it must be used with caution because it can also decrease cerebral perfusion. Diuretics (e.g., furosemide) have no identified role in the treatment of aneurysmal SAH.

A 23-year-old man presents after being thrown from a motorcycle onto a cement median. He is awake and appears alert on exam. He believes that his helmet was knocked off at point of impact, but he cannot recall the specifics of his crash. A head CT scan is consistent with a cerebral contusion. What findings on a CT scan would you see with this diagnosis? Answer Choices 1 A peripheral ring of enhancement around areas of hematoma 2 Biconvex shaped hematoma over the cerebral hemisphere 3 No cortical disruption, but tissue necrosis, edema, and punctate hemorrhages 4 No cortical disruption or necrosis, but multiple small isolated hemorrhages throughout the cortex, with edema 5 Enlarged lateral ventricles, third and fourth ventricles, and basal cistern; normal or absent sulci

No cortical disruption or necrosis, but multiple small isolated hemorrhages throughout the cortex, with edema The correct response is no cortical disruption or necrosis, but multiple, small, isolated hemorrhages throughout the cortex, with edema. A cerebral contusion is a focal lesion to the brain, secondary to trauma. Contusions usually occur over the crest of gyri, largest at the surface of the brain and tapering in size into the white matter. Cerebral contusions can occur under the site of impact, or on the contralateral side of the brain (a contrecoup lesion). The CT scan of a cerebral contusion characteristically reveals punctate hemorrhages and edema under the site of impact. A cerebral contusion is defined as "bruising" of the brain without interruption of the cortex. Frequently, a subarachnoid hemorrhage overlies the area of contusion. A biconvex-shaped hematoma over the cerebral hemisphere is indicative of epidural hematoma. A peripheral ring of enhancement around areas of a hematoma is seen 1 to 6 weeks after a traumatic intracerebral hematoma, not in cerebral contusion. Panventricular enlargement is a feature of hydrocephalus.

A 72-year-old man presents with the inability to comprehend spoken language. His speech is fluent and has a normal cadence and rhythm; however, when he talks, it is gibberish. He frequently substitutes one word for another. What is this phenomenon called? Answer Choices 1 Paraphasia 2 Echolalia 3 Alexia 4 Apraxia 5 Agraphia ANS:1

Paraphasia Paraphasia is a type of aphasia. The substitution of a similar sounding word for another word is called paraphasia. With paraphasia, the words can also be jumbled. If a patient is able to hear things and repeat them, it is called echolalia. With echolalia, the patient does not understand what he has heard. This is also referred to as echophrasia. Alexia is a type of aphasia. An aphasia in which there is a problem with reading is called alexia. Alexia is word blindness or text blindness. Alexia is also called optical aphasia or visual aphasia. Apraxia refers to the condition in which a patient has difficulty performing motor acts, despite having the muscular capacity and coordination to do so. A patient with apraxia cannot execute the intended movement. A writing disturbance is called agraphia. There are various forms of agraphia. With absolute agraphia, even simple letters cannot be written. This is also referred to as literal agraphia.

A colleague asks you to provide a neurosurgical consultation on a 75-year-old man. The consultation request form informs you that, "This man is having difficulty tolerating his medications as well as difficulty with his day-to-day activities". Your colleague wants to know if neurosurgery could help the patient's neurologic disorder. As a prudent provider, you plan to visit this man prior to recommending or denying definitive surgery. For what disorder may neurosurgery become a reasonable therapeutic option? Answer Choices 1 Alzheimer's 2 Parkinson's 3 Amyotrophic lateral sclerosis 4 Down's syndrome 5 Multiple sclerosis ANS:2

Parkinson's As per Svennilson E, Torvik A, Lowe R, and Leskell L, "Treatment of Parkinsonism by Stereotactic Thermo-lesions in the Pallidal Region," Acta Psychiatry Neurology Scandinavia, 35:358-377, 1960, neurosurgery was provided for patients afflicted with Parkinson's disorder (PD) 50 years ago. Providers now know that dopamine replacement, carbidopa-levodopa (Sinemet) for example, is not a cure for PD. Eskandar, EN, Cosgrove GR, and Shinobu LA, "Surgical Treatment of Parkinson's Disease," The Journal of the American Medical Association, Volume 286, Number 24, pages 3056 - 3059, December 26, 2001, endorse that thalamotomy and pallidotomy can improve tremor and rigidity in selected patients with PD, sparing patients the risk of the adverse effects of the dopamine receptors. Neurosurgery has not been reasonably considered for patients with Alzheimer's disorder, ametropic lateral sclerosis, multiple sclerosis, or Down's syndrome, all of which represent a diffuse involvement of the brain and central nervous system in contrast to PD, which represents a focal involvement of the brain and central nervous system. In addition, the reduced intellectual functioning commonly associated with Down's syndrome would prevent achieving scholastic and academic rank.

A 60-year-old woman presents for evaluation of loss of recent and remote memory and has psychomotor slowing with difficulty in complex tasks. On examination, the patient also shows some tremors in the hands and face. She also presents with a shuffling gait. Family history reveals 2 of her brothers had a movement disorder; they are now deceased. What would be your provisional diagnosis? Answer Choices 1 Delirium 2 Dementia of the Alzheimer's type 3 Vascular dementia 4 Parkinson's disease 5 Substance induced dementia

Parkinson's Dementia due to medical conditions include loss of memory and other cognitive deficits due to diseases such as Huntington's disease, Parkinson's, Pick's disease, multiple sclerosis, HIV disease, vitamin B-12 deficiencies, and hypothyroidism. In addition, head traumas and tumors can cause dementia, and all physical causes have to be ruled out first. Vascular dementia presents with the same signs and symptoms as other dementias, but the patient usually has a strong history of cerebrovascular disease, multiple infarcts, or history of arteriosclerotic disease. Vascular dementia usually has a sudden acute onset after the infarcts. Vascular dementia may also include behavioral disturbances in the patient. In patients with a delirium, there are usually rapid fluctuations in cognitive functioning, brief durations of normalcy, and disturbances in the sleep-wake cycle. In addition, deliriums come on acutely and rapidly. It is somewhat common in older patients who have had heart surgery and in those who have experienced a change of environments, such as a hospital intensive unit. In addition, falls have been known to bring on a delirium. When an older adult has a fall, it is very important to perform a CT scan of the head to rule out concussions, head trauma, hematomas, and other injuries. Clients with substance-induced dementia have cognitive deficits related to a drug or alcohol use or abuse and/or toxicity of prescription drugs; the toxicity can be accidental or intentional. This appears to be more common in the elderly than once thought. Many elderly take prescription drugs incorrectly or mix them with contraindicated over-the-counter drugs. Also, patients may suffer side effects and not report them to their physicians. Dementia of the Alzheimer's type involves gradual loss of memory and other cognitive functioning, which results in the inability to accomplish activities of daily living. Before a diagnosis is made, all other physiological causes should be ruled out. This type of dementia may also present itself as uncomplicated, with depressed mood, with delirium, or with behavioral disturbance.

A 6-year-old girl presents because her teacher is concerned about behaviors at school. The teacher has noticed the girl "staring off into space" frequently throughout the day, and the teacher is able to get her attention only occasionally. There are also periods when she appears to be talking to herself, but there is no sound coming from her mouth. The girl's mother states that there are times when the patient does not seem to be paying attention to what the mother is saying. Shortly after these "episodes", the child engages in conversation without any problem; therefore, the mother did not think the episodes were an issue. There is no concern about other abnormal behavior or discipline issues at home or at school. Question What is the most likely diagnosis? Answer Choices 1 Attention-deficit disorder (ADD) 2 Syncopal episodes 3 Petit mal (absence seizures) 4 Narcolepsy 5 Childhood disintegration disorder (CDD) ANS:3

Petit mal (absence) seizures The patient is probably experiencing petit mal (absence) seizures. Absence seizures are a form of generalized seizures seen in children, and they usually cease by age 20. They are characterized by abrupt onset of impaired consciousness and can be associated with enuresis or automatisms. Patients often appear as though they are "staring off into space" during the seizure, and the episodes often terminate as quickly as they came. These children are often categorized as "being in their own world", leading to a delay in diagnosis or a misdiagnosis of attention deficit disorder. These types of seizures are diagnosed using an electroencephalogram (EEG), showing crusts of bilaterally synchronous and symmetric 3-Hz spike-and-wave activity. Attention-deficit disorder (ADD) is a form of Attention-deficit hyperactivity disorder (ADHD) that is characterized more by inattentiveness without the presence of restless or impulsive behavior. This is the most common emotional, cognitive, and behavioral disorder treated in children/adolescents. The diagnosis is made by careful clinical history, revealing a significant level of inattentiveness and distractibility, plus-or-minus impulsivity, and hyperactivity that is inappropriate for the developmental stage of the child. These patients often change activities frequently, have a hard time with organizational skills, and are commonly caught daydreaming. The symptoms of ADD are usually pervasive, even though they may not occur in all settings. ADD/ADHD also does not involve any impairment of consciousness. Syncopal episodes are better known as "fainting spells." These are episodes of diminished or complete loss of consciousness accompanied by flaccidity that usually occur in relation to postural change, emotional stress, instrumentation, pain, straining, cardiac arrhythmias, or other physiologic disturbance. Associated signs/symptoms include pallor, sweating, nausea, and malaise. While this could be in the differential diagnosis for our patient, the cause of the syncopal episode would still need to be determined by conducting a battery of tests. In addition, our patient is not noted to lose postural tone during her episodes; this can distinguish her presentation from true syncope. Narcolepsy is a condition that is associated with "sleep attacks" that are uncontrollable and occur in inappropriate and occasionally dangerous situations. Most of these attacks are brief; however, this is not always true. Associated symptoms can include cataplexy (a sudden loss of muscle tone), hypnagogic hallucinations (dream-like experiences while falling asleep), and sleep paralysis. Childhood disintegration disorder (CDD) is a rare syndrome on the autism spectrum. It is characterized by normal development until the age of 2 that is followed by a major loss of skills, both social and verbal. Motor skills are often preserved. The presentation is often very abrupt and very dramatic. This differs greatly from the child presented above; she has had a more gradual onset of symptoms that have almost gone "unnoticed".

A 16-year-old previously healthy boy fell on a cemented surface while skateboarding 3 weeks ago, and he lost consciousness for 2 minutes. He was not wearing a helmet. A CT scan showed no abnormalities. He is now back at school and complains of difficulty concentrating in class, frequent spells of dizziness, and headaches. His neurologic exam is normal. Question What condition should you consider as the likely explanation for his symptoms? Answer Choices 1 Traumatic Brain Injury 2 Dysthymia 3 Complex partial seizures 4 Post Concussion Syndrome 5 Subdural hemorrhage ANS:4

Post concussion syndrome This adolescent displays the characteristic features of post-concussion syndrome, which is usually diagnosed on the basis of presence of 3 out of the following 8 features after a significant injury event: 1) headache; 2) dizziness; 3) fatigue; 4) irritability; 5) insomnia; 6) concentration or 7) memory dif?culty; and 8) intolerance of stress, emotion, or alcohol. It is unlikely to be a traumatic brain injury because he had a short duration of loss of consciousness, normal head CT, and his current neurologic exam is normal. Symptoms of dysthymia or depression can seem to overlap with those of PCS, but he had no such symptoms prior to the traumatic event. Complex partial seizures are unlikely; they are usually associated with focal seizure activity in the form of tonic/tonic-clonic movements. Subdural hemorrhage is usually evident on head CT, but delayed subdural hemorrhages are rarely seen. In those situations, they are associated with abnormalities on neurologic examination.

A 48-year-old man presents with a 5-year history of hand tremor. The tremor was initially mild and has progressed over the last year. It diminishes at rest, but it intensifies with intentional movements and upon emotional stress and fatigue. He is unable to write or drink from a cup during stressful situations. He never drinks alcohol, and he does not currently take any drug except metformin for non-insulin dependent diabetes mellitus. Family history is negative. Physical exam is unremarkable. His gait, speech, and posture are normal. Question What is the best first-line agent for treatment of this condition? Answer Choices 1 Nimodipine 2 Alprazolam 3 Amitriptylline 4 Primidone 5 Theophylline

Primodone Essential tremor is the most common idiopathic movement disorder and may occur in at least 14% of adults (Schwartz and Andrasik, 2003). The disease is more common in men than in women, with a mean age of onset of 45 years and 60% positive family history (Schwartz and Andrasik, 2003). According to the consensus statement of the Movement Disorder Society (MDS, 1998), the core criteria for diagnosing essential tremor include the presence of a visible, bilateral postural tremor (predominantly symmetric, with or without kinetic tremor or head tremor) of the hands or forearms that does not feature abnormal posturing and/or other neurological signs (Deuschl et al., 1998). The tremor may have duration greater than 3 years, and it may diminish with alcohol consumption (Deuschl et al., 1998; Chen and Swope, 2003). Primidone (and propranolol) are the best initial agents for treatment of essential tremor (Louis, 2001). Relative contraindications for the use of propranolol are asthma, congestive heart failure, diabetes mellitus, and atrioventricular block (Louis, 2001).

A 21-year-old woman presents with double vision that occurs only when she looks to her right side. The double vision began when she woke up in the morning. She had an episode of left leg numbness while at summer camp 6 years ago, but it resolved over 3 days and she never told anyone. Also, 3 years ago she saw her private physician after a 7-day episode of right eye pain and visual blurring. He attributed her symptoms to an ocular migraine. There is no history of head trauma. She hasn't had any infections, fevers, or immunizations recently. She is awake, alert, and in no acute distress. Positive findings include mild pallor and atrophy of the right optic disc. Bedside visual fields and acuity are normal. Testing external ocular motion in both eyes together reveals that there is no left eye movement beyond midline when attempting to look to her right, accompanied by right eye lateral nystagmus. When the left eye is tested with the right eye closed, eye movements are full. No other motor signs are found. No skin, sensory, or hearing findings are found. A magnetic resonance imaging scan (MRI) of the head with gadolinium enhancement reveals a 2 x 3 cm lucency in the region of the right parietal white matter without swelling or enhancement. Multiple sclerosis (MS) is suspected. Question Given this history, how would this patient's condition best be subtyped? Answer Choices 1 Relapsing Remitting MS (RRMS) 2 Secondary Progressive MS (SPMS) 3 Primary Progressive MS (PPMS) 4 Progressive Relapsing MS (PRMS) 5 There is insufficient data at this time to subclassify ANS:1

Relapsing remitting MS The patient has had 3 discrete clinical exacerbations (right parietal white matter/left leg numbness, right optic nerve/ visual blurring, eye pain with residual disc pallor, and median longitudinal fasciculus/resolved diplopia), with full clinical resolution. It should be kept in mind that this is a clinical descriptive and disability rating. Therefore, residual abnormalities on exam and ancillary lab testing do not define the subtype. The patient is best subtyped at this time as Relapsing Remitting MS. Subtyping may change over time with new clinical events. This patient might later develop persistent clinical disability (e.g., severe ataxia, persistent urinary incontinence, extremity weakness, diplopia, etc.). She would then best be reclassified as Secondary Progressive MS, whether these occurred as residua of discrete relapses or as an independent progressive deterioration independent of relapses. Unlike the classic Relapsing/Remitting presentation of MS, some patients present with a progression of neurological disability with no clear relapses, per se. These are subtyped as Primary Progressive MS. Some patients may initially rate a Primary Progressive MS rating, yet later develop episodes of superimposed relapses and remissions. These patients are then reclassified as Progressive Relapsing MS.

A 46-year-old man presents with severe insomnia and anxiety. While hospitalized, an overnight polysomnogram was performed over 2 consecutive nights. Sleep latency was 60 minutes; REM latency was 45 minutes. He reports feeling paresthesias deep within his legs while lying in bed, especially while falling asleep. He denies recent illness or illicit drug use. His physical exam and lab work were within normal limits. Question What is the most likely cause of the patient's symptoms? Answer Choices 1 Deep vein thrombosis 2 Peripheral neuropathy 3 Restless legs syndrome 4 Alcoholic peripheral neuropathy 5 Periodic limb movements in sleep ANS:3

Restless leg syndrome Restless legs syndrome (RLS) is a neurological disorder with symptoms of an unpleasant sensations in the legs, such as insects crawling inside the legs, burning, tugging, or creeping. There is an uncontrollable urge to move the limb when at rest (lying down and trying to relax activates the symptoms). Most people with RLS have difficulty falling asleep and staying asleep. Women may be slightly more affected than men. Symptoms may begin at any stage of life, although the disorder is more common with increasing age. The severity of the disorder appears to increase with age. In some patients, symptoms will improve over a period of weeks or months. In most cases, the cause of RLS is idiopathic. A family history of the condition is seen in many cases, suggesting a genetic component. People with familial RLS tend to be younger when symptoms start, and they have a slower progression of the condition. Hypoglycemia can worsen the condition. The disorder is diagnosed clinically by evaluating the patient's history and symptoms. Needle electromyography and nerve conduction studies should be considered if polyneuropathy is suspected on clinical grounds, even if results of neurologic examination are apparently normal. Ropinirole is approved for the treatment of moderate-to-severe RLS. Benzodiazepines may be prescribed for patients who have mild or intermittent symptoms. These drugs help patients obtain a more restful sleep, but they do not fully alleviate the symptoms and can cause daytime drowsiness. For more severe symptoms, opioids may be prescribed since they can cause relaxation and decrease pain. Anticonvulsants, such as carbamazepine and gabapentin, are also useful for some patients; they can decrease the creeping and crawling sensations. There may be no symptoms associated with deep vein thrombosis (DVT), but the classical symptoms of DVT include pain, swelling, redness of the leg, and dilation of the surface veins. Peripheral neuropathy, of which alcoholic peripheral neuropathy is a type, causes tingling or burning pain in the feet. At times, it may be so severe that it interferes with walking, which is a result of injury to sensory fibers. As the condition worsens, the pain typically decreases and numbness increases. Periodic limb movements in sleep are associated with periodic episodes of highly repetitive limb movements during sleep. These repetitive episodes of muscle contractions are usually grouped into series.

A 42-year-old man presents to the emergency department with a severe headache. He has been getting several of these headaches recently and has tried all over-the-counter pain relievers and headache medicines with no relief. His current headache started 15 minutes ago. He describes the pain as located next to and behind his left eye and "stabbing/excruciating" in nature. He feels like his left eye tears up profusely with these headaches. He reports he has been healthy otherwise, with no chronic medical conditions, no history of surgery, no medications, and no drug allergies. He denies recent stressors that may have caused his headaches. On physical exam, the patient appears slightly agitated and appears uncomfortable. His left eye's conjunctiva is mildly injected, and lacrimation is noted. His right eye is normal. Cranial nerves II-VII are intact, although the patient expresses discomfort when the light is shown in his left eye. Speech, gait, coordination, and reflexes are all normal. The remainder of his exam is normal. Head MRI is performed and reported as normal. Question Which of the following aspects of patient history would be most consistent with this patient's suspected condition? Answer Choices 1 Head injury prior to onset of headaches 2 Periodic episodes of aphasia and gait disturbances 3 Recent exposure to outdoor pollens 4 Rhinorrhea associated with headaches 5 2 to 3-day duration of each headache ANS:4

Rhinorrhea associated with HA This patient is suffering from cluster headaches. Cluster headaches are classified as trigeminal autonomic cephalalgias and are most common in men. Cluster headaches may be triggered by alcohol, histamine, and nitroglycerin, but often no cause is identified. There may be a genetic component. Classic presentation of cluster headache includes episodic, unilateral, severe headache, usually located in the temporal and/or orbital region, along with associated lacrimation, injection, eyelid edema, miosis or ptosis of the ipsilateral eye, and nasal congestion and rhinorrhea associated with the headaches. Imaging studies, such as head MRI, should be performed to rule out other primary causes of headache, such as vascular malformation, neoplasm, and infection. A head injury prior to the onset of headaches is not associated with cluster headaches. Post-concussion headaches tend to have longer duration, but less severe intensity compared to cluster headaches. Other clues to a post-concussion headache may be memory problems, "fogginess," and nausea, and/or vomiting. The eye symptoms and signs this patient displayed are not consistent with a prior head injury. Periodic episodes of aphasia and gait disturbances would be concerning for a severe disorder, ranging from vascular, infectious, neurodegenerative, and neoplastic in etiology. Patients with cluster headaches would not be expected to have these types of neurologic symptoms. Recent exposure to outdoor pollens would be expected in a patient experiencing allergy-related headaches. A patient with allergy-related headaches would be expected to have much less intense severity and bilateral eye and nasal symptoms. 2 to 3-day duration of each headache can be seen with tension-type headaches and migraines. However, cluster headaches are noted to last 15-180 minutes of duration.

A 61-year-old man with hypertension presents with sudden onset of right eye blindness while shaving this morning. He describes a "curtain falling over my right eye," which cleared spontaneously after 10 minutes. He had no other concurrent neurologic symptoms and no prior episodes of similar description. His general examination reveals a blood pressure of 140/90 mmHg, but it is otherwise unremarkable. Neurologic exam is normal. Question What is the most likely diagnosis? Answer Choices 1 Right internal carotid artery transient ischemic attack (TIA) 2 Basilar artery TIA 3 Left posterior cerebral TIA 4 Left internal carotid artery TIA

Right ICA TIA The diagnosis in this patient is amaurosis fugax, or transient monocular blindness. This disorder is most often due to ipsilateral internal carotid artery (ICA) atherosclerosis, leading to a transient ischemic attack (TIA) of the retina. A TIA is defined as a focal neurologic deficit (e.g. amaurosis fugax, hemiplegia, aphasia) lasting less than 24 hours, which is referable to the distribution of the carotid or vertebral-basilar circulation. The vascular supply of the retina is through the ophthalmic artery, which is an intracranial branch of the ICA. Thus, this patient had a TIA of right ICA origin. Visual disturbances of the basilar or posterior cerebral circulation include diplopia, bilateral (binocular) blindness, and hemianopia. Atheroembolism from the ICA to the ipsilateral ophthalmic artery is the most likely mechanism of amaurosis fugax. Other causes of TIA include hemodynamic (low perfusion) impairment, arteritis, thrombocytosis, polycythemia vera, and cardiogenic embolus. Rarely, a cerebral mass lesion (neoplasm, abscess, vascular malformation) produces syndromes that masquerade as TIA's. Management of patients with carotid TIA is directed at identifying an accountable vascular (or cardiac) lesion, since the annual risk of ipsilateral stroke is approximately 15%. Appropriate workup for a patient with TIA includes: brain CT or MRI to visualize silent infarcts or mass lesions; carotid duplex, magnetic resonance or CT angiography, and conventional angiography to determine the presence of carotid stenosis; and cardiac evaluation (ECG, echocardiography, Holter monitor) to exclude a cardiogenic source of embolism. Medical therapy for stroke prevention includes antiplatelet agents (aspirin, ticlopidine) for patients with primary carotid disease with less than 70% stenosis; anticoagulation has been shown to be effective for cardiogenic stroke prevention (e.g. atrial fibrillation). Surgical treatment, i.e. carotid endarterectomy, is superior to medical therapy in patients with 70% or greater stenosis of the carotid artery. The latter was confirmed by the North American Symptomatic Carotid Endarterectomy Trial (NASCET) in 1991. The benefits of carotid endarterectomy decline in patients with significant comorbidity (i.e. cancer, coronary artery disease), and this procedure should be performed by experienced surgeons. The treatment of asymptomatic carotid stenosis remains controversial.

A patient presents with bilateral spastic paresis of his legs. His gait is noticeably stiff and his legs are advanced slowly. He uses short steps and crosses his legs in front of the other as he advances. What type of gait does he have? Answer Choices 1 Scissors gait 2 Steppage gait 3 Sensory ataxia gait 4 Cerebellar ataxia gait 5 Myopathic gait

Scissors Gait Scissors gait is apparent when each leg is advanced slowly and the thighs tend to cross each other. The gait is stiff and the steps are short. This is associated with bilateral spastic paresis of the legs. Steppage gait is associated with a foot drop, often secondary to lower neuron disease. The patient drags his foot, or lifts the knee high and slaps the foot against the ground. This gait may be unilateral or bilateral. Sensory ataxia gait is unsteady and wide-based. The feet are thrown forward, out, and then down. There is a distinct 2-slap sound, secondary to the heel striking first, then the forefoot. This is associated with polyneuropathy or posterior column damage. The gait characterized by cerebellar ataxia is associated with a disease of the cerebellum or affiliated tracts. The gait is staggering, unsteady, and wide-based. The patient has difficulty with turns, and cannot stand steady when feet are together and eyes closed (Romberg's sign). A myopathic gait is where there is weakness on one side. This displays a a drop in the pelvic area on the contralateral side of the pelvis while walking (Trendelenburg sign). When there is a bilateral weakness, it will display as a dropping of the pelvic girdle on both sides of the body while walking that results in a waddle. This is seen in patients whom are afflicted with myopathies, such as muscular dystrophy.

You have determined that a multiple sclerosis patient with relapsing-remitting disease has not returned to her baseline functional status with her most recent exacerbations, despite her returning to baseline with exacerbations for many years. Her exam also reveals residual neurologic dysfunction. Which of the following multiple sclerosis clinical types would you classify this patient within? A Clinically isolated syndrome B Primary progressive C Progressive/relapsing D Relapsing/remitting E Secondary progressive ANS: E

Secondary progressive This patient would be classified within the secondary progressive clinical type. Secondary progressive MS follows a relapsing/remitting course initially, during which the patient returns to baseline neurologically and functionally between exacerbations, then evolves into a course of the patient having deterioration in function and neurologic status, which is not associated with acute exacerbations. The patient is also noted to not return to baseline following acute exacerbations. Primary progressive MS is associated with a decline in functional and neurologic status from the onset of disease, and disability is common. Progressive/relapsing MS is associated with progressive neurologic and functional decline from the onset of disease, but with associated exacerbations during the clinical course as well. The designation of clinically isolated syndrome indicates that a patient has had a single clinical event suggestive of MS but does not meet MS diagnostic criteria, thus requiring a waiting period and additional studies.

A 47-year-old man presents with numbness, tingling, and pain anterior and to the left side of his thigh. Pain is provoked even with light touch, and tingling continues for several minutes after the touch. Symptoms started a couple of months ago and worsen when he wears a belt and walks down slopes and stairs, as well as after prolonged standing. Symptoms are relieved when he puts a pillow between his thighs. Patient also assumes hunched posture while standing to avoid unpleasant sensory symptoms. His BMI is 40, and he has a protruding, pendulous abdomen. Question What else do you expect to find on examination? Answer Choices 1 Sensory loss in anterolateral thigh down to left upper knee 2Sensory loss on the dorsum of the foot 3 Foot drop 4 Loss of foot inversion in plantar flexion 5 Loss of m. quadriceps reflex

Sensory loss in anterolateral thigh down to left upper knee Your patient most probably has entrapment syndrome caused by the compression of lateral femoral cutaneous nerve and suffers from the condition called meralgia paresthetica. Pendulous abdomen and tight belt compress the inguinal ligament downward and onto the nerve. Lateral femoral cutaneous nerve is pure sensory nerve conducting sensory signals from the region of anterolateral thigh down to the upper knee. On examination, you can find decreased sensitivity to pinprick together with a hyperpathic reaction to touch. This angulation of the nerve is further exaggerated with extension of the thigh and relaxed with flexion. Foot drop is a sign of common peroneal nerve entrapment. Loss of foot inversion in plantar flexion is a sign of L5 radiculopathy, in which posterior tibialis function is disturbed. The quadriceps, patellar reflex, or knee-jerk is a deep tendon reflex that can be lost in the case of the lesion of L2, L3, and L4 root. This myotatic reflex tests motor function. Your patient has no signs and symptoms of radiculopathy or motor function disturbance.

A 9-year-old boy presents to a neurologist's office accompanied by his mother. She reports that he has had 3 episodes of what she thinks are seizures. She describes these episodes in detail to the neurologist. Question What would lead the neurologist to consider the diagnosis of tonic-clonic seizures? Answer Choices 1 Sudden loss of consciousness with convulsions that do not cease without medical attention and recur on a semi-regular basis 2 Sudden loss of consciousness with convulsions and confusion that lasts about 15 minutes each time, then resolves spontaneously 3 Sudden episodes of staring into space lasting 2-4 seconds each time and occurring 3-6 times per day 4 Alternating jerking of each of his extremities while maintaining consciousness in episodes that last several minutes 5 Episodes of blank staring that last a few minutes, during which he walks the same pattern in the room and does not remember it afterwards ANS:2

Sudden loss of consciousness with convulsions and confusion that lasts about 15 minutes each time, then resolves spontaneously The correct answer is the type of seizure that involves a sudden loss of consciousness with convulsions and confusion that lasts about 15 minutes each time, then resolves spontaneously. These types of seizures are also known as generalized convulsive seizures, or tonic-clonic seizures. They are associated with loss of consciousness, convulsions, and muscle rigidity lasting minutes to hours. Sudden loss of consciousness with convulsions that do not cease without medical attention and recur regularly is not the correct answer because this description is closest to status epilepticus given the recurrent and severe nature. Epilepsy is a condition during which a person has recurrent seizures that typically need medical attention. Any seizure that lasts longer than 5 minutes should be treated medically. Episodes of starting into space that last only a few seconds occur in patients with absence seizures. Absence seizures are a type of generalized seizure known as a generalized non-convulsive seizure. Tonic-clonic seizures involve loss of consciousness and some sort of convulsion, so this type of seizure is not a tonic-clonic seizure. Alternating jerking of each of his extremities while maintaining consciousness in episodes that last several minutes is not the correct answer, as this description closest describes simple partial seizures. Patients with simple partial seizures do not experience loss of consciousness. 'Episodes of blank staring that last a few minutes, during which he walks the same pattern in the room and does not remember it afterwards,' is not the correct answer, as this describes a complex partial seizure. This type of seizure includes impairment of awareness and some form of automatism, such as lip smacking, chewing, walking, fidgeting, or another involuntary, repetitive coordinated movement.

A 35-year-old woman is being treated for an acute migraine headache. She received an injection approximately 20 minutes ago; she is now experiencing significant chest pain. What medication do you suspect the patient received? Answer Choices 1 Promethazine 2 Metoclopramide 3 Ketorolac 4 Sumatriptan 5 Trimethobenzamide

Sumatriptan The clinical picture is suggestive of a side effect of sumatriptan. Sumatriptan is commonly used for treatment of acute migraine attacks. This drug can cause coronary vasospasms in 1 - 5% of patients; this is perceived as chest pain or discomfort. Promethazine has sedative and antiemetic properties; it does not cause chest discomfort as a side effect. Metoclopramide is commonly used to treat nausea and vomiting, but it does not cause chest discomfort as a side effect. Ketorolac is an NSAID and does not cause chest discomfort. Trimethobenzamide is an antiemetic used for the treatment of nausea and vomiting; it does not cause chest discomfort as a side effect.

A 45-year-old African-American man with no significant past medical history presents with a 1-hour history of left retroorbital headache. The headache was of a sudden onset and began upon waking that morning. It is described as excruciating, stabbing, sharp, and lancinating; it is rated as severe in intensity. He denies any preceding infections, nausea, vomiting, photophobia, or osmophobia; he also denies fever, chills, stiff neck, focal weakness, numbness, tingling, vision, hearing, gait, or speech changes. He recalls a similar episode several months ago; it lasted about a week, and it dissipated without complications. His physical exam is remarkable for painful distress, lacrimation with conjunctival injection, nasal congestion, rhinorrhea, left ocular miosis, and left forehead diaphoretic flushing. Question What pharmacologic agent is the most beneficial for this patient at this time? Answer Choices 1 Sumatriptan 2 Verapamil 3 Lithium 4 Topiramate 5 Prednisone ANS: 1

Sumatriptan The correct response is sumatriptan. This patient's most likely diagnosis is most likely a cluster headache. Pharmacologic management of cluster headache may be divided into abortive/symptomatic and preventive/prophylactic strategies. Abortive agents are used to stop or reduce the severity of an acute attack, and include oxygen, triptans, ergot alkaloids, and anesthetics. Inhalation of high-flow concentrated oxygen is extremely effective for aborting attacks. 5-Hydroxytryptamine-1 (5-HT1) receptor agonists, such as triptans or ergot alkaloids with metoclopramide, are often the first line of treatment. Stimulation of 5-HT1 receptors produces a direct vasoconstrictive effect and may abort the attack. Subcutaneous injection of sumatriptan can be effective, in large part because of the rapidity of onset. Studies have indicated that intranasal administration is more effective than placebo but not as effective as injections. Prophylactic agents are used to reduce the frequency and intensity of individual headache exacerbations. Preventive and prophylactic medications include calcium channel blockers, mood stabilizers, and anticonvulsants. Verpamil is the most effective calcium channel blocker for prophylaxis. It inhibits calcium ions from entering slow channels, select voltage-sensitive areas, or vascular smooth muscle, thereby producing vasodilation and preventing the initial vasoconstrictive phase of cluster headaches. It can be combined with ergotamine or lithium. Preliminary evidence suggests that prophylactic lithium may interfere with substance P and vasoactive intestinal peptide (VIP)-induced arterial relaxation. Anticonvulsants such as Divalproex and Topiramate are preventative medications whose mechanism of action may involve regulation of central sensitization. Prednisone is very effective for aborting the cluster headache cycle or providing intermediate prophylaxis as bridging therapy between acute and prophylactic agents. It is effective for treatment that does not respond to lithium.

A 20-year-old female college student presents to the student health center with a 1-week history of daily headaches. She has no significant past medical history. Upon further questioning, she admits to headaches occasionally over the past 2 years, but it is lasting longer this time. She classifies the pain as a 4/10 that is generalized, but it is worse in the back of the head. The headaches are not debilitating or throbbing, and she describes it as an annoying pain. Acetaminophen helps some, but does not make the headache go away completely. On physical exam, there are no neurologic deficits, vision is 20/20 uncorrected, and vital signs are within normal limits. She does complain of tenderness upon palpation of the muscles of the head, neck, and shoulders. Question What is the most likely diagnosis for this patient? Answer Choices 1 Cluster headache 2 Giant cell arteritis 3 Intracranial mass 4 Migraine headache 5 Tension headache

Tension HA Tension Headache is the most common type of primary headache disorder. Common complaints include: pericranial tenderness, poor concentration, and daily headaches (often vice-like or tight). These can be exacerbated by emotional stress, fatigue, noise, or glare. Cluster headaches affects mostly middle-aged men and typically present with unilateral periorbital pain. Migraine headaches usually present with a lateralized throbbing pain that occurs episodically following its onset in adolescence or early adult life. These headaches can be associated with nausea, vomiting, anorexia, phonophobia, photophobia, and blurred vision. Giant cell arteritis affects mostly elderly patients and can cause headaches that are preceded by myalgia, malaise, anorexia, and weight loss. Loss of vision is a common manifestation. Intracranial masses can also cause headaches that can range from mild to severe. Often these types of headaches are accompanied by neurologic deficits.

A 37-year-old Hispanic man presents with a 4-month history of mild-to-moderate headaches; on average, he gets them 3 - 4 days per week. He has tried over-the-counter analgesics with minimal relief. He is seeking care now because he had been promoted to store manager several months prior to presentation, and he is worried that his headaches are affecting his concentration. His headaches are generalized in location, described as starting at the base of his head and extending all over, feeling "tight" in nature. He denies memory loss, photophobia, nausea/vomiting, rhinorrhea, lacrimation, and upper respiratory symptoms associated with the headaches. He also denies seizures, syncope, incoordination, vertigo, weakness, and paresthesias. The patient mentions his concerns for his work several times. Although he enjoys his work, he admits to having some anxiety about being able to handle his new duties. His family history is negative for headache. Physical exam is performed; vitals, neurological, cardiovascular, and HEENT findings are all normal. Question What is the most likely diagnosis? Answer Choices 1 Cluster headache 2 Headache secondary to brain tumor 3 Migraine headache 4 Subarachnoid hemorrhage 5 Tension headache

Tension HA This patient presents with a most likely diagnosis of tension headache. Tension headaches, the most common type of headache, present as rather diffuse/generalized head pain, without the classic characteristics of the other types of headaches. A component of psychological stress is often present. Cluster headaches are a severe type of headache, most often occurring in middle-aged men, in which the severe headache is accompanied by several other symptoms, such as lacrimation, nasal rhinorrhea, and congestion. The pain is usually localized in the periorbital region. About 1/3 of patients with a brain tumor will present with a headache; however, tension-type headaches are far more common in the general population. Headaches secondary to brain tumor can vary in character and intensity. New onset headaches later in life, especially those without a coinciding significant psychosocial stressor, are of greater concern; they have a larger possibility of being caused by an intracranial mass. Nausea and vomiting or neurologic symptoms may accompany headaches caused by brain tumor. Migraine headaches are also quite common. They are classically described as throbbing in nature and are accompanied by nausea (and possibly vomiting), photophobia (sensitivity to light), and phonophobia (sensitivity to sounds)' visual changes and aura are also possible. Migraines are more common in women than men, but they can occur in men. They are usually a chronic condition, beginning in adolescence or the early adult years. Stress can be among many triggers for migraines. This patient's pain pattern and age of onset are inconsistent with migraine, and he lacks the associated symptoms. Subarachnoid hemorrhage (SAH) is classically described by patients as an acute headache that is the "worst headache of my life." There may be preceding trauma and associated nausea and vomiting; nuchal rigidity (due to meningeal irritation) and mental status changes are also seen. This patient's headache history is inconsistent with SAH.

What anticonvulsant is most effective for primary generalized seizures? Answer Choices 1 Phenytoin 2 Phenobarbital 3 Valproic acid 4 Gabapentin 5 Carbamazepine

The correct response is valproic acid. A seizure is defined as a discrete neurologic disorder in which there is excessive and synchronous discharge of cortical neurons. Epilepsy is a syndrome characterized by recurrent, unprovoked seizures. A seizure may be classified as generalized or partial. In a generalized seizure, there is bilateral cerebral hemisphere disturbance; in partial seizures, the abnormal neuronal discharge is limited to a specific brain area (e.g., right anterior temporal lobe). Generalized seizures may be primary (bilateral brain involvement at seizure onset) or secondary generalized (seizure starts in a focal brain area, then spreads to encompasses both cerebral hemispheres). Valproic acid is usually the first-line medication for primary generalized seizures. Phenytoin, phenobarbital, carbamazepine, and gabapentin have efficacy in partial and secondary generalized seizures. Phenytoin and phenobarbital have a role in the treatment of primary generalized seizures, but valproic acid is superior.

Headaches may be considered primary in origin (migraine, tension-type, and cluster) or due to underlying metabolic or organic disease. Primary headaches are not associated with any structural disease, but it is believed they are caused by biochemical dysfunction. What statement correctly describes cluster headaches? Answer Choices 1 Patients usually have a positive family history for cluster headaches 2 The headache lasts an average of 30 minutes to 3 hours 3 Cluster headaches occur approximately with equal frequency in males and females 4 Cluster headache pain is typically a moderate, bilateral, steady pain 5 Menstruation is an exacerbating factor for cluster headaches

The headache lasts an average of 30 minutes to 3 hours Cluster and migraine headaches are attributable to cerebrovascular imbalances and are both referred to as vascular headaches. The cerebrovascular dysfunction in cluster headaches is likely different from that of migraine headaches. Cluster headaches usually last from 30 minutes to 3 hours; they occur in groups of 1 to 3 daily over a period of several weeks to a few months. Headache-free intervals range from several months to several years. Clusters occur more often in the spring. The pain is described as excruciating and as seeming to bore from one eye back into the head. Accompanying characteristics include ipsilateral tearing, conjunctival congestion, nasal stuffiness, and a partial Horner's syndrome. Aggravating factors are mostly unknown, but alcohol and sleep may be triggers. Physical activity is known to ameliorate cluster headaches. There is rarely a positive family history and cluster headaches occur more frequently in males. Migraine headaches occur more often in females. Alcohol, chocolate, menstruation, weather changes, and physical activity are considered triggering factors. Migraine headaches usually have moderate to severe pain described as a unilateral throbbing, and they can last from several hours to several days. Associated symptoms include nausea, photophobia, phonophobia and, less commonly, aura. Most patients have a family history positive for migraines

A 42-year-old man presents following an episode of loss of consciousness. The patient was in excellent health until the evening of admission; his wife found him unconscious on the concrete deck surrounding their swimming pool. His swim trunks were fully wet and there was vomitus on the deck near him. He regained consciousness in the ambulance, at which time he was confused, and complained of severe generalized headache. Past medical history is positive for well-treated hypertension, 2 packs per day smoking history of 24 years duration, occasional social drinking, and a 2-month history of intermittent right ocular pain. Physical exam reveals a temperature of 98.5 F by mouth, blood pressure of 190/110 mm Hg, pulse rate of 116/min and regular, with respirations 22/min and unlabored. He is a well-developed, well-nourished, middle-aged man. He is awake, but drowsy, and he has a severe headache. There is mild nuchal rigidity. Fundi are clear. The rest of the neurological exam is normal. There are no signs of trauma. Labs reveal a white blood cell count of 11,500/mm3, with a normal differential. The chemistry profile is normal. Electrocardiogram reveals prolonged QRS complexes, prolonged QT interval, and deeply inverted and symmetric T waves. Unenhanced computed tomography of the head reveals a large collection of blood in the right Sylvian fissure and lesser amounts in the basal cisterns. Question The patient is identified as having an aneurysmal subarachnoid hemorrhage (SAH). What statement about this condition is most valid? Answer Choices 1 Early mortality after SAH is about 10% 2 Normal head CT rules out the diagnosis of SAH 3 The risk of rebleeding is highest in the first 6 to 24 hours after the initial bleed 4 He has an increased risk of developing right-sided hemiparesis 5 He has a worse prognosis because of his history of hypertension

The risk of rebleeding is highest in the first 6 to 24 hours after the initial bleed The patient has had a hemorrhagic stroke due to a ruptured saccular ("berry") aneurysm. Rebleeding is most common in the first 6 to 24 hours. Early mortality from this condition approaches 50%, and those who survive often suffer significant permanent deficits (4). Noncontrast head CT is the most common modality for diagnosis of SAH and is nearly 100% sensitive, especially in the first few hours after a bleed. However, when the bleed is more remote or more minor, the CT scan may be negative. In this case, a lumbar puncture should be performed and will reveal red blood cells (6). Hypertension is seen in almost all patients following a bleed; it tends to be mild to moderate and fluctuate with the degree of head pain (1). His history of hypertension does not worsen his prognosis; a history of preexisting hypertension is only slightly more likely in patients with ruptured saccular aneurysms as compared to matched controls. Hemiparesis certainly may occur, but it would likely occur on his left side based on the location of the intracranial bleed.

A 32-year-old man presents with severe headaches. He reports that he had experienced 2 of these severe headaches about a month ago, and now has had 2 more, which have awoken him from sleep. The headaches lasted anywhere from 15 minutes to about an hour, then resolved entirely. His wife urged him to seek care for this one, as she was worried about a stroke. He complains of severe head pain, and his left eye appears "droopy," slightly swollen, and is watering. His nose is runny. He denies history of head trauma, memory loss, gait disturbances, muscle weakness, nausea, and vomiting. Over-the-counter acetaminophen and ibuprofen have not been helpful. He has no known medical conditions, takes no regular medications, and has no known allergies. He has never had surgery. He denies any family history of headaches or neurological conditions. He smokes 1 pack/day and drinks alcohol 3-4 times/week. He denies the use of drugs. On physical exam, the patient appears restless. Ptosis and pupillary constriction are observed in his left eye, and some clear nasal discharge is noted. The remainder of his exam is normal. Question What test results best support this patient's likely diagnosis? Answer Choices 1 Inflammation on temporal artery biopsy 2 Normal magnetic resonance imaging (MRI) 3 Polyspikes and slow wave pattern on electroencephalogram (EEG) 4 Positive toxicology for cocaine 5 Thickened sinus mucosa on computed tomography (CT)

This patient is most likely experiencing cluster headaches, in which he would be expected to have normal magnetic resonance imaging (MRI). Cluster headaches are intense, episodic headaches, which occur more commonly in men than women. Symptoms are unilateral, often accompanied by rhinnorhea and lacrimation, miosis, and ptosis of the eye. Inflammation on temporal artery biopsy indicates temporal arteritis, which can cause severe unilateral headache. Temporal arteritis is more likely to occur in an older person and is not associated with the rhinorrhea and these optic symptoms. (There may be amaurosis fugax, diplopia, and vision loss.) Polyspikes and slow wave pattern on electroencephalogram (EEG) indicates seizure activity. This patient's symptoms and headache pattern are not associated with any seizure disorder. If the patient had positive toxicology for cocaine, he could present with restlessness and mydriasis (dilation of the pupils - bilaterally), rather than unilateral pupillary constriction. Thickened sinus mucosa on computed tomography (CT) indicates chronic sinusitis, which can cause headaches. The headaches associated with chronic sinusitis are much less intense/acute and episodic than those of cluster headaches.

A 54-year-old man presents after having a generalized seizure. The patient is HIV positive, but he has been unable to afford antiretroviral therapy since losing his job 2 years ago. Other than cachexia, the physical exam is unremarkable. Upon further inquiry, the patient also notes that he has become short-tempered and hypercritical; at times, he seems confused. An MRI of the brain is performed, and it reveals several cortical ring-enhancing lesions. Question What is the most likely diagnosis? Answer Choices 1 AIDS dementia complex 2 Cryptococcal meningitis 3 Cytomegalovirus encephalitis 4 Progressive multifocal leukoencephalopathy 5 Toxoplasma encephalitis ANS:5

Toxoplasma encephalitis The patient's symptoms and MRI findings are most consistent with the diagnosis of toxoplasma encephalitis. Toxoplasmosis is the most common cerebral mass lesion among HIV-positive patients. Infection with the Toxoplasma gondii parasite is relatively common and usually asymptomatic. Reactivation occurs in HIV positive patients due to failing cellular immunity, and it causes a multifocal necrotizing encephalitis. Seizures may be the initial manifestation of central nervous system (CNS) infection; other common clinical manifestations include focal neurologic deficits, such as impaired speech and hemiparesis. Personality change, lethargy, headache, and confusion are also observed. The MRI in patients with toxoplasma encephalitis characteristically reveals multiple, ring-enhancing lesions with surrounding edema; these lesions usually occur bilaterally in the frontal and parietal cortices. AIDS dementia complex describes a constellation of cognitive symptoms seen among HIV positive patients. The condition occurs when HIV virus disseminates to the CNS. Within the CNS, the virus tends to concentrate in the basal ganglia and subcortical regions. Symptoms include a constellation of cognitive, behavioral, and motor disturbances that cause varying degrees of functional impairment. Characteristic MRI findings include non-enhancing white matter, cerebral atrophy, and ventricular enlargement. The diagnosis requires that other central nervous system infections, carcinoma, as well as general medical conditions and substance abuse have been excluded. Cryptococcal meningitis is caused by the encapsulated fungus Cryptococcus neoformans. Among HIV positive patients, the illness may be the result of new infection or reactivation of latent infection. Presenting signs are often nonspecific; they include headache, fever, change in mental status, and nausea or vomiting. Nuchal rigidity and photophobia may also be present, and elevated intracranial pressure is not uncommon. MRI findings vary, but they include lesions in the basal ganglia; meningeal enhancement, cerebral edema, and shrunken ventricles may also be seen. Cytomegalovirus (CMV) infection causing encephalitis is usually observed in patients with evidence of CMV infection at other sites. MRI findings vary, but they often show areas of focal necrosis within the brain parenchyma, meninges, or periventricular regions. Symptoms typically reflect progressive dementia, with episodes of confusion, apathy, and focal neurologic deficits. Progressive multifocal leukoencephalopathy is often a fatal disorder; it is caused by reactivation of a latent JC viral infection. Focal neurologic deficits such as hemiparesis and gait disturbance are often the initial presenting symptoms; they are followed by progressive cognitive decline, coma, and death. The MRI commonly reveals multiple, non-contrast enhancing foci in cerebral white matter.

What is the treatment of choice for primary generalized seizures? Answer Choices 1 Phenytoin 2 Phenobarbital 3 Valproic acid 4 Carbamazepine 5 Gabapentin

Valporic Acid Epilepsy is defined as a syndrome characterized by recurrent, unprovoked seizures. A seizure is best defined as a discrete neurologic event caused by abnormal, excessive, spontaneous discharge of cerebral cortical neurons. The phenomenology of a single seizure is dependent upon the origin and subsequent spread of neuronal discharge. Epileptic seizures, as opposed to symptomatic seizures provoked by acute brain disturbances (e.g. hyponatremia, meningitis, cocaine use), are treated medically with anticonvulsants. Specific anti-epileptic drugs are efficacious for different types of seizure. Primary generalized epilepsy is commonly associated with developmental brain disorders of childhood (congenital and acquired). Types of primary generalized seizures include tonic-clonic, absence, tonic, clonic, myoclonic, and atonic. Anticonvulsants indicated for primary generalized seizures include valproic acid, clonazepam, and ethosuximide. Localization-related epilepsy results in partial (focal) seizures with or without secondary generalization. These are best treated with phenytoin, carbamazepine, phenobarbital, and valproic acid. The newer anticonvulsants, gabapentin, felbamate, and lamotrigine are also effective for partial seizures.

A 70-year-old man presents with memory loss that has been worsening over the past few months. He also has history of slurred speech and walking difficulty. There is a history of getting lost in familiar surroundings. His history is also significant for a stroke 2 years ago. On examination, there are focal neurological signs including plantar response, gait abnormalities, exaggeration of deep tendon reflexes, and weakness of the right lower limb. Brain imaging studies show multiple large-vessel infarcts and extensive periventricular white matter lesion. What is indicated by evidence of relevant cerebrovascular disease on brain imaging and by focal signs, along with temporal relationship of stroke and dementia, or abrupt cognitive deterioration and stepwise progression? Answer Choices 1 Lewy body dementia 2 Vascular Dementia 3 Dementia of the Alzheimer's Type 4 Dementia Due to Parkinson's disease 5 Delirium ANS;2

Vascular dementia Dementia is defined as a multifaceted decline in cognitive functioning that causes impaired functioning in daily life. Memory impairment is generally regarded as a necessary aspect, but decline in one or more other cognitive domains (such as language, praxis, gnosis, visuoconstructive function, frontal-executive functions) also should be demonstrated. Vascular dementia is the second most common cause of dementia after Alzheimer's. Multi-infarct dementia is the most common type of vascular dementia. The criteria for the diagnosis of vascular dementia includes evidence of relevant cerebrovascular disease on brain imaging (brain imaging must show evidence of multiple large-vessel infarcts, a single strategically placed infarct, multiple basal ganglia, white-matter lacunae's or hemorrhages, extensive periventricular white-matter lesions, or combinations of these), by focal signs along with temporal relationship of stroke and dementia, or abrupt cognitive deterioration and stepwise progression. The course must come on gradually and be accompanied by continuing cognitive decline, and there must be no evidence that the dementia is due to a physical condition. Frequently, laboratory studies, MRI, or CT scans will confirm the diagnosis. Lewy body dementia is caused by the abnormal microscopic deposition of protein in nerve cells, called Lewy bodies, which cause destruction of the nerve cells over time. These deposits can cause symptoms typical of Parkinson disease, such as tremor and muscle rigidity, as well as dementia similar to that of Alzheimer's disease. Delirium is a state of transient global disorder of cognition. White matter underlying the cortex can aid in determining the etiology of this disorder. Alzheimer's dementia is the most common type of dementia that is caused by a loss of brain cells. It is a form of degenerative brain disease resulting in progressive mental deterioration with disorientation, memory disturbance, and confusion. Parkinson's disease is a progressive neurological disorder in which there is limb stiffness, speech problems, and tremor. Dementia may develop late in the disease, but not everyone with Parkinson's disease has dementia.

A 70-year-old woman is brought to your attention by her family because of the slowly progressive gait disorder, the impairment of mental function, and urinary incontinence. About 1 year ago, she started having weakness and tiredness in her legs, followed by unsteadiness; her steps became shorter and shorter, and she also experienced unexplained backward falls. She is becoming emotionally indifferent, inattentive, and her actions and thinking have became "dull". Over the past month, she has started having urinary urgency and involuntary leaking of urine. Besides multivitamins and local application of the Timolol for glaucoma, she takes no other medications; there are no other symptoms. Question What is most likely the best method of treating the patient's urinary problems? Answer Choices 1 Antimuscarinic drug (Tolterodine) 2 Antibiotic (Sulfamethoxazole/trimethoprim) 3 Acetylcholinesterase inhibitor (Donepezil) 4 Ventriculoperitoneal shunt 5 Kegel exercises ANS: 4

Ventriculoperitoneal shunt Clinical triad of slowly progressive gait disorder, followed by impairment of mental function and then sphincteric incontinence strongly suggests the presence of normal-pressure hydrocephalus. Ventricular expansion is the cause of symptoms, and surgical CSF shunting is the main treatment modality. The potential benefit from surgery is usually evaluated by testing gait, cognition, and micturition before and after CSF drainage. Antimuscarinic Tolterodine is an antispasmodic that is used for symptomatic treatment of urinary incontinence in patients with an overactive bladder (urge incontinence). Antimuscarinic drugs are contraindicated in patients with glaucoma. A urinary tract infection will probably manifest with a strong, persistent urge to urinate, burning sensation when urinating, passing frequent, small amounts of urine that has unusual smell and the appearance. Your patient has no such signs and symptoms; therefore, in this case, antibiotics are not indicated. Donepezil is used to treat dementia, but in the case of normal-pressure hydrocephalus, the problem is anatomic (the distortion of the periventricular limbic system and frontal lobes), and the best treatment is probably surgical. Kegel exercises can prevent or control urinary incontinence and other pelvic floor problems in cases of pelvic sphincter weakness. However, pelvic sphincter weakness will probably manifest as stress incontinence.

A 17-year-old adolescent male presents with unexplained neurological symptoms. His liver is enlarged on palpation, and he has other symptoms of hepatitis. Blood work reveals depressed ceruloplasmin levels. An ophthalmological examination reveals Kayser-Fleischer rings. What is the most likely diagnosis? Answer Choices 1 Krabbe disease 2 Lesch-Nyan syndrome 3 Marfan syndrome 4 Wilson disease 5 Hunter syndrome ANS:4

Wilson Disease This young man is suffering from Wilson's disease, a genetic disorder of copper metabolism. It is inherited as an autosomal recessive mutation in the ATP7B genes located on chromosome 13. The protein of the ATP7B gene is a copper-transporting ATPase The frequency of the heterozygous carriers is relatively high (1/90). The incidence of homozygous recessive affected individuals is about 1/30,000. It affects all ethnic groups and both sexes equally. Neurological symptoms, hepatitis, and Kayser-Fleisher rings, greenish-brown deposits of copper in the corneal endothelial (Descemet membrane) basement membrane near the peripheral cornea where it meets the iris (limbus), are characteristic of Wilson's disease. About 40% of the dietary copper is absorbed in the gastrointestinal tract, and makes its way to the liver bound to albumin. In the liver, it is complexed with ceruloplasmin, a blood protein that carries most of the copper. Ceruloplasmin levels are abnormally low in patients with Wilson disease, although the disease is not, per se, a mutation in the ceruloplasmin gene. Ceruloplasmin is recycled in the liver by the usual lysosomal degradation pathway, and the unused copper is excreted in bile. When excessive copper is absorbed in the gut, it accumulates in the brain (producing neurological symptoms), the liver (producing hepatitis and hepatomegaly), and the cornea. Penicillamine can be used to treat this disease. It chelates copper and provides some symptomatic relief. Unfortunately, when used appropriately in pregnant women to treat potentially life-threatening Wilson disease, it is harmful to the fetus and can produce cutis laxa in the newborns of penicillamine-treated patients (see Figure H6.3).

A 43-year-old woman presents for the evaluation of possible seizures. The patient was well until the day of admission to the hospital. She works as a secretary at a law firm. She was having a heated argument about work assignments with her supervisor when she slumped back in her chair, became pale, and then had a 5-10 second spell of generalized tonic-clonic jerking. She was placed on the floor, and she began to rouse after 15-30 seconds. Past medical history reveals a history of 3 generalized tonic-clonic seizures at age 4. Electroencephalogram at that time revealed intermittent generalized spike and wave discharges. She was started on divalproex sodium (Depakote) and had no further seizures. At age 8, a repeat electroencephalogram was normal and divalproex was discontinued. Seizures did not reoccur. She was in a major automobile accident at age 27, was in the hospital for 3 months, and in rehabilitation for another 3 months. She made a full recovery. The patient describes her health now as 'excellent'. Physical examination reveals a well-developed, well-nourished woman; she is awake, alert, fully-oriented, and in no acute distress. Temperature is 98.8 F orally. Blood pressure is 129/85mmHg. Heart rate is 76/min and regular. Respirations are 14/min and unlabored. General neurological and medical exams are normal. Lab studies include a normal complete blood count and differential, blood chemistry panel, urinalysis, chest X-ray, and cardiogram. A portable electroencephalogram is done and reported as normal. Syncope and seizures are being considered, but pseudoseizures are too because of the relationship of this spell to emotional distress. Question What profile is best correlated with the likelihood of pseudoseizures (psychogenic non-epileptic seizures) in patients where pseudoseizures are a consideration? Answer Choices 1 Young (20 - 30 years old) women with history of sexual abuse 2 Middle-aged (40 - 60 years old) men with a history of seizures 3 Young (20 - 30 years old) men with serious medical problems 4 Middle-aged (40 - 60 years old) men with a history of fibromyalgia 5 Middle-aged (40 - 60 years old) males with a history of drug abuse 6 Elderly men with a history of physical abuse

Young (20 - 30 years old) women with history of sexual abuse While pseudoseizures (Psychogenic non-epileptic seizures) can occur at any age, the great bulk of them occur in the 20-30 year old age group. Women predominate (75%) in this age group, although this gender difference fades with years and nears 50% in the elderly. A history of sexual abuse can be obtained in 32% of women with pseudoseizures in this age group. The incidence of reported sexual abuse decreases steadily with age (4% in the over 55 age group). The middle years (20-40) are relatively quiet for the incidence of pseudoseizures. Although the percentage of men among patients with pseudoseizures increases with age, it never reaches 50%. While incidence of true epilepsy is frequent among patients with seizures (it is felt that 5-20% of those with a diagnosis of seizures have pseudoseizures), this correlation is seen mostly in the young and is not a serious factor over 55 years of age (2). Although the 20-30 year old age group contains the bulk of patients with pseudoseizures, the men in this age group are a distinct minority (25%). Among the elderly who have pseudoseizures, there is a relatively high incidence of major health problems (42%), especially serious cardiovascular issues (2), but this is not seen in the young age group. Again, the middle years are relatively quiet with respect to pseudoseizures. Men remain in the minority. However, a history of fibromyalgia does have a predictive value of 85% for pseudoseizures (3). Still again, the middle years are relatively quiet for pseudoseizures and men remain in the minority. There is, however, a strong relationship between drug abuse and pseudoseizures, but not as strong as among the young. While the elderly can have pseudoseizures, it is relatively uncommon. Again, men are in the minority. A history of physical abuse is uncommon in males. Summary of pseudoseizure profile ('risk factors'-none in themselves are absolute indicators) 1) Ironically, a group at increased risk for pseudoseizures is epileptics, where epilepsy and pseudoseizures frequently coexist (5). While 0.5-1.0% of the population may have a diagnosis of epilepsy, it is felt that 5-20% of these actually have pseudoseizures. This association decreases with age. 2) Age; predominantly younger patients (10-30) but can be seen in any age. 3) Gender; 78% of younger patients are women (1), but this difference fades with advancing age. 4) A diagnosis of fibromyalgia has an 8% predictive value that the patient has pseudoseizures (3). 5) A diagnosis of chronic pain syndrome also has a predictive value of 85% that the patient's spells represent pseudoseizures (3). 6) The rule of 2's; a history of 2 normal EEGs, 2 seizures per week, and resistance to 2 antiepileptic drugs have a positive predictive value for of 85%. 7) A history of sexual abuse is seen in about 32% of young women, but much less so in men, and drops to a major degree in both genders with age (2). 8) A history of major health problems becomes increasingly common with age with a cardiovascular predominance (2). 9) A history of illicit drug abuse is common (1). 10) Pending compensation issues (5).

A 40-year-old man presents to the emergency department with severe pain in his left eye, decreased vision, nausea, and abdominal pain. On examination, the patient's left pupil is moderately dilated and nonreactive. The cornea is "steamy" in appearance and generally the eye is red. What do you suspect is the cause? Answer Choices 1 Conjunctivitis 2 Acute uveitis 3 Acute angle-closure glaucoma 4 Corneal ulcer 5 Corneal infection ANS:3

acute angle closure glaucoma This patient has acute angle-closure glaucoma. The typical characteristics of this condition are all exhibited by this patient (i.e., steamy cornea, severe pain, blurred vision, dilated, and nonreactive pupil). A physical exam finding in a patient with acute uveitis would be a small pupil. In addition to these symptoms, patients may also present with abdominal pain. A corneal infection and corneal ulcer would cause circumcorneal injection and watery or purulent discharge.

A 33-year-old African-American woman presents with a new onset of left facial droop. The droop was present when she awoke this morning, and it has worsened since then. She reports that she had been feeling fine. She denies headaches, visual changes, and weakness or tingling in the extremities. Exam reveals an anxious, overweight woman. She is afebrile. Cranial nerve exam reveals weakness of the entire left side of the face, including the forehead. The left eye does not close entirely, and there is a widened palpebral fissure. Sensation is intact. All cranial nerves, other than VII, appear intact. Her tympanic membranes are clear, and there are no skin lesions around the face or head. The rest of her exam is unremarkable. Labs reveal normal CBC, serum electrolytes, thyroid function tests, and liver transaminases. A chest X-ray is clear. Question What is the most likely cause of this patient's facial droop? Answer Choices 1 Bell's palsy 2 Sarcoidosis 3 Ramsay-Hunt syndrome 4 Lyme disease 5 Cerebrovascular accident (CVA) ANS:1

bell's palsy This patient has a classic presentation of Bell's palsy. Bell's palsy is a sudden, unilateral facial weakness and affects about 1 out of every 60 or 70 persons in their lifetime. The etiology is unknown at this time, but herpes simplex virus type 1 has been associated with some cases. The presumed pathogenesis of the palsy is that inflammation of the facial nerve causes swelling then secondary compression, and ischemia occurs where the nerve courses through the temporal bone. Pain behind the ear may precede the paralysis by a day or 2. Hyperacusis and loss of taste may occur, if the lesion is proximal enough; lacrimation may be affected as well. Sensation is not affected, although patients may experience numbness or heaviness in the face. In some cases, there is a mild lymphocytosis of the CSF. MRI may reveal swelling and uniform enhancement of the geniculate ganglion and facial nerve. Also, in some cases, MRI may reveal a swollen facial nerve entrapped within the temporal bone. Prognosis depends upon the degree of paralysis. Approximately 80% of patients will recover within a few weeks to a month. Of those with an incomplete paralysis in the 1st week, nearly all will experience complete recovery within several months. Of those with a more complete paralysis, like this patient, nerve conduction studies and electromyography can indicate prognosis. The likelihood of completely recovery is 90% if the nerve branches in the face retain normal excitability to supramaximal electrical stimulation; however, the likelihood is only about 20% if electrical excitability is absent. Treatment includes symptomatic care, such as taping the upper eyelid closed during sleep and using lubricating agents in the eye In order to prevent corneal desiccation. Some studies have shown improvement in patients treated with glucocorticoids; other studies showed improvement with prednisone and acyclovir together. Sarcoidosis can cause a form of bilateral facial paralysis (facial diplegia) called uveoparotid fever (Heerfordt's syndrome). Given this patient's African American background, sarcoidosis should be considered as a possible etiology of her symptoms. However, the unilateral nature of the paralysis, combined with a negative chest X-ray, makes sarcoidosis unlikely. A serum angiotensin converting enzyme (ACE) could be drawn if sarcoidosis is a concern. Ramsay-Hunt syndrome is a facial palsy associated with a vesicular eruption in the pharynx, external auditory canal, and other surrounding areas of skin. This is thought to be secondary to herpes zoster infection of the geniculate ganglion; often the 8th cranial nerve is affected, as well. This patient did not have any skin lesions around the ear, thus making this diagnosis very unlikely. However, this demonstrates the importance of a thorough examination in facial palsy patients, including careful examination inside the ears. Lyme disease is a frequent cause of facial paralysis in areas where Lyme infection is endemic. However, the majority of patients who have facial palsy caused by Lyme disease note an antecedent rash adjacent to the site of a tick bite. Blood tests for Lyme disease can help identify that as an etiology. However, this patient denied any risk factors for tick bites such as camping/ travel, making this diagnosis less likely. The family history of CVA and the fact that she is currently taking oral contraceptives do confer some increased risk for thrombotic events in this patient. However, with a stroke, the forehead is usually spared from facial palsy, since the upper facial muscles (the frontalis and orbicularis oculi muscles) are innervated by corticobulbar pathways from both motor cortices. The lower facial muscles are innervated only by the contra-lateral hemisphere. This patient's facial palsy is complete and includes the forehead because the facial nerve itself is affected in Bell's palsy.

A 60-year old woman has a history of stress related migraine headaches. Many conventional treatments have failed. What treatment would you now try on this client? Answer Choices 1 Aversion therapy 2 Biofeedback 3 Flooding 4 Systematic desensitization 5 Token economy ANS:2

biofeedback There are several different types of behavioral therapies, and each one is unique in regards to the type of problem being treated. Biofeedback is a form of behavior therapy involving relaxation and the power of suggestion. It is used to treat psychosomatic-related ailments such as asthma, depression, migraine headaches, hypertension, stress, chronic pain, and digestive problems. Token economy involves the use of rewards to condition behavior. These rewards may include money, privileges, food passes, and gifts. This therapy is utilized a lot in inpatient or residential settings, such as prisons, mental hospitals, nursing homes, and substance abuse treatment centers. Aversion therapy is a type of conditioning in which a negative stimulus is connected with an undesired behavior. For example, to assist an individual to stop smoking, one might add an unpleasant taste or smell when a smoker lights up a cigarette. This type of therapy is used frequently in smoking, weight loss, or other undesired behaviors. Systematic desensitization is a behavioral technique in which a client with some type of avoidant behavior linked to a specific stimulus is asked to construct a hierarchy of anxiety-provoking images until the avoidant behavior diminishes. Examples of fears the technique may work on include heights, snakes, airplane travel, elevators, escalators, and many others. Flooding is a technique in which the client is exposed to the highest possible anxiety-provoking experience related to whatever they fear the most. For example, instead of proceeding slowly as in systematic desensitization, the client is immediately exposed to a tall building, snake, elevator, escalator, or whatever they fear the most. This is a highly controversial therapy, and it should only be used with a great degree of caution and knowledge. Treatment of major depression in older adults is most successful with a combination of antidepressant medications plus interpersonal supportive psychotherapy. Behavioral, cognitive, and family therapy has shown less promising results. Brief or short therapy has shown to be as effective, or more effective, compared to long-term psychotherapy. In addition, most insurance programs do not fund long-term therapies. Older adults with adjustment reactions usually have much less severe symptoms, and these usually occur after a major stress such as a death, change in residence, or other psychosocial stressors.

A 19-year-old woman presents to the Accident and Emergency Department following a road traffic accident 20 minutes prior. She states her car was just lightly bumped from behind, but she is concerned she might have whiplash, as her neck is stiff and sore. She also says she's experiencing tingling in her left arm down to her thumb. On physical examination, her bicep strength is +3/5 on the left and +5/5 on the right. Her bicep tendon reflex is 2+ on the left and 2+ on the right. In addition, her brachioradialis tendon reflex is 1+ on the left and 2+ on the right. The remainder of the physical examination is normal. Question Based on the above presentation, what cervical nerve root is most likely affected? Answer Choices 1 C4 2 C5 3 C6 4 C7 5 C8

c6 The most likely nerve affected in this case is the C6 nerve root. Impingement of this nerve can cause numbness and tingling down the arm into the thumb, with weakness in the bicep muscle and diminished brachioradialis tendon reflex in the affected extremity. Impingement of the C4 nerve root may cause neck and upper shoulder numbness and pain. Impingement of C5 nerve root may cause deltoid and shoulder numbness and pain, and bicep tendon reflex may be diminished. Impingement of C7 also causes numbness and pain down the affected arm, but reflex may be diminished into the middle finger and the triceps,. Impingement of C8 may cause numbness and tingling primarily on the lateral surface of the arm and into the lateral hand into the fourth and fifth digits. It may also cause dysfunction of the hand as it innervates the small hand muscles.

A 73-year-old man presents after a 15-minute episode of right eye vision loss, which he described as being "like a shade being pulled down". What diagnostic test is most likely to be abnormal? Answer Choices 1 Carotid ultrasound 2 Computed tomography 3 Electrocardiogram 4 Erythrocyte sedimentation rate 5 Electroencephalogram ANS:1

carotid u/s The case described above is a classic example of amaurosis fugax, which is caused by transient occlusion of the ophthalmic artery, a branch of the internal carotid artery. This occurs most often in the setting of internal carotid artery stenosis. Like other transient ischemic attacks, it is often a harbinger of an impending stroke.

A 4-year-old boy presents with erratic movements of his left arm and leg. These movements have been present since birth and occur on the left side. They have not been getting worse. The boy's mother admits that he has been using his right hand for almost all activities since the age of 1. Further questioning reveals that the child exhibited delayed milestones, such as rolling over at 6 months, sitting up at 1 year, and walking at 2 years. He does not speak, and he does not respond to his mother's verbal commands. His physical exam is noteworthy for spasticity, hyperreflexia, ataxia, involuntary movements, and weakness of his left arm and leg. His left arm and left leg are measured to be shorter and are atrophied in comparison to the right. Additionally, the child does not respond to verbal stimuli. Question What is the most likely diagnosis? Answer Choices 1 Duchenne muscular dystrophy 2 Lesch-Nyhan syndrome 3 Cerebral palsy 4 Huntington's disease 5 Rett syndrome

cerebral palsy The most likely diagnosis in this patient is cerebral palsy. Cerebral palsy is a chronic, nonprogressive impairment of muscle tone, strength, coordination, or movements. Cerebral palsy is due to cerebral insult or injury before birth, during delivery, or in the perinatal period. The most common form of cerebral palsy (75% of cases) involves spasticity of the limbs. Depending on the type and severity of the motor deficits, associated neurologic deficits or disorders may occur as follows: seizures in up to 50%; mild mental retardation in 26%; and severe retardation in up to 27%. Disorders of language, speech, vision, hearing, and sensory perception are found in varying degrees and combinations. Gross motor milestones of concern with cerebral palsy include head control at age 2 months, rolling at age 4 months, sitting at age 6 months, and walking at age 1 year. Infants with cerebral palsy may have significantly delayed gross motor milestones or show an early hand preference when younger than 1.5 years, suggesting the relative weakness of 1 side. While this patient demonstrates a hemiplegic pattern, other patterns are possible, such as monoplegia or quadriplegia. The findings on physical examination in those with spastic cerebral palsy are variable and are predominantly those of spasticity, clonus, hyperreflexia, ataxia, and involuntary movements. Physical indicators of cerebral palsy include joint contractures secondary to spastic muscles, growth delay, and persistent primitive reflexes. Microcephaly is frequently present. In patients with hemiplegia, the affected arm and leg may be smaller and shorter than the unaffected limbs. Duchenne-type muscular dystrophy, which is also called pseudohypertrophic muscular dystrophy, is a progressive disease that affects boys. Early signs of disease include pseudohypertrophy of the calf, planovalgus deformity of the feet, and proximal muscle weakness. Muscle weakness in the hips may be exhibited by the Gower sign, in which the patient uses the arms to support the trunk while attempting to rise from the floor. Other signs are hesitance when climbing stairs, acceleration during the final stage of sitting, and shoulder weakness. Lesch-Nyhan syndrome is characterized by hyperuricemia, self-mutilative behavior, choreoathetosis, spasticity, and mental retardation. Hyperuricemia results from urate overproduction and can cause uric acid crystalluria, nephrolithiasis, obstructive uropathy, and gouty arthritis. The onset of Huntington's disease is usually between 30 and 50 years of age. The initial symptoms may consist of either abnormal movements or intellectual changes, but ultimately both occur. The earliest mental changes are often behavioral. Symptoms may include irritability, moodiness, antisocial behavior, and a psychiatric disturbance. A more obvious dementia subsequently develops. Dyskinesia may initially manifest as fidgetiness or restlessness, but eventually choreiform movements and some dystonic posturing may occur. Progressive rigidity and akinesia (rather than chorea) sometimes occur in association with dementia, especially in cases with childhood onset. Rett syndrome affects 1 of every 10,000 to 15,000 girls. After 6 to 18 months of normal development, motor skills and mental abilities seem slowly to regress. Certain handwringing and other stereotyped hand movements appear as the disease progresses and are characteristic of the disease.

A 19-year-old college freshman living in a dormitory presents with fever and chills, intense headache, vomiting, and stiff neck. His temperature is 39.2°C and pulse 140/min. After examination, the clinical diagnosis of meningitis is made. Lumbar puncture reveals fluid that is turbid and under pressure. The cerebrospinal fluid (CSF) and blood are sent for microbiological studies. The patient is placed on intravenous ceftriaxone. Gram stain of smears made from centrifuged deposit of CSF show plenty of polymorphonuclear leucocytes, intracellular gram-negative diplococci, and a few extracellular Gram-negative diplococci. A culture of CSF on chocolate agar media incubated at 37°C in presence of 5% CO2 grows colonies of Gram-negative diplococci that are oxidase-positive. By conventional tests and agglutination reaction, the isolated pathogen is identified as Neisseria meningitidis group B. Question What prophylactic measure should be taken by other students staying in the dormitory? Answer Choices 1 Chemoprophylaxis with vancomycin 2 Immunization with tetravalent meningococcal polysaccharide vaccine 3 Chemoprophylaxis with oral penicillin 4 Chemoprophylaxis with rifampin 5 Immunization with tetravalent meningococcal conjugate vaccine

chemoprophylaxis with Rifampin At present, chemoprophylaxis is the only prophylactic measure available against N. meningitidis group B. Among the drugs listed, rifampin is the one used for the purpose. Vancomycin is not effective against most Gram-negative bacteria, including neisseria species. Oral penicillin does not eradicate N. meningitidis from the nasopharynx. The tetravalent vaccines do not contain group B polysaccharide and have no effect against group B meningococcal infection. Close contacts of the patient with meningococcal disease are at greater risk of acquiring the infection. Prophylaxis is indicated in those individuals and in populations with increased carrier rates. Nasopharyngeal mucosa is the site of colonization of N. meningitidis. Overcrowding, as is often found dormitories, causes an increase in carrier rates. Prevalence of carriage is reported to be highest among adolescents and young adults. Transmission occurs through direct contact with large droplets of respiratory secretions from patients or asymptomatic carriers. After colonization, development of invasive disease depends on the strain, environmental, and host factors. Chemoprophylaxis eradicates the carrier state and thus prevents spread of infection. It should be administered as early as possible, ideally within 24 hours of identifying the index case, in order to be effective in preventing secondary cases. Rifampin is one of the drugs commonly used. Mass prophylaxis with rifampin during large outbreak settings is a cause for concern as it might lead to circulation of rifampin-resistant strains. However, mass chemoprophylaxis with rifampin might be considered in outbreaks involving limited populations, such as in a dormitory, single school, or college. It would be especially advisable in cases caused by serogroup B, for which an effective vaccine is not yet available. Rifampin resistance develops due to mutations in the rpoB gene. Alternative drugs that are recommended for chemoprophylaxis are ciprofloxacin and ceftriaxone. Ceftriaxone is administered intramuscularly. Ciprofloxacin is not indicated in small children. N. meningitidis is now the leading cause of bacterial meningitis in the US, and each year 1,400-2,800 cases occur. More than 98% are sporadic infections. Localized outbreaks involving colleges, schools, and nursing homes have been reported. Groups B, C, and Y are major causes of meningococcal disease in the US, and group B is responsible for approximately 30% of cases. The disease affects mainly infants, adolescents, and young adults. No vaccine is yet available against N. meningitidis group B. Development of effective polysaccharide vaccine is hindered by the poor immunogenicity of group B polysaccharide. The lack of immunogenicity is due to immunological cross reactivity with fetal neural antigens. A vaccine containing group B polysaccharide might trigger autoimmunity because of the molecular mimicry. Vaccines using non-capsular antigens like outer membrane proteins (outer membrane vesicle vaccine) and factor H-binding protein (multi-component group B recombinant protein vaccine) are under development. These require standardization and validation. Vancomycin is not used for chemoprophylaxis of meningococcal infection. Most of the Gram-positive normal flora is susceptible to this drug. So this antibiotic is used as a constituent of modified Thayer martin medium for selective isolation of pathogenic neisseria. The medium can be used for isolation of N. meningitidis from nasopharyngeal swabs for detection of carriers. Immunization with tetravalent meningococcal polysaccharide vaccine (MPSV4) is not useful as a measure of prophylaxis for infection by group B meningococcus. The preparation contains only polysaccharides from groups A, C, Y, and W-135 and protects against infection by these 4 serogroups. MPSV4 is effective in older children and adults, and it is not immunogenic in children less than 2 years of age. Oral penicillin is not used for chemoprophylaxis of meningococcal meningitis, although most strains of meningococci are sensitive to penicillin. It is not found effective in eradicating meningococci from the nasopharynx. Tetravalent meningococcal conjugate vaccine (MCV4) is not useful against group B meningococcal infection. It contains polysaccharides from A, C, Y, and W-135 serogroups of N. meningitidis conjugated with diphtheria toxoid, and it is useful for preventing infection by these 4 serogroups. This vaccine is considered more effective than MPSV4. It stimulates both B-cell and T-cell dependent immune responses. As a result, the vaccine induces a strong booster effect that confers long-term protection, and it helps in reducing nasopharyngeal carriage. It is also reported to be safe and immunogenic in 2- to 10-year-olds. Centers for Disease Control and Prevention recommends this vaccine for routine immunization of adolescents, college freshmen living in dormitories, military recruits, and other populations at increased risk of developing invasive meningococcal disease. MCV4 has been licensed for use in individuals 11-55 years old.

A 55-year-old man starts to develop behavioral and mental changes. In addition, he is having coarse, spasmodic, involuntary movements that involve his face and his extremities. What is the classification of this abnormal movement? Answer Choices 1 Chorea 2 Athetosis 3 Ballism 4 Asterixis 5 Dystonia ANS:1

chorea Chorea is a spasmodic involuntary movement that involves the face and limbs. Chorea can be seen with Huntington's disease (Huntington's chorea). Athetosis is slow, writhing, twisting involuntary movement. Almost all parts of the body can be affected by athetosis. Speech can also be affected when the athetosis is affecting the tongue. Depending on the characteristic of the movements that are seen, it is sometimes modified and called choreoathetosis or athetotic dystonia. Athetosis can be seen with cerebral palsy and some other conditions. Ballism is a flailing, jerky movement of the limbs. It affects the proximal muscles. If the abnormal movement is on one side, it is called hemiballism. Asterixis is also called "liver flap". It is a flapping movement of the hands or arms. During the physical examination to test for asterixis, have the patient hold up his hands (dorsiflex) and spread his fingers. Asterixis is seen with metabolic encephalopathies. Dystonia is an abnormal movement where there is abnormal contraction of muscles. Dystonia can be divided into primary and secondary. Spasmodic torticollis is a form of dystonia.

An 70-year-old woman is brought to your attention because of the slowly progressive gait disorder; she is also experiencing forgetfulness and problems with micturition. About 1 year ago, she started having weakness and tiredness in her legs, followed by unsteadiness and her steps became shorter and shorter. Sometimes, she forgets where she put things; she has forgotten to turn off the oven 2 times. Over the last month, she has started experiencing urinary urgency, the need for frequent urination, and the involuntary leaking of urine that worries her. She notes no other symptoms. Question What is the most likely cause of her urinary problems? Answer Choices 1 Increased cerebrospinal fluid pressure 2 Urinary tract infection 3 Frontal lobe dysfunction 4 Detrusor muscle instability 5 Pelvic sphincter weakness

detrusor muscle instability The correct response is detrusor muscle instability. A clinical triad of slowly progressive gait disorder, followed by impairment of mental function and sphincteric incontinence, highly suggests the presence of normal-pressure hydrocephalus. Ventricular expansion in hydrocephalus involves frontal horns, affecting the functions of frontal lobes and of basal ganglionic-frontal motor control. The central white matter is affected more than cortical gray matter, thalami, basal ganglia, and brainstem. The distortion of the central portion of the corona radiata by the distended ventricles affects the sacral motor fibers that innervate legs and the sacral bladder center, causing the abnormal gait and incontinence. Decreased inhibition of bladder contractions leads to the instability of bladder detrusors. In cases of normal-pressure hydrocephalus, the formation of CSF equilibrates with absorption; there might be variations in the CSF pressure, but overall, pressure is considered normal. A urinary tract infection will probably manifest with a strong, persistent urge to urinate, a burning sensation when urinating, and passing frequent, small amounts of urine that has unusual smell. Its appearance may be cloudy; it may be red, pink, or cola-colored if there is blood in the urine. Dementia is a consequence of the distortion of the periventricular limbic system and frontal lobes in the cases of normal-pressure hydrocephalus. However, incontinence is only a result of a lack of concern for micturition in extremely advanced cases, in which there is frontal lobe dysfunction. Your patient has insight into her problems and worries; therefore, frontal lobe dysfunction resulting in a lack of concern for micturition is probably not the cause of her urinary problems. Pelvic sphincter weakness will probably manifest as stress incontinence; incontinence happens when a weak sphincter cannot hold the urine and physical pressure is placed on the urinary system, such as when a patient coughs, sneezes, or laughs.

An unidentified motorcyclist is transported to the emergency department via helicopter from the scene of a highway accident. According to eyewitnesses, he appears to have been trying to avoid a piece of debris in the road; however, in doing so, he lost control of his bike and hit the highway median. On arrival, the emergency medical team conducts a rapid assessment, and he is assigned a Glascow Coma Score of 12. Question In addition to motor and verbal responsiveness, what needs to be assessed in order to determine the patient's Glascow Coma Score? Answer Choices 1 Appearance (skin color) 2 Eye response 3 Reflex response 4 Pulse 5 Respirations

eye response The Glascow Coma Score (GCS) is a neurological scale used to assess level of consciousness. It is used as an initial measure of consciousness following head injury, and it is also used to monitor patients following such injuries. There are 3 elements to the assessment: motor response, verbal response, and eye response. Scores range from 3 (deep unconsciousness) to 15 (normal), and are determined as follows: The GCS correlates to degree of brain injury: GCS ≥13, minor injury; GCS 9-12, moderate injury; and GCS 3-8, severe injury (coma). A GCS ≤ 8 generally indicates the need for airway protection via endotracheal intubation.

Viral encephalitis MCC in the US

herpes simplex is the most common cause of sporadic encephalitis in the U.S.

A 60-year-old man presents for evaluation of loss of recent and remote memory, psychomotor slowing, and difficulty with complex tasks. On exam, the patient also shows some tremor tics, dystonia, and impaired coordination. Family history reveals he had 2 brothers who both died of a strange "movement" disorder. What is the provisional diagnosis? Answer Choices 1 Delirium 2 Dementia of the Alzheimer's type 3 Vascular dementia 4 Huntington's disease 5 Substance induced dementia ANS:4

huntington's disease Dementia due to medical conditions include loss of memory and other cognitive deficits due to diseases such as Huntington's disease, Parkinson's, Pick's disease, multiple sclerosis, HIV disease, vitamin B-12 deficiencies, and hypothyroidism. Head traumas and tumors can also cause dementia, and all physical causes must be ruled out first. Vascular dementia presents with the same signs and symptoms as other dementias; however, the patient usually has a strong history of cerebrovascular disease, multiple infarcts, or history of arteriosclerotic disease. Vascular dementia usually has a sudden acute onset after the infarcts. Vascular dementia may also include behavioral disturbances in the patient. In patients with a delirium, there are usually rapid fluctuations in cognitive functioning, brief durations of normalcy, and disturbances in the sleep-wake cycle. In addition, deliriums come on acutely and rapidly. Deliriums are somewhat common in older patients who have had heart surgery and in those who have experienced a change of environments, such as a hospital intensive unit. Clients with substance-induced dementia have cognitive deficits related to a drug or alcohol use or abuse and/or toxicity of prescription drugs; the toxicity can be accidental or intentional. This appears to be more common in the elderly than once thought. Many elderly take prescription drugs incorrectly or mix them with contraindicated over-the-counter drugs. Also, patients may suffer side effects and not report them to their physicians. Dementia of the Alzheimer's type involves gradual loss of memory and other cognitive functioning, which results in the inability to accomplish activities of daily living. Before a diagnosis is made, all other physiological causes should be ruled out.

A 3-year-old boy presents following a car accident. He had not been buckled in, and he was thrown against the windshield. When the paramedics arrived at the scene, he was crying and clinging to his mother. Upon arrival at the emergency department, he is sobbing and does not react to her any more. He sustained lacerations of the face and both arms. His blood pressure is 150/84, pulse 62, and the pupils react sluggishly. He also has a temperature of 39.2° C. What do his symptoms indicate? Answer Choices 1 Viral infection 2 Reye's syndrome 3 Increased intracranial pressure 4 Anxiety attack 5 Guillain-Barré syndrome

increased intracranial pressure This child most certainly sustained head injuries with increased intracranial pressure in the accident. It is very likely, since he was not buckled in, that his face and arms were cut when he was thrown against the windshield. His pupil reaction, disorientation, and not recognizing his mother all support that theory. Elevated temperature is common in patients with head injuries and is called 'central fever'. The only symptom that points to viral infection is the fever, but there would not be any changes in pupil reaction. Reye's syndrome is caused by aspirin medication with a viral infection being present. It affects the central nervous system, but it is not related to trauma. A child may cry and be very anxious in a hospital setting, but that would not cause temperature elevation and sluggish pupils. Guillain-Barré syndrome, or acute idiopathic polyneuritis, is a rapidly progressing ascending motor neuron paralysis of unknown etiology, which often follows viral infections.

A 48-year-old HIV positive man starts to develop headaches. At first, he attributes the headaches to stress. However, they persist and become worse over the next few weeks. He develops nausea and vomiting, and he thinks he has a fever. He starts to become confused, so he seeks medical attention. On physical examination, his temperature is 100°F. He has signs of meningeal irritation. A lumbar puncture is performed, and there is an elevation of his CSF pressure. Cerebrospinal fluid is sent to the lab. The CSF is centrifuged. A drop of India ink is placed on the slide along with a drop of the spun CSF. The India ink test is positive. Question What diagnostic sign may be found in this patient? Answer Choices 1 Kernig's sign 2 Babinski's sign 3 Kussmaul's sign 4 Quincke's sign 5 Chvostek's sign ANS:1

kerning sign Kernig's sign is a sign of meningeal irritation. The patient lies on his back, and his leg is raised and knee bent at a 90-degree angle. If there is pain or resistance to further extension when the examiner straightens the knee, Kernig's sign is said to be positive. This patient has signs and symptoms of cryptococcal meningitis. Cryptococcus neoformans is a yeast with a capsule made of polysaccharides. India ink exam is positive for Cryptococcus neoformans. When visualized on high dry magnification, encapsulated organisms (Cryptococcus neoformans) have capsules that look like halos because they exclude the India ink. Cryptococcus neoformans can cause meningitis, and there is an increased risk of Cryptococcus neoformansmeningitis in immunosuppressed individuals. Babinski's sign is an abnormal response to stimulation on the sole of the foot that results in dorsiflexion of the big toe and fanning of the other toes. Babinski's sign is seen with pyramidal disease. Kussmaul's sign is an increase in venous pressure during inspiration; it can be seen with cardiac tamponade. Quincke's sign is also called Quincke's pulse. This sign is seen in the nails. It consists of whitening and reddening of the nail bed coinciding with each heartbeat. Quincke's sign can be seen with aortic insufficiency. Chvostek's sign can be seen with tetany. When tapping on the facial nerve produces contraction on that side of the face, it is called Chvostek's sign.

A 24-year-old man presents with head trauma received in a motor vehicle accident (MVA) 30 minutes ago. Paramedics relate loss of consciousness for 1 minute, with a complete recovery. At the initial evaluation, the physical exam reveals a patient with a Glasgow Coma Scale (GCS) of 12, right hemiparesis, and a left fixed dilated pupil. What is the most probable finding in the brain CT scan? Answer Choices 1 A left epidural hematoma 2 A right epidural hematoma 3 A left subdural hematoma 4 A right subdural hematoma 5 A subarachnoid hemorrhage ANS:2

left epidural hematoma The patient presents with a history of head trauma, a brief initial period of unconsciousness, a lucid interval lasting minutes, subsequent deterioration in neurological status revealed in a Glasgow coma scale of 12, right hemiparesis, and left fixed dilated pupil. This is a classical presentation of left epidural hematoma, which is seen as a biconvex hyper-density (whiteness) on a CT scan. A right epidural hematoma would produce a left hemiparesis and a right fixed dilated pupil.

A 31-year-old man with a known generalized seizure disorder is brought to your emergency department. His friends tell you that the patient had a seizure and did not wake up. When he did not wake up after 30 minutes, his friends called 911. On examination, he is breathing and his heart is beating. He is warm, dry, and pink. His basic laboratory values are within normal limits, and the computerized axial tomography (C.A.T.) scan of his brain is unremarkable. An emergency electroencephalogram (E.E.G.) is not available. His only medications are phenytoin and phenobarbital. What is the best initial treatment in this case? Answer Choices 1 Glucose 2 Lorazepam (Ativan) 3 Phenytoin (Dilantin) 4 Phenobarbital 5 Craniotomy with burr hole placement by a neurosurgeon ANS:2

lorazepam (Ativan) According to consulting neurologist Wade S. Smith, Audio-Digest Family Practice, Vol. 49, 2001, "Consciousness is usually regained in a few minutes following a generalized seizure. When consciousness is not promptly regained following a seizure, then the diagnosis of status epilepticus becomes a reasonable consideration." When the emergency electroencephalogram (E.E.G.) is not readily available on a 24-hour basis, Professor Smith recommends that empiric pharmacotherapy for a working diagnosis of status epilepticus be provided. The first pharmacotherapy recommended for status epilepticus is lorazepam (Ativan). The second line pharmacotherapy for status epilepticus is phenytoin (Dilantin). The third line pharmacotherapy for status epilepticus is phenobarbital. Note that the absence of tonic-clonic movements does not exclude the diagnosis of status epilepticus, as it is possible that it will present with the solitary manifestation of persistent loss of consciousness. With his normal laboratory values, glucose would not be of value for this patient. Additionally, with the normal computerized axial tomography (C.A.T.) scan, the neurosurgical placement of burr holes by craniotomy would be nonfunctional

A 34-year-old woman, an immigrant from South America, presents with an acute onset of chest palpitations and shortness of breath. The patient also gives a 1-week history of fever, fatigue, and weakness. Echocardiography reveals that her heart is dramatically increased in size. Electrocardiogram (EKG) shows low voltage QRS with atrioventricular (AV) conduction abnormality. It is determined that she has Chagas' disease. Question What is a complication that occurs during the acute phase of this patient's condition? Answer Choices 1 Megaesophagus 2 Thromboembolism 3 Gastric dilatation 4 Meningoencephalitis 5 Megaureter ANS:4

meningoencephalitis Chagas' disease is also referred to as American trypanosomiasis. Chagas' disease is seen in the Western Hemisphere, primarily in South America, Central America, and Mexico. It is caused by the protozoa Trypanosoma cruzi. It can be acute or chronic. The manifestations in the acute phase include prolonged fever, tachycardia, fatigue, anemia, weakness, mild hepatosplenomegaly, and lymphadenopathy. Meningoencephalitis is the complication occurring during the acute phase. The acute stage resolves spontaneously by 4-8 weeks in 90% of the individuals who are infected. The persistence leads to development of intermittent or chronic phase. Chronic infection can lead to cardiomyopathy. The cardiac changes seen with chronic Chagas' disease include an enlarged heart and EKG findings of right bundle branch block and premature ventricular contractions. Chronic infection can also lead to megaesophagus, megacolon, and thromboembolism. Gastric dilatation and megaureter may occur, but they are rare.

A 32-year-old woman starts to notice that she is having difficulty brushing her hair. She also notices that her eyes are extremely tired after reading the evening newspaper. One night, her husband comments that she has droopy eyelids. She is relieved the next morning when her eyelids appear normal. Her condition deteriorates, and she has weakness and fatigability to the point that she finally sees her family doctor. On history, she denies pain. Her physical exam demonstrates weakness in her limbs. Her deep tendon reflexes are within normal limits. Her doctor wants to run a diagnostic test by injecting a drug. What is the most likely diagnosis? Answer Choices 1 Myasthenia gravis 2 Duchenne's dystrophy 3 Polymyositis 4 Familial periodic paralysis 5 Botulism ANS:1

myasthenia gravis This patient's symptoms indicate myasthenia gravis. Ocular muscle weakness and ptosis are common. Muscular weakness and fatigue are consistent with this condition. Myasthenia gravis occurs due to the presence of antibodies to the acetylcholine receptor. A diagnostic aid is the Tensilon test. The Tensilon test entails the administration of a rapid acting anticholinesterase, edrophonium. By administering an anticholinesterase, any acetylcholine released will be present longer and will have a longer period of time to act at the neuromuscular junction. Duchenne's dystrophy would present at a very young age. It is X-linked. Polymyositis can present with weakness. However, the weakness is proximal; it is usually of the hips and thigh, but sometimes it is of the shoulder girdles. Polymyositis would spare the ocular muscles. Familial periodic paralysis can present with weakness. However, the weakness would be in the extremities and the trunk. It would also spare the ocular muscles. Botulism can present with weakness. In addition, the ocular muscles can be involved. However, botulism would present with a progressive weakness, rather than an intermittent weakness. There would also be a loss of deep tendon reflexes. Furthermore, there is nothing in her history to suggest ingestion of botulism toxin.

A newborn baby boy is listless and febrile. He has diarrhea and vomiting as well. There is no nuchal rigidity, however CSF examination confirms meningitis. He is being treated with gentamicin and ampicillin. Question What is a common side effect he may experience? Answer Choices 1 Rash 2 Constipation 3 Thrombocytopenia 4 Leukopenia 5 Elevated liver enzymes ANS: 1

rash Explanation Skin rash is a common side effect of ampicillin. Rash occurs at a rate of approximately 3-7%. Constipation is not a side effect of ampicillin. Diarrhea is a side effect of ampicillin, rather than constipation. Thrombocytopenia, leukopenia, and elevated liver enzymes can be seen but less frequently than a rash. References:

The mechanism of sumatriptan in migraine involves what neurotransmitter? Answer Choices 1 Serotonin (5HT1) 2 Serotonin (5HT2) 3 Norepinephrine 4 Dopamine 5 Substance P

serotonin (5HT1) The headache and associative symptoms of migraine are postulated to result from the spontaneous activation of the trigeminovascular system. The peripheral fibers of the trigeminal nerve (ophthalmic division) supply innervation to the supratentorial meninges (dura mater, pia mater) and the intracranial blood vessels (arteries, veins, and venous sinuses). When activated, the peripheral axons of these bipolar neurons release vasoactive peptides (e.g. substance P, calcitonin gene-related protein, prostaglandins) adjacent to craniovascular structures, resulting in sterile inflammation, vasodilation, and pain. Central stimulation of the trigeminovascular system disrupts activity in the brainstem and hypothalamus, which is likely responsible for the associated nausea, vomiting, photophobia, and phonophobia of migraine. Treatment of migraine can be abortive or prophylactic. The trigeminovascular neurons are inhibited by serotonin 5HT1D- receptors, which are located both centrally and peripherally. Activation of these receptors aborts acute migraine headache and associative symptoms. Sumatriptan and dihydroergotamine (DHE) are specific 5HT1D- agonists used as abortive agents for migraine. Other medications used for acute migraine include acetaminophen, aspirin, naproxen sodium, ketorolac, caffeine, and butalbital. These drugs presumably act by reducing inflammation and promoting vasoconstriction within and around cranial blood vessels. Migraine prophylaxis is appropriate in patients who have frequent attacks (more than 3 per month) that interfere with social and occupational functioning. Prophylactic agents include tricyclic antidepressants (e.g. amitriptyline), β-blockers (e.g. propranolol, atenolol, nadolol), verapamil, and valproic acid.

An 11-year-old boy presents after waking with a new onset of right hemiparesis. He is also experiencing slurring of speech, double vision, numbness of the face, dizziness, neck pain, and headache. His cognition is normal. He had been playing hockey on the evening prior to presentation and was struck by a puck on his neck. He has no significant past medical history. On physical examination, blood pressure is 110/60 mmHg, pulse 72/minute, and respirations 16/minute. His heart sounds and breath sounds are normal. The power in the left upper and lower limbs is 3/5, with sensory loss on the left side of the face and arms. What caused the boy's hemiparesis? Answer Choices 1 Brachial plexus injury 2 Cervical cord injury 3 Conversion reaction 4 Stroke due to carotid dissection 5 Transverse myelitis

stroke due to carotid dissection Dissection of the craniocervical arteries is the most common non-atherosclerotic cause of strokes in young people. He probably has experienced a stroke due to carotid dissection. Blunt trauma to the neck, such as being struck with a hockey puck, can cause clotting at the site of the dissection, resulting in an embolic stroke in the area perfused by the middle cerebral artery. Carotid angiography in such patients reveals internal carotid arterial occlusion. This type of stroke is not uncommon in childhood, and it may also occur following intraoral trauma, chiropractic manipulation, and cervical spinal injury following motor vehicle accidents. It usually presents with neurological deficits preceded by headache and neck pain. The patient may present with dizziness, dysarthria, diplopia, nystagmus, numbness, and features of Horner's syndrome. Diagnosis is by imaging methods such as Magnetic Resonance Imaging (MRI), Computed Tomography (CT), or Magnetic Resonance Angiography (MRA). Initial treatment is with anticoagulants. An injury of the cervical cord should cause bilateral symptoms and possibly respiratory compromise, rather than hemiparesis and sensory loss over the face and arms. A lesion of the brachial plexus should not cause weakness of the leg. Transverse myelitis usually causes bilateral symptoms, and a specific sensory level deficit is present. Conversion reaction is a psychopathological condition characterized by the presence of bodily symptoms that have no discernable physical cause. Conversion reaction is a diagnosis of exclusion and would not be the most likely explanation of these findings. Another risk factor for stroke in childhood is sickle cell disease. The incidence of stroke among children who have sickle cell disease is estimated to be as high as 6%. Transfusion to decrease the percentage of hemoglobin S has been found to lower that risk significantly. Hemorrhagic stroke in childhood may be due to arteriovenous malformations, arterial aneurysm, or cavernomas. Underlying hematologic conditions, including coagulopathy (e.g. hemophilia) or thrombocytopenia (e.g. idiopathic thrombocytopenic purpura), also may be a cause. Strokes have also been associated with cocaine use or trauma. Despite careful and complete evaluation, the etiology in some cases remains cryptogenic.

A 67-year-old man with an 80-pack/year smoking history and chronic bronchitis presents 30 minutes after suffering what was described as a "seizure" by his wife. According to his wife, this was his first seizure. During the seizure episode, the patient did not lose consciousness, but was observed to have developed an "involuntary jerking in his right arm and leg" that lasted about 5 minutes, with right arm weakness immediately following this involuntary activity. On physical exam, the patient appears drowsy, confused, and answers questions slowly. His right arm has objective weakness rated as 4/5; the remainder of the neurological exam is normal. Question Ico-delete Highlights What aspect of this patient's presentation most closely identifies this event as a simple focal seizure? Answer Choices 1 Involuntary extremity jerking 2 Right arm weakness is noted 3 There was no impairment of consciousness 4 There is postictal confusion 5 First seizure as an adult ANS: 3

there was no impairment of consciousness

A 19-year-old woman presents with worsening headaches. She reports a multi-year history of episodic, throbbing headaches. They have intensified, and she now misses classes and work periodically due to her headaches. Recently, the headaches occur about 4-6 times per month, up from 1-2 per month when she first started experiencing headaches. Her headaches last 2-3 days duration and are accompanied by nausea, vomiting, and light sensitivity. After the headache resolves, she denies any residual symptoms. She denies neurologic symptoms, such as vision or taste changes, gait disturbances, and memory loss. She has tried multiple over-the-counter pain medications without relief. Her mother and maternal aunt experienced similar headaches. She reports some increased stressors and less sleep since starting college. Her past medical history is unremarkable, with no known medical conditions (except for the headaches), no surgeries, no chronic medications, and no drug allergies. She has never been sexually active and reports regular menses. She denies the use of tobacco, alcohol and drugs. On physical exam, the patient appears comfortable and reports no headache at this time. Her entire exam, including neurological, is normal. Question In addition to treatment options for acute headache, this patient should be educated about which of the following daily medications to prevent headache episodes? Answer Choices 1 Cetirizine 2 Clopidogrel 3 Hydrocodone 4 Sumatriptan 5 Topiramate

topiramate This patient is suffering from migraine headaches, common headaches classically described as throbbing in nature and accompanied by nausea (and possibly vomiting), photophobia (sensitivity to light), phonophobia (sensitivity to sounds), and possibly visual changes and aura. Migraines are more common in women than men, but can occur in men. They are usually a chronic condition, beginning in adolescence or early adult years. Often a family history is present. Stress, sleep deprivation, and various foods can be among many triggers for migraines. With a classic history and no alarm features of a more serious condition, no testing would be necessary. Treatment approach includes offering treatment for acute headache and, with frequent migraine such as this patient's, prophylactic treatments, such as topiramate, propranolol, amitriptyline, valproic acid, and botulinum toxin type A. Topiramate augments GABA activity and has primarily been used for seizure disorders. It is approved and effective for prevention of migraine headaches.

A 44-year-old man presents after an episode what is described as "passing out". He has a past medical history of hypertension, diabetes mellitus type II, and osteoarthritis. Earlier in the day, the patient visited his primary care physician for routine blood work. During his blood draw, the patient had an acute onset of syncope. The patient never had such an episode before. Vital signs upon presentation are as follows: BP 124/82 mmHg, P 88 beats/min, R14/min. EKG is within normal limits. Cardiovascular examination reveals a normal S1 and S2, with no rubs, murmurs, or gallops. The patient is being treated with atenolol and hydrochlorothiazide for his hypertension. His blood sugar is 96 mg/dl. Question What is the most likely cause of the patient's syncope? Answer Choices 1 Hypoglycemic episode 2 Arrhythmia 3 Neurogenic shock 4 Vasovagal episode 5 Orthostatic hypotension ANS:4

vasovagal episode This patient is having syncope due to a vasovagal episode. This reaction is common in patients who are exposed to blood or are having their blood drawn. In fact, vasovagal episodes are the most common cause of syncope. These individuals typically present with loss of consciousness when exposed to a certain trigger. The trigger leads to an activation of the nucleus tract solitarius of the brainstem, which results in a surge of parasympathetic response, leading to the cardioinhibitory and vasodepressor effects. Although this patient has a history of diabetes mellitus, is not currently hypoglycemic. Hypoglycemia, if severe, can potentially lead to a loss of consciousness. However, diabetics who are hypoglycemic typically have a sympathetic response. Arrhythmia is not the correct answer choice. This patient does not have any cardiac history of arrhythmia. As the patient had the syncopal episode during the blood draw, and there were no ECG changes, this answer choice is unlikely. Orthostatic hypotension is a reasonable answer choice. However, this patient's acute syncopal episode in response to the blood draw makes a vasovagal reaction more likely. In addition, this patient has no history of dehydration or any other cause that would lead to this condition.

A 35-year-old man presents with paralysis and confusion. His wife states her husband had been working in the yard about 3 days prior to presentation and suddenly began to not feel well. She states that it has progressed to the point where he appears quite confused, is extremely agitated, and does not seem to be able to close his right eye completely or smile on the right side of his face. The patient has a rash on the right arm that does not seem to be pruritic. Physical examination of the patient reveals findings of altered mental status, a cranial nerve palsy of the facial nerve (cranial nerve VII), exaggerated deep tendon reflexes, and the presence of a maculopapular rash on the right forearm. Question What is the most likely source of this patient's condition? Answer Choices 1 Mumps 2 West Nile virus 3 St. Louis virus 4 Rabies 5 Varicella-zoster virus ANS:2

west nile virus This patient has encephalitis secondary to exposure to the West Nile virus. Patients with encephalitis, regardless of the cause, will present with altered mental status that has a huge variability (from subtle deficits to 100% unresponsiveness). Meningeal irritation may also be seen if involvement of the meninges is present (e.g., photophobia and nuchal rigidity). Seizures will also be seen, as well has hemiparesis, cranial nerve palsies, exaggerated deep tendon, or even pathologic reflexes. Patients will appear confused, agitated, or obtunded. The most common cause of viral encephalitis in the United States is the West Nile virus. Many times, patients may be misdiagnosed as having Guillain-Barre syndrome because of the presence of the symptom of flaccid paralysis that is specific to this infection. Another key characteristic of this encephalitis is the presence of a nonpruritic maculopapular rash, estimated to be seen in up to 50% of patients with this type of infection. This patient most likely has encephalitis secondary to the West Nile virus. A patient who is unvaccinated and presents with parotitis and altered mental status will more likely have encephalitis secondary to mumps. If a patient has tremors of the eyelids, tongue, lips, and extremities, the most likely cause is the St. Louis virus. Findings such as hydrophobia, pharyngeal spasm, and hyperactivity would be seen in a patient with rabies-induced encephalitis. A patient who has encephalitis from the varicella-zoster virus will more commonly have the dermatomal pattern of grouped vesicles that is typically seen in a zoster infection.


Related study sets

N317- Musculoskeletal System Practice Questions

View Set

what are three methods of data collection?

View Set

Neuroscience 4: Ventricular System of the Brain

View Set

Week 2: DNA Replication & Mitosis

View Set

Chp 6 bone remodeling &factors affecting bone growth

View Set

Vomiting (Emesis) + Anti-Emetics

View Set

Chapter 2 - Lewis Acids and Bases

View Set

chapter 9, 19, 6 study guide 47, 4 exam 4

View Set